PDA

نسخه کامل مشاهده نسخه کامل : اتاق ریاضیات(طرح سؤالات)



صفحه ها : 1 2 3 4 5 6 7 8 [9] 10 11 12 13 14 15 16 17 18 19 20

davy jones
30-06-2009, 10:54
ایول بابا. سرعت عملتون رو عشقست.:31:
میتونین راه حل ریاضی و کلاسیکی برای این مساله ارائه بدین؟ چون تقریبا هر دوی شما از راه استقرا مساله رو حل کردین.

موفق باشین.
88/4/9

davy jones
30-06-2009, 11:39
حالا که اینقدر پایه این یه سوال سخت تر براتون میذارم.
این سوال رو هنگامی که مشغول بازی بسیار مفرح مافیا بودیم یکی مطرح کرد:
فرض کنید 100 نفر رو در اختیار دارید که 49 نفر از اونها روی پیشونیشون یه خال مشخص دارن و 51 تای بقیه مث آدمای عادی اند. اون 49 نفر خودشون از وجود خال روی پیشونیشون بی اطلاعند و فرض میکنیم که چیزی به نام آینه هم هنوز اختراع نشده! این صد نفر روزی یکبار دور هم جمع میشن و فقط و فقط همدیگر رو نگاه میکنن و به هیچ وجه نه با هم حرف میزنن و نه با اشاره به همدیگه میفهمونن که تو خال داری یا نه. هر نفر فقط میتونه ببینه که چند نفر خال دارن ولی نمیتونه به اون افراد مستقیما یا با اشاره چیزی بفهمونه. حال فرض میکنیم داشتن خال چیز خیلی بدی باشه به حدی که اگه هر کس به هر نحوی ( باتوجه به ممنوعیتهای ذکر شده ) بفهمه که خال داره خودشو میکشه و دیگه از فردای اون روز تو جلسات روزانه شرکت نمیکنه. بقیه افراد هم متوجه این موضوع هستند که اگر فردی در جلسه حاضر نشد مطمئنا به خالدار بودن خود پی برده و خودکشی کرده. این صد نفر نمیدونن که دقیقا چند آدم خالدار وجود داره. تنها چیزی که میدونن اینه که حداقل یه خالدار وجود داره.
حال سوال اینه که بعد از گذشت چند روز همه ی افراد خالدار متوجه میشن و خودکشی میکنن و بقیه هم از این که خال ندارن مطمئن میشن؟

بازم تکرار میکنم که جوابشو فقط به شرطی میزارم که روش فکر و بحث بشه.

موفق باشین.
88/4/9

davy jones
30-06-2009, 15:10
یه سوال دیگه (نسبتا راحت):
جواب این انتگرال چی میشه:
[ برای مشاهده لینک ، لطفا با نام کاربری خود وارد شوید یا ثبت نام کنید ]
موفق باشین.
88/4/9

mahsa1469
30-06-2009, 15:43
حالا که اینقدر پایه این یه سوال سخت تر براتون میذارم.
این سوال رو هنگامی که مشغول بازی بسیار مفرح مافیا بودیم یکی مطرح کرد:
فرض کنید 100 نفر رو در اختیار دارید که 49 نفر از اونها روی پیشونیشون یه خال مشخص دارن و 51 تای بقیه مث آدمای عادی اند. اون 49 نفر خودشون از وجود خال روی پیشونیشون بی اطلاعند و فرض میکنیم که چیزی به نام آینه هم هنوز اختراع نشده! این صد نفر روزی یکبار دور هم جمع میشن و فقط و فقط همدیگر رو نگاه میکنن و به هیچ وجه نه با هم حرف میزنن و نه با اشاره به همدیگه میفهمونن که تو خال داری یا نه. هر نفر فقط میتونه ببینه که چند نفر خال دارن ولی نمیتونه به اون افراد مستقیما یا با اشاره چیزی بفهمونه. حال فرض میکنیم داشتن خال چیز خیلی بدی باشه به حدی که اگه هر کس به هر نحوی ( باتوجه به ممنوعیتهای ذکر شده ) بفهمه که خال داره خودشو میکشه و دیگه از فردای اون روز تو جلسات روزانه شرکت نمیکنه. بقیه افراد هم متوجه این موضوع هستند که اگر فردی در جلسه حاضر نشد مطمئنا به خالدار بودن خود پی برده و خودکشی کرده. این صد نفر نمیدونن که دقیقا چند آدم خالدار وجود داره. تنها چیزی که میدونن اینه که حداقل یه خالدار وجود داره.
حال سوال اینه که بعد از گذشت چند روز همه ی افراد خالدار متوجه میشن و خودکشی میکنن و بقیه هم از این که خال ندارن مطمئن میشن؟

بازم تکرار میکنم که جوابشو فقط به شرطی میزارم که روش فکر و بحث بشه.

موفق باشین.
88/4/9
این یکم سخت شد من که یکم باید فکر کنم جوابش رو زود تذار:20::46:

davy jones
30-06-2009, 16:07
این یکم سخت شد من که یکم باید فکر کنم جوابش رو زود نذار:20::46:

باشه. ولی تا هر جایی که رسیدی بقیه رو هم در جریان بذار.

ask_bl
30-06-2009, 19:18
یه سوال دیگه (نسبتا راحت):
جواب این انتگرال چی میشه:
[ برای مشاهده لینک ، لطفا با نام کاربری خود وارد شوید یا ثبت نام کنید ]
موفق باشین.
88/4/9



[ برای مشاهده لینک ، لطفا با نام کاربری خود وارد شوید یا ثبت نام کنید ]

mahsa1469
30-06-2009, 19:58
(چرا هیچ کس نمیاد؟!)
من هم میتونم سوال مطرح کنم تا بقیه حل کنن؟!
اگه آره .... سوال هاش میتونه غیر ریاضی(هوش) هم باشه.
خوب شما هم مطرح کن
ولی خودمونیما بهتر نیست یه تاپیک جدگونه داشته باشه تا اینجا فقط رفع اشکال بشه؟؟:46:

davy jones
01-07-2009, 07:51
[ برای مشاهده لینک ، لطفا با نام کاربری خود وارد شوید یا ثبت نام کنید ]


ممنونم دوست عزیز ولی جواب شما غلطه چون توان n مربوط به کل لگاریتمه. فقط وقتی که x داخل آرکومان لگاریتم به یه توانی برسه میشه اون رو به عنوان ضریب آورد پشت لگاریتم.

اگه میتونی بازم روش فکر کن.

موفق باشین.
88/4/10

davy jones
01-07-2009, 07:56
(چرا هیچ کس نمیاد؟!)
من هم میتونم سوال مطرح کنم تا بقیه حل کنن؟!
اگه آره .... سوال هاش میتونه غیر ریاضی(هوش) هم باشه.


خوب شما هم مطرح کن
ولی خودمونیما بهتر نیست یه تاپیک جدگونه داشته باشه تا اینجا فقط رفع اشکال بشه؟؟[ برای مشاهده لینک ، لطفا با نام کاربری خود وارد شوید یا ثبت نام کنید ]

سلام دوستان عزیز
ببخشید که دیر اومدم. من اکانت شبانه ندارم و متاسفانه روزها تا ساعت 5 بعد از ظهر بیشتر نمیتونم در خدمتتون باشم.
به خاطر اینکه تاپیک خیلی بی بخار شده بهتره برای اینکه موقتا از این حالت در بیاد هر کی هر سوالی داره (حتی هوش) همینجا طرح کنه ولی مثلا تا وقتی که همزمان 3 تا سوال مطرحه از طرح سوالات بعدی خودداری گنین.

موفق باشین.
88/4/10

davy jones
01-07-2009, 08:08
من تو حل سوال به چند تا چیز برخوردم(اولی رو جدی نگیرید):
1.مگه اون یارو ها دست ندارن که به پیشونیشون دست بزنن!
2.اگر یه نفر از اونا(خال دار) بفهمه پس چون شرایطش با بقیه ی خال دار ها برابره پس در همون روز همه ی خال دار ها خود کشی میکنن.
3.با شرایط سوال هیچ کدوم هیچ وقت نمی فهمه که خال داره.
4. یا من منظور سوال رو نفهمیدم یا سوال مشکل داره.\/
(البته اینا فقط نظر منه)

دوست عزیزم! شما در نکته ی دومی که بهش اشاره کردی به جواب خیلی نزدیک شدی.

موفق باشین.
88/4/10

davy jones
01-07-2009, 08:51
میتونین راه حل ریاضی و کلاسیکی برای این مساله ارائه بدین؟ چون تقریبا هر دوی شما از راه استقرا مساله رو حل کردین.

موفق باشین.
88/4/9

ظاهرا کسی این رو جدی نگرفته!
خیلی خب، خودم جواب کلاسیکش رو میذارم:

میدونیم که هر عدد طبیعی رو میشه به صورت حاصلضرب چند عدد اول که هر کدام از آنها توانی دارند نوشت. مثلا:

[ برای مشاهده لینک ، لطفا با نام کاربری خود وارد شوید یا ثبت نام کنید ]
و...
در حالت کلی هر عدد طبیعی مثل N رو میشه به این صورت نوشت:
[ برای مشاهده لینک ، لطفا با نام کاربری خود وارد شوید یا ثبت نام کنید ]
که توانها میتونن صفر هم باشن بدان معنا که اگه توان یکی از پایه ها (که اعداد اول هستن) صفر بود ، آن عدد اول و توانهاش جزء مقسوم علیه های عدد N نیست.
بعد از یادآوری مقدمه بالا به این نکته دقت میکنیم که هر مقسوم علیه دلخواه عدد N بدین صورت به دست میآد که میتونه از هر عدد اولی مانند p هیچی نباشه، یا فقط یکی باشه یا دوتا یا .... یا آلفا تا.
مثلا در عدد 20 که در بالا مشاهده میکنین هر مقسوم علیه دلخواه میتونه فقط از عناصر 2 و 5 تشکیل بشه( تا اینجاش که واضحه) و در مورد 5 میتونه هیچی 5 توش نباشه یا فقط یکبار عامل 5 توش ظاهر بشه ( یعنی 5 به توان 1) و در مورد 2 هم میتونه اصلا نباشه یا یکبار باشه یا دوبار باشه ( یعنی 2 به توان 2 هم میتونه باشه)
پس هر کدوم از اعداد اول [ برای مشاهده لینک ، لطفا با نام کاربری خود وارد شوید یا ثبت نام کنید ]حالت دارد. پس در کل داریم:
[ برای مشاهده لینک ، لطفا با نام کاربری خود وارد شوید یا ثبت نام کنید ]
که x تعداد کل مقسوم علیه های عدد N هستش. ( چون اعداد اول اینجا برای تشکیل یک مقسوم علیه دلخواه از هم مستقلند پس تعداد حالات کل حاصلضرب تعداد حالات هر یک از اعداد اوله)
حال مساله ما وقتی جواب داره که تعداد مقسوم علیه ها عددی فرد باشه. یعنی x فرده. میدونیم که وقتی چند عدد تو هم ضرب میشن اگه تنها یکیشون هم زوج باشه کل حاصلضرب رو زوج میکنه ( میتونید امتحان کنید ولی واضحه) پس تک تک اون پرانتزها باید عدد فرد باشن تا x هم فرد باشه. بنابرین همه ی آلفا ها باید زوج باشن تا وقتی که با یک جمع میشن عدد فرد بشن. و این یعنی عددی که تعداد مقسوم علیه هاش فرد هستش توان همه ی اعداد اولی که تجزیه ی اون رو میسازن باید زوج باشه و این یعنی اون عدد مربع کامله!

موفق باشین.
88/4/10

mahsa1469
01-07-2009, 09:16
حالا که اینقدر پایه این یه سوال سخت تر براتون میذارم.
این سوال رو هنگامی که مشغول بازی بسیار مفرح مافیا بودیم یکی مطرح کرد:
فرض کنید 100 نفر رو در اختیار دارید که 49 نفر از اونها روی پیشونیشون یه خال مشخص دارن و 51 تای بقیه مث آدمای عادی اند. اون 49 نفر خودشون از وجود خال روی پیشونیشون بی اطلاعند و فرض میکنیم که چیزی به نام آینه هم هنوز اختراع نشده! این صد نفر روزی یکبار دور هم جمع میشن و فقط و فقط همدیگر رو نگاه میکنن و به هیچ وجه نه با هم حرف میزنن و نه با اشاره به همدیگه میفهمونن که تو خال داری یا نه. هر نفر فقط میتونه ببینه که چند نفر خال دارن ولی نمیتونه به اون افراد مستقیما یا با اشاره چیزی بفهمونه. حال فرض میکنیم داشتن خال چیز خیلی بدی باشه به حدی که اگه هر کس به هر نحوی ( باتوجه به ممنوعیتهای ذکر شده ) بفهمه که خال داره خودشو میکشه و دیگه از فردای اون روز تو جلسات روزانه شرکت نمیکنه. بقیه افراد هم متوجه این موضوع هستند که اگر فردی در جلسه حاضر نشد مطمئنا به خالدار بودن خود پی برده و خودکشی کرده. این صد نفر نمیدونن که دقیقا چند آدم خالدار وجود داره. تنها چیزی که میدونن اینه که حداقل یه خالدار وجود داره.
حال سوال اینه که بعد از گذشت چند روز همه ی افراد خالدار متوجه میشن و خودکشی میکنن و بقیه هم از این که خال ندارن مطمئن میشن؟

بازم تکرار میکنم که جوابشو فقط به شرطی میزارم که روش فکر و بحث بشه.

موفق باشین.
88/4/9
خیلی روش فکر کردم اما با روش های ریاضیاتی به نظر نمی یاد بشه به نتیجه ای رسید :13:خوب از اون جایی که که هیچ کدومشون نمیدونن خال دارن یا نه و حتی نمیدونن که چند نفر خالدار هست و خالدار بودن یه چیز خیلی بدیه و تنها چیزی که می دونن اینه که دست کم یه خالدار وجود داره با دیدن این همه خالدار این شبهه براشون به وجود می یاد که نکنه اونم خالدار باشه بعد می ره خودکشی میکنه:31:پس همون روز اول همشون می میرن:41::41:

davy jones
01-07-2009, 09:25
خیلی روش فکر کردم اما با روش های ریاضیاتی به نظر نمی یاد بشه به نتیجه ای رسید :13:خوب از اون جایی که که هیچ کدومشون نمیدونن خال دارن یا نه و حتی نمیدونن که چند نفر خالدار هست و خالدار بودن یه چیز خیلی بدیه و تنها چیزی که می دونن اینه که دست کم یه خالدار وجود داره با دیدن این همه خالدار این شبهه براشون به وجود می یاد که نکنه اونم خالدار باشه بعد می ره خودکشی میکنه:31:پس همون روز اول همشون می میرن:41::41:
کسی با خیال کردن خودکشی نمیکنه.:41: فقط وقتی که مطمئن میشه دست به خودکشی میزنه.:46:
برای اینکه یکم راهنماییتون کرده باشم از استقرا کمک بگیرین. یعنی با تعداد کم شروع کنین.
ضمنا اون انتگرالی رو هم که گذاشتم رو هم حل کنین.( پست شماره ی 2010)

موفق باشین.
88/4/10

behnam_tr
01-07-2009, 11:47
یه سوال دیگه (نسبتا راحت):
جواب این انتگرال چی میشه:
[ برای مشاهده لینک ، لطفا با نام کاربری خود وارد شوید یا ثبت نام کنید ]
موفق باشین.
88/4/9

شما اول باید انتگرال را به صورت xlnx dx بنویسید بعد با روش انتگرای جز به جز پیش برید ........

davy jones
01-07-2009, 12:06
شما اول باید انتگرال را به صورت xlnx dx بنویسید بعد با روش انتگرای جز به جز پیش برید ........

خب چجوری باید این کار رو بکنیم.:31:
ضمنا من خودم جواب سوال رو میدونم. چون خودم طرحش کردم.:46::46:

موفق باشین.
88/4/10

davy jones
01-07-2009, 12:08
من تازه دوم دبیرستانم و انتگرال بلد نیستم.[ برای مشاهده لینک ، لطفا با نام کاربری خود وارد شوید یا ثبت نام کنید ]

اوووووووووووووخیییییییییی یییییییییییییش....:14::14::14::14::1 4::14::14::14::14::14::14::14::14::14::14::14:
:34::34::34::34::34::34::34::34::34::34::34::34::3 4::34:

موفق باشین.
88/4/10

behnam_tr
01-07-2009, 19:29
خب چجوری باید این کار رو بکنیم.:31:
ضمنا من خودم جواب سوال رو میدونم. چون خودم طرحش کردم.:46::46:

موفق باشین.
88/4/10

این خیلی آسون بود از این سخترهاشم حریفم !!!
فقط عکس بده جنازه تحویل بگیر !!!:31:


[ برای مشاهده لینک ، لطفا با نام کاربری خود وارد شوید یا ثبت نام کنید ]

mahsa1469
02-07-2009, 13:40
این خیلی آسون بود از این سخترهاشم حریفم !!!
فقط عکس بده جنازه تحویل بگیر !!!:31:


[ برای مشاهده لینک ، لطفا با نام کاربری خود وارد شوید یا ثبت نام کنید ]



چرا اين n رفته پشت:13:

mahsa1469
02-07-2009, 13:58
یه سوال دیگه (نسبتا راحت):
جواب این انتگرال چی میشه:
[ برای مشاهده لینک ، لطفا با نام کاربری خود وارد شوید یا ثبت نام کنید ]
موفق باشین.
88/4/9
اينم جواب:(انتگرال رو از راه جز به جز گرفتم!)



[ برای مشاهده لینک ، لطفا با نام کاربری خود وارد شوید یا ثبت نام کنید ]

behnam_tr
02-07-2009, 14:08
چرا اين n رفته پشت:13:

چون n یه عدد هست و میتونه از انتگرال بیاد بیرون :8:

behnam_tr
02-07-2009, 14:11
اينم جواب:(انتگرال رو از راه جز به جز گرفتم!)



[ برای مشاهده لینک ، لطفا با نام کاربری خود وارد شوید یا ثبت نام کنید ]


میشه یه کم توضیح بدی چجوری حل شده :18: پس In-1 کو ؟؟؟

mahsa1469
02-07-2009, 14:11
چون n یه عدد هست و میتونه از انتگرال بیاد بیرون :8:
منظورم اينه كه n توان كل ln هستش و نمي تونه به عنوان ضريب بره پشت؟؟(توي همون خط اول):20:

mahsa1469
02-07-2009, 14:15
میشه یه کم توضیح بدی چجوری حل شده :18: پس In-1 کو ؟؟؟
چون اين جا مقداري كه مي خوايم ازش انتگرال بگيريم به توان n رسته بهترين راه به دست آوردن فرمول كاهشي اون هستش In-1 همون انتگراله هست كه به جاي n-1،n قرار گرفته و كافيه جاي n عدد بذاريم و از راه فرمول داده شده انتگرال رو حساب كنيم:20:

behnam_tr
02-07-2009, 14:32
منظورم اينه كه n توان كل ln هستش و نمي تونه به عنوان ضريب بره پشت؟؟(توي همون خط اول):20:

:27::27::27::27::27::27::27:

پس من اشتباه حل کردم نه ؟؟؟؟:5:

behnam_tr
02-07-2009, 14:34
[ برای مشاهده لینک ، لطفا با نام کاربری خود وارد شوید یا ثبت نام کنید ]میشه یکم در مورد این حل انتگرال به روش اندیس کاهش رو بگی ::: فرمول کلی با یه مثال ::

[ برای مشاهده لینک ، لطفا با نام کاربری خود وارد شوید یا ثبت نام کنید ]مرسی

mahsa1469
02-07-2009, 17:25
[ برای مشاهده لینک ، لطفا با نام کاربری خود وارد شوید یا ثبت نام کنید ]میشه یکم در مورد این حل انتگرال به روش اندیس کاهش رو بگی ::: فرمول کلی با یه مثال ::

[ برای مشاهده لینک ، لطفا با نام کاربری خود وارد شوید یا ثبت نام کنید ]مرسی
اين توي اكثر كتاب ديفرانسيل ها هست مطلب زير رو از روي يكي از همين كتابها براتون نوشتم
فرمول هاي كاهشي
مساله ي يافتن انتگرال [ برای مشاهده لینک ، لطفا با نام کاربری خود وارد شوید یا ثبت نام کنید ]را در نظر بگيريد . البته مي توان اين انتگرال را با 10 بار جز به جز به سرانجام رساند ولي روش زير ترجيح دارد.
فرض كنيد براي [ برای مشاهده لینک ، لطفا با نام کاربری خود وارد شوید یا ثبت نام کنید ]

[ برای مشاهده لینک ، لطفا با نام کاربری خود وارد شوید یا ثبت نام کنید ]


مي خواهيم [ برای مشاهده لینک ، لطفا با نام کاربری خود وارد شوید یا ثبت نام کنید ]را پيدا كنيم. اگر به روش جزبهجز انتگرال بگيريم،فرمول [ برای مشاهده لینک ، لطفا با نام کاربری خود وارد شوید یا ثبت نام کنید ]را برحسب[ برای مشاهده لینک ، لطفا با نام کاربری خود وارد شوید یا ثبت نام کنید ]به دست مي آوريم.
[ برای مشاهده لینک ، لطفا با نام کاربری خود وارد شوید یا ثبت نام کنید ] [ برای مشاهده لینک ، لطفا با نام کاربری خود وارد شوید یا ثبت نام کنید ]قرا ر دهيد

[ برای مشاهده لینک ، لطفا با نام کاربری خود وارد شوید یا ثبت نام کنید ]

فرمول

[ برای مشاهده لینک ، لطفا با نام کاربری خود وارد شوید یا ثبت نام کنید ]

به نام فرمول كاهشي ناميده مي شود ، يرا مقدار انتگرال[ برای مشاهده لینک ، لطفا با نام کاربری خود وارد شوید یا ثبت نام کنید ]را برحسب [ برای مشاهده لینک ، لطفا با نام کاربری خود وارد شوید یا ثبت نام کنید ]مي دهد.

behnam_tr
02-07-2009, 17:51
اين توي اكثر كتاب ديفرانسيل ها هست مطلب زير رو از روي يكي از همين كتابها براتون نوشتم
فرمول هاي كاهشي
مساله ي يافتن انتگرال [ برای مشاهده لینک ، لطفا با نام کاربری خود وارد شوید یا ثبت نام کنید ]را در نظر بگيريد . البته مي توان اين انتگرال را با 10 بار جز به جز به سرانجام رساند ولي روش زير ترجيح دارد.
فرض كنيد براي [ برای مشاهده لینک ، لطفا با نام کاربری خود وارد شوید یا ثبت نام کنید ]

[ برای مشاهده لینک ، لطفا با نام کاربری خود وارد شوید یا ثبت نام کنید ]


مي خواهيم [ برای مشاهده لینک ، لطفا با نام کاربری خود وارد شوید یا ثبت نام کنید ]را پيدا كنيم. اگر به روش جزبهجز انتگرال بگيريم،فرمول [ برای مشاهده لینک ، لطفا با نام کاربری خود وارد شوید یا ثبت نام کنید ]را برحسب[ برای مشاهده لینک ، لطفا با نام کاربری خود وارد شوید یا ثبت نام کنید ]به دست مي آوريم.
[ برای مشاهده لینک ، لطفا با نام کاربری خود وارد شوید یا ثبت نام کنید ] [ برای مشاهده لینک ، لطفا با نام کاربری خود وارد شوید یا ثبت نام کنید ]قرا ر دهيد

[ برای مشاهده لینک ، لطفا با نام کاربری خود وارد شوید یا ثبت نام کنید ]

فرمول

[ برای مشاهده لینک ، لطفا با نام کاربری خود وارد شوید یا ثبت نام کنید ]

به نام فرمول كاهشي ناميده مي شود ، يرا مقدار انتگرال[ برای مشاهده لینک ، لطفا با نام کاربری خود وارد شوید یا ثبت نام کنید ]را برحسب [ برای مشاهده لینک ، لطفا با نام کاربری خود وارد شوید یا ثبت نام کنید ]مي دهد.


:27:مرسی
:13:همون جز به جز دیگه !!!

behnam_tr
02-07-2009, 18:03
منم برات چندتا انتگرال طرح کردم ببین میتونی حلشون کنی منتظر جواب هات هستم :::

[ برای مشاهده لینک ، لطفا با نام کاربری خود وارد شوید یا ثبت نام کنید ]

برای مشاهده محتوا ، لطفا وارد شوید یا ثبت نام کنید

mahsa1469
02-07-2009, 18:56
:27:مرسی
:13:همون جز به جز دیگه !!!
آره ديگه:31:


منم برات چندتا انتگرال طرح کردم ببین میتونی حلشون کنی منتظر جواب هات هستم :::

[ برای مشاهده لینک ، لطفا با نام کاربری خود وارد شوید یا ثبت نام کنید ]

برای مشاهده محتوا ، لطفا وارد شوید یا ثبت نام کنید
حتما روشون فكر مي كنم:11:

amin_
03-07-2009, 04:05
بچه ها من به دنبال یک الگوریتم برای محاسبه مقادیر ویژه و بردارهای ویژه حقیقی یا مختلط یک ماتریس هستم . آیا کسی هست مرا یاری کند.

می خواهم برنامه ای بر این اساس بنویسم. اگر الگوریتمی به من معرفی کنید ممنون میشم

dropdead
03-07-2009, 12:53
من تازه دارم ميرم دوم دبيرستان... [ برای مشاهده لینک ، لطفا با نام کاربری خود وارد شوید یا ثبت نام کنید ]
ا.مدم اينجا سرم گيج رفت...[ برای مشاهده لینک ، لطفا با نام کاربری خود وارد شوید یا ثبت نام کنید ]

behnam_tr
03-07-2009, 13:05
من تازه دارم ميرم دوم دبيرستان... [ برای مشاهده لینک ، لطفا با نام کاربری خود وارد شوید یا ثبت نام کنید ]
ا.مدم اينجا سرم گيج رفت...[ برای مشاهده لینک ، لطفا با نام کاربری خود وارد شوید یا ثبت نام کنید ]

[ برای مشاهده لینک ، لطفا با نام کاربری خود وارد شوید یا ثبت نام کنید ]از کدوم مبحث ها عزیزم ؟؟؟

zahedy2006
03-07-2009, 13:28
بچه ها من به دنبال یک الگوریتم برای محاسبه مقادیر ویژه و بردارهای ویژه حقیقی یا مختلط یک ماتریس هستم . آیا کسی هست مرا یاری کند.

می خواهم برنامه ای بر این اساس بنویسم. اگر الگوریتمی به من معرفی کنید ممنون میشم
سلام
برنامه كه تا دلتون بخواهد هست

كافيه eigen value program و eigen vector سرچ كنيد نتيجه بهتون مي ده

در برنامه maple هم كافيه ماتريس رو تعريف كنيد و دستور يكي از دو عبارت بالا رو بنويسيد (تو هلپ خود ميپل هست)

يه چندتا برنامه 400 - 500 كيلوبايتي داشتم. الان پيداشون نمي كنم. پيدا شد مي گذارم

behnam_tr
03-07-2009, 15:25
[ برای مشاهده لینک ، لطفا با نام کاربری خود وارد شوید یا ثبت نام کنید ]انتگرال بدید حل کنیم بابا بیکاریم الان ..........

davy jones
05-07-2009, 08:05
سلام به همگی، دوباره برگشتم.:26:
ولی از اینکه دیر برگشتم عذر میخوام.:42:
مدتی کمرنگ بودم.:41:
اول بریم سر حل اون انتگرال که انگار سر و صدای زیادی کرده:27::
اینم جوابش:
[ برای مشاهده لینک ، لطفا با نام کاربری خود وارد شوید یا ثبت نام کنید ]

در مورد سوال آدمای خالدار هم انگار همگی به بن بست رسیدین.
خب عیبی نداره. همونطور که گفته بودم برای شروع بهتره که از تعداد کم شروع کنیم. فرض کنید که از بین 3 نفر، یکی خال داشته باشه. همون روز اول کسی که خالداره چون خالدار دیگه ای رو نمیبینه و چون مطمئنه که حداقل یه خالدار وجود داره پس همون روز اول خودشو میکشه. اون دو نفر دیگه هم روز اول میبینن که یک خالدار وجود داره و منتظرن که اون یارو خودشو بکشه.
حال فرض کنیم که از بین 3 نفر 2تاشون خالداشته باشن. آدمای خالدار فقط 1 خالدار تو جمع میبینن و کسی که خال نداره 2تا خالدار میبینه. اون خالدارها هر کدوم منتظرن تا اون یکی بعد از گذشت 1 روز خودشو بکشه و چون هر دو منتظرن اون یکی خودشو بکشه پس هیچ کدوم بعد از روز اول خودشون رو نکشتن و در روز بعد با دیدن همدیگه متوجه میشن که طرف مقابل هم همون خیالی رو کرده که خودش کرده. به این ترتیب از خالدار بودن خودش مطمئن میشه و در روز دوم هر دو خالدار خودشون رو در یک روز میکشن. در روز سوم، نفر سوم با دیدن این که اون 2 خالداری که در روزهای قبل میدیدشون، دیگه نیومدن از خالدار نبودن خودش مطمئن میشه. چرا که اگه هر 3تا خالدار بودن هر کدوم از این 3 نفر 2 روز منتظر میشد که دو خالداری که میبینه خودشون رو بکشن و در روز سوم وقتی اون دو تای دیگه رو بازم میبینه متوجه میشه که اون 2 تا هم هر کدوم 2 خالدار دیدن که همون خیالی رو کردن که خودش داشته. پس از خالدار بودن خودش مطمئن میشه و در روز سوم هر 3 تا با هم خودکشی میکنن.
و ...
در حالت 100 نفر که 49 تاشون خالدار باشن هم هر کدوم از این 49 نفر 48 خالدار میبینه و 48 روز منتظر میشه تا بعد از گذشت این 48 روز خالدارهایی رو که میدیده خودکشی کنن. و چون در روز 49 بازم اونها رو میبینه مطمئن میشه که هر کدوم از این 48 نفر هم 48 خالدار دیدن و همون خیالی رو کردن که خودش داشته و چون خالدار دیگه ای وجود نداره پس خودش خالداره و در روز 49 همگی خودکشی میکنن. در روز 50ام بقیه با دیدن اینکه 49 نفر خالدار دیگه نیومدن از خالدار نبودن خودشون مطمئن میشن.
در حالت کلی از بین N نفر که K تاشون خالدارن هر خالدار، K منهای 1 روز صبر میکنه و وقتی که میبینه بازم بقیه اومدن به خالدار بودن خودش مطمئن میشه و در روز K ام همگی با هم خود کشی میکنن و بقیه از اینکه خال ندارن مطمئن میشن.

موفق باشین.
88/4/14

ask_bl
05-07-2009, 09:52
حالا که اینقدر پایه این یه سوال سخت تر براتون میذارم.
این سوال رو هنگامی که مشغول بازی بسیار مفرح مافیا بودیم یکی مطرح کرد:
فرض کنید 100 نفر رو در اختیار دارید که 49 نفر از اونها روی پیشونیشون یه خال مشخص دارن و 51 تای بقیه مث آدمای عادی اند. اون 49 نفر خودشون از وجود خال روی پیشونیشون بی اطلاعند و فرض میکنیم که چیزی به نام آینه هم هنوز اختراع نشده! این صد نفر روزی یکبار دور هم جمع میشن و فقط و فقط همدیگر رو نگاه میکنن و به هیچ وجه نه با هم حرف میزنن و نه با اشاره به همدیگه میفهمونن که تو خال داری یا نه. هر نفر فقط میتونه ببینه که چند نفر خال دارن ولی نمیتونه به اون افراد مستقیما یا با اشاره چیزی بفهمونه. حال فرض میکنیم داشتن خال چیز خیلی بدی باشه به حدی که اگه هر کس به هر نحوی ( باتوجه به ممنوعیتهای ذکر شده ) بفهمه که خال داره خودشو میکشه و دیگه از فردای اون روز تو جلسات روزانه شرکت نمیکنه. بقیه افراد هم متوجه این موضوع هستند که اگر فردی در جلسه حاضر نشد مطمئنا به خالدار بودن خود پی برده و خودکشی کرده. این صد نفر نمیدونن که دقیقا چند آدم خالدار وجود داره. تنها چیزی که میدونن اینه که حداقل یه خالدار وجود داره.
حال سوال اینه که بعد از گذشت چند روز همه ی افراد خالدار متوجه میشن و خودکشی میکنن و بقیه هم از این که خال ندارن مطمئن میشن؟

بازم تکرار میکنم که جوابشو فقط به شرطی میزارم که روش فکر و بحث بشه.

موفق باشین.
88/4/9

تو سوال گفتین که توجمع صدنفری "تنها چیزی که میدونن اینه که حداقل یه خالدار وجود داره" خوب از کجا معلوم که تو جمع سه نفری یکیشون خالدار باشه یا نه.

davy jones
05-07-2009, 10:04
تو سوال گفتین که توجمع صدنفری "تنها چیزی که میدونن اینه که حداقل یه خالدار وجود داره" خوب از کجا معلوم که تو جمع سه نفری یکیشون خالدار باشه یا نه.
منظورم این بود که صرف نظر از اینکه جمع آدما چند نفره اس، همواره یک خالدار وجود داره و این موضوع رو همه میدونن. اگه بد رسوندم عذر میخوام ولی این موضوع پایه ی استقرا در حل مسئله است و بدون اون نمیشه مسئله رو حل کرد.

موفق باشین.
88/4/14

davy jones
05-07-2009, 10:43
منم برات چندتا انتگرال طرح کردم ببین میتونی حلشون کنی منتظر جواب هات هستم :::

[ برای مشاهده لینک ، لطفا با نام کاربری خود وارد شوید یا ثبت نام کنید ]

برای مشاهده محتوا ، لطفا وارد شوید یا ثبت نام کنید

مطمئنی این توابعی که نوشتی دارای تابع اولیه هستن؟
من فکر نمیکنم که این انتگرالها جواب داشته باشه.
اگه جواب داره لطف کن و یکم راهنمایی کن و زود جوابشو نذار.
ممنون.

موفق باشین.
88/4/14

behnam_tr
05-07-2009, 11:05
مطمئنی این توابعی که نوشتی دارای تابع اولیه هستن؟
من فکر نمیکنم که این انتگرالها جواب داشته باشه.
اگه جواب داره لطف کن و یکم راهنمایی کن و زود جوابشو نذار.
ممنون.

موفق باشین.
88/4/14

بله که دارند یعنی میگی ندارند ؟؟

سوال :: آیا تمام انتگرال ها تابع اولیه دارند ؟؟

davy jones
05-07-2009, 11:24
بله که دارند یعنی میگی ندارند ؟؟

سوال :: آیا تمام انتگرال ها تابع اولیه دارند ؟؟
معلومه که ندارن. کی گفته همه ی توابع دارای تابع اولیه ی انتگرالن؟
بطور مثال:
[ برای مشاهده لینک ، لطفا با نام کاربری خود وارد شوید یا ثبت نام کنید ]
تابع بالا جواب نامعین نداره و فقط به صورت معین اونهم با روشهای محاسبات عددی قابل حله.

موفق باشین.
88/4/14

chessmathter
05-07-2009, 11:26
بله که دارند یعنی میگی ندارند ؟؟

سوال :: آیا تمام انتگرال ها تابع اولیه دارند ؟؟
پیوسته بودن یه تابع یعنی اون تابع انتگرال داره انتگرال یه تایع هم یعنی اون تابع تابع اولیه داره ولی ممکنه توابع اولیه یه تابع بدست نیاد یعنی میشه اندازه موهای سرت تابع در آورد که توابع اولیه اشو نشه بدست آورد
مثلا این تابع فقط با یکی از این 3 شرط میتونی بدست بیاری تابع اولیشو امتحان کن:31:

[ برای مشاهده لینک ، لطفا با نام کاربری خود وارد شوید یا ثبت نام کنید ]

chessmathter
05-07-2009, 11:36
معلومه که ندارن. کی گفته همه ی توابع دارای تابع اولیه ی انتگرالن؟
بطور مثال:
[ برای مشاهده لینک ، لطفا با نام کاربری خود وارد شوید یا ثبت نام کنید ]
تابع بالا جواب نامعین نداره و فقط به صورت معین اونهم با روشهای محاسبات عددی قابل حله.

موفق باشین.
88/4/14
همونطور که davy jones گفت
یه راش استفاده از بسط تیلور
یه راه دیگشم انتگرال 2 گانه
:20::5::46:
محاسبه معین

davy jones
05-07-2009, 11:38
پیوسته بودن یه تابع یعنی اون تابع انتگرال داره انتگرال یه تایع هم یعنی اون تابع تابع اولیه داره ولی ممکنه توابع اولیه یه تابع بدست نیاد یعنی میشه اندازه موهای سرت تابع در آورد که توابع اولیه اشو نشه بدست آورد
مثلا این تابع فقط با یکی از این 3 شرط میتونی بدست بیاری تابع اولیشو امتحان کن:31:

[ برای مشاهده لینک ، لطفا با نام کاربری خود وارد شوید یا ثبت نام کنید ]



ممنونم دوست عزیز.
معلومه که هر تابع قطعه به قطعه پیوسته حتما انتگرال داره. ولی همون جور که گفتی بعضی از توابع رو نمیشه براشون تابع اولیه ی انتگرال پیدا کرد.
در مورد شروطی هم که نوشتی باید بگم که شرط 3 که از روی 1و2 به وضوح بدست میآد و عملا تکراریه ولی من با شرط دوم خیلی موافق نیستم. به نظرم همون شرط اول برای وجود تابع اولیه کافیه.

موفق باشین.
88/4/14

davy jones
05-07-2009, 12:07
حالا یه سوال نسبتا ملایم (:27:) طرح میکنم:
جواب این حد چی میشه؟ :
[ برای مشاهده لینک ، لطفا با نام کاربری خود وارد شوید یا ثبت نام کنید ]

جوابش رو به شرط بحث و بررسی میذارم.

موفق باشین.
88/4/14

behnam_tr
05-07-2009, 12:26
حالا یه سوال نسبتا ملایم (:27:) طرح میکنم:
جواب این حد چی میشه؟ :
[ برای مشاهده لینک ، لطفا با نام کاربری خود وارد شوید یا ثبت نام کنید ]

جوابش رو به شرط بحث و بررسی میذارم.

موفق باشین.
88/4/14

:21::21:جوابش 1 هست ؟؟؟

chessmathter
05-07-2009, 12:42
ممنونم دوست عزیز.
معلومه که هر تابع قطعه به قطعه پیوسته حتما انتگرال داره. ولی همون جور که گفتی بعضی از توابع رو نمیشه براشون تابع اولیه ی انتگرال پیدا کرد.
در مورد شروطی هم که نوشتی باید بگم که شرط 3 که از روی 1و2 به وضوح بدست میآد و عملا تکراریه ولی من با شرط دوم خیلی موافق نیستم. به نظرم همون شرط اول برای وجود تابع اولیه کافیه.

موفق باشین.
88/4/14
عزیز انگار نگرفتی منظورم این بود اگه فقط یکی از این شرط هار و داشته باشه میشه بدست آورد

davy jones
05-07-2009, 12:51
عزیز انگار نگرفتی منظورم این بود اگه فقط یکی از این شرط هار و داشته باشه میشه بدست آورد

اوووووکییییی متوجه شدم.
ممنون.

موفق باشین.
88/4/14

davy jones
05-07-2009, 12:52
:21::21:جوابش 1 هست ؟؟؟

نه.:31:

موفق باشین.
88/4/14

chessmathter
05-07-2009, 12:58
حالا یه سوال نسبتا ملایم (:27:) طرح میکنم:
جواب این حد چی میشه؟ :
[ برای مشاهده لینک ، لطفا با نام کاربری خود وارد شوید یا ثبت نام کنید ]

جوابش رو به شرط بحث و بررسی میذارم.

موفق باشین.
88/4/14
[ برای مشاهده لینک ، لطفا با نام کاربری خود وارد شوید یا ثبت نام کنید ]

davy jones
05-07-2009, 13:03
[ برای مشاهده لینک ، لطفا با نام کاربری خود وارد شوید یا ثبت نام کنید ]

دوست عزیز میشه بگی چی کار کردی؟:18:
اولا منهای یک رو از کجا همین جوری به عبارت داخل پرانتز پایه اضافه کردی؟
شاید منظورت این بوده که کسینوس به توان دو هستش 1 منهای سینوس به توان 2 ولی شما همینطوری یه منهای یک به پرانتز اضافه کردی!:13:
ثانیا گیرم که اصلا اونهم هیچی جواب عبارتی که خودت ساختی بازم 1 تقسیم بر e نمیشه.
ثالثا خواهش میکنم یه کم مرتب تر بنویس. در زیر لیمیت ها "e به توان" چرا آوردی؟
ولی در کل به جواب نزدیک شدی.:46:

موفق باشین.
88/4/14

behnam_tr
05-07-2009, 13:04
[ برای مشاهده لینک ، لطفا با نام کاربری خود وارد شوید یا ثبت نام کنید ]

چی شد ؟؟ من دارم گیج میزنم

davy jones
05-07-2009, 13:09
چی شد ؟؟ من دارم گیج میزنم

حق داری!:27:
آخه خیلی قاطی پاتی نوشته! ( البته قصد توهین نداشتم.:31:)

حالا تو هم سعی کن یه راه حل از خودت بدی. مهم نیست که بتونی حلش کنی. مهم اینه که این مسئله باعث بشه به فکر فرو بری!:5:

موفق باشین.
88/4/14

behnam_tr
05-07-2009, 14:10
حق داری!:27:
آخه خیلی قاطی پاتی نوشته! ( البته قصد توهین نداشتم.:31:)

حالا تو هم سعی کن یه راه حل از خودت بدی. مهم نیست که بتونی حلش کنی. مهم اینه که این مسئله باعث بشه به فکر فرو بری!:5:

موفق باشین.
88/4/14

به جواب نرسیدم ولی مطمئنم (( 90 درصد )) جواب 1 هست //////

[ برای مشاهده لینک ، لطفا با نام کاربری خود وارد شوید یا ثبت نام کنید ]

davy jones
05-07-2009, 14:19
به جواب نرسیدم ولی مطمئنم (( 90 درصد )) جواب 1 هست //////

[ برای مشاهده لینک ، لطفا با نام کاربری خود وارد شوید یا ثبت نام کنید ]

تو محاسباتت دچار اشتباه نشدی ولی به نظرم لقمه رو دور سر خودت پیچوندی!
اون عبارتی که من نوشته بودم که خیلی از این ساده تر بود.
لگاریتم رو از کجا آوردی!:18:
در آخر هم باید بگم که این حد دارای ابهام از نوع +1 به توان بینهایت هستش که با همه ی این اوصافی که نوشتی ولی هنوز ابهام رفع نشده. جوابش هم 1 نمیشه.

موفق باشین.
88/4/14

chessmathter
05-07-2009, 14:20
دوست عزیز میشه بگی چی کار کردی؟:18:
اولا منهای یک رو از کجا همین جوری به عبارت داخل پرانتز پایه اضافه کردی؟
شاید منظورت این بوده که کسینوس به توان دو هستش 1 منهای سینوس به توان 2 ولی شما همینطوری یه منهای یک به پرانتز اضافه کردی!:13:
ثانیا گیرم که اصلا اونهم هیچی جواب عبارتی که خودت ساختی بازم 1 تقسیم بر e نمیشه.
ثالثا خواهش میکنم یه کم مرتب تر بنویس. در زیر لیمیت ها "e به توان" چرا آوردی؟
ولی در کل به جواب نزدیک شدی.:46:

موفق باشین.
88/4/14
خوب جواب حد میشه 1 به توان بینهایت که مبهمه و برای بر طرف کردنش :

[ برای مشاهده لینک ، لطفا با نام کاربری خود وارد شوید یا ثبت نام کنید ]

اثباتشم بخواین هست:31::20:
که میشه e بتوان منفی 1 که همون 1 تقسیم بر e میشه
من به جواب نزدیک نشدم جواب دقیقا همینه :46::20:

davy jones
05-07-2009, 14:24
خوب جواب حد میشه 1 به توان بینهایت که مبهمه و برای بر طرف کردنش :

[ برای مشاهده لینک ، لطفا با نام کاربری خود وارد شوید یا ثبت نام کنید ]

اثباتشم بخواین هست:31::20:
که میشه e بتوان منفی 1 که همون 1 تقسیم بر e میشه
من به جواب نزدیک نشدم جواب دقیقا همینه :46::20:




تازه الان متوجه شدم چی کار کردی.
جواب درسته. ممنونم.
ولی خودمونیم خیلی بد نوشته بودی. هر کسی رو میتونس به اشتباه بندازه.
خودمم یه کم گیج شده بودم. برای همین فکر میکردم که جواب e به توان 1+ هستش نه 1-.
بازم ممنون.

موفق باشین.
88/4/14

davy jones
05-07-2009, 14:29
یه سوال هوش:
فرض کنید یه ساعت عقربه ای دارید و الان ساعت 1 و 5 دقیقه ی بعد از ظهره. تا ساعت 5 و 25 دقیقه ی صبح فردا چند بار عقربه های ساعت شمار و دقیقه شمار از روی هم عبور میکنن؟

موفق باشین.
88/4/14

behnam_tr
05-07-2009, 14:33
تو محاسباتت دچار اشتباه نشدی ولی به نظرم لقمه رو دور سر خودت پیچوندی!
اون عبارتی که من نوشته بودم که خیلی از این ساده تر بود.
لگاریتم رو از کجا آوردی!:18:
در آخر هم باید بگم که این حد دارای ابهام از نوع +1 به توان بینهایت هستش که با همه ی این اوصافی که نوشتی ولی هنوز ابهام رفع نشده. جوابش هم 1 نمیشه.

موفق باشین.
88/4/14

:21:این روش حد گیری لگاریتمی هست نشنیدی مگه ؟؟:27:

chessmathter
05-07-2009, 14:39
یه سوال هوش:
فرض کنید یه ساعت عقربه ای دارید و الان ساعت 1 و 5 دقیقه ی بعد از ظهره. تا ساعت 5 و 25 دقیقه ی صبح فردا چند بار عقربه های ساعت شمار و دقیقه شمار از روی هم عبور میکنن؟

موفق باشین.
88/4/14
16 بار
آقا باس حداقل 10 حرف نوشت :31::46:

davy jones
05-07-2009, 15:06
16 بار
آقا باس حداقل 10 حرف نوشت :31::46:
درسته.
ممنونم.

موفق باشین.
88/4/14

davy jones
05-07-2009, 15:07
:21:این روش حد گیری لگاریتمی هست نشنیدی مگه ؟؟:27:

بابا به چه زبونی بگم غلط کردم؟:19::19::19::19::19::19:

موفق باشین.
88/4/14

davy jones
05-07-2009, 15:13
یه سوال دیگه که واقعا ساده اس:
پلکانی شامل 30 پله است. قورباغه ای در پله اول ایستاده است و میخواهد به پله ی 30ام برود.
اگر قورباغه با هر بار پرش هر تعداد پله که بخواهد بتواند بالا بپرد در آن صورت به چند حالت میتواند به پله ی 30ام برسد؟

موفق باشین.
88/4/14

davy jones
05-07-2009, 15:16
بازم یه سوال دیگه تو همون مایه ها:
یه جدول m در n داریم. تعداد کل مستطیلهای موجود در این جدول چند تاس؟
راهنمایی: مربع ها هم نوعی مستطیل محسوب میشوند.

فعلا تا فردا نمیتونم بیام.
خداحافظ.

موفق باشین.
88/4/14

behnam_tr
05-07-2009, 15:18
یه سوال دیگه که واقعا ساده اس:
پلکانی شامل 30 پله است. قورباغه ای در پله اول ایستاده است و میخواهد به پله ی 30ام برود.
اگر قورباغه با هر بار پرش هر تعداد پله که بخواهد بتواند بالا بپرد در آن صورت به چند حالت میتواند به پله ی 30ام برسد؟

موفق باشین.
88/4/14

در هر حالت میزان پرش یکسان خواد بود ؟؟ یعنی مثلا اگه دوتا ی اول رو با یک پرش بره دوتای بعدی رو هم باید با دو پرش بره یا نه مثلا میتونه اول دوتا بعد مثلا 4 و ....

davy jones
05-07-2009, 15:21
در هر حالت میزان پرش یکسان خواد بود ؟؟ یعنی مثلا اگه دوتا ی اول رو با یک پرش بره دوتای بعدی رو هم باید با دو پرش بره یا نه مثلا میتونه اول دوتا بعد مثلا 4 و ....

منظورم این بود که مثلا یکضرب هم میتونه بپره روی پله ی 30 ام. میتونه هم یکی یکی بپره. اینکه در پرش kام چند پله بپره ربطی به قبلیا نداره ولی نکته ای که مهمه اینه که فقط رو به بالا میپره و دیگه راه رفته رو بر نمیگرده.

موفق باشین.
88/4/14

behnam_tr
05-07-2009, 15:23
این انتگرال موجوده ؟؟

[ برای مشاهده لینک ، لطفا با نام کاربری خود وارد شوید یا ثبت نام کنید ]

zahedy2006
05-07-2009, 18:20
این انتگرال موجوده ؟؟

اون شرايط صفر شدن و غيره اش را بيخيال شويم ميشه
cos(ln(cos))-cos

behnam_tr
05-07-2009, 20:02
اون شرايط صفر شدن و غيره اش را بيخيال شويم ميشه
cos(ln(cos))-cos

nدوست عزیز بهتر بود راه حلتون رو ارئه میدادید به هر حال من این جوری اینو حل میکنم از روش جز به جز


[ برای مشاهده لینک ، لطفا با نام کاربری خود وارد شوید یا ثبت نام کنید ]


[ برای مشاهده لینک ، لطفا با نام کاربری خود وارد شوید یا ثبت نام کنید ]

behnam_tr
05-07-2009, 20:08
حالا این یکی رو حل کنید .....

البته اولی رو بدون استفاده از اضافه و کم کردن 1 و دومی رو بدون استفاده از رابطه موجود برای سینوس به توان 2 و کوسینوس به توان 2 ((( منظورم همون cos2x+1/2 , هست ))

[ برای مشاهده لینک ، لطفا با نام کاربری خود وارد شوید یا ثبت نام کنید ]

reza-atom
08-07-2009, 20:29
سلام
ميخواستم بدونم از چه طريقي ميتونيم برد توابع fog و gof را پيدا كنيم؟

davy jones
11-07-2009, 07:34
سلام
ميخواستم بدونم از چه طريقي ميتونيم برد توابع fog و gof را پيدا كنيم؟

در مورد برد توابع ترکیبی ( مثل fog ) باید ابتدا برد تابع g رو به عنوان دامنه برای تابع f در نظر بگیری و محدودیتهای تابع f رو هم لحاظ کنی تا به برد تابع کلی برسی.

موفق باشین.
88/4/20

davy jones
11-07-2009, 07:37
حالا این یکی رو حل کنید .....

البته اولی رو بدون استفاده از اضافه و کم کردن 1 و دومی رو بدون استفاده از رابطه موجود برای سینوس به توان 2 و کوسینوس به توان 2 ((( منظورم همون cos2x+1/2 , هست ))

[ برای مشاهده لینک ، لطفا با نام کاربری خود وارد شوید یا ثبت نام کنید ]

دوست عزیز! آدم مجبور نیست تو زندگی لقمه رو درو سر خودش بچرخونه!:41:
اگه نخوایم از روشهایی که گفتی استفاده کنیم باید 4 یا 5 مرحله انتگرال جزء به جزء بگیریم که ممکنه گیج بشیم یا باعث بشه تو یکی از مراحل اشتباه کنیم ولی متوجه نشیم.

موفق باشین.
88/4/20

davy jones
11-07-2009, 08:09
یه سوال دیگه که واقعا ساده اس:
پلکانی شامل 30 پله است. قورباغه ای در پله اول ایستاده است و میخواهد به پله ی 30ام برود.
اگر قورباغه با هر بار پرش هر تعداد پله که بخواهد بتواند بالا بپرد در آن صورت به چند حالت میتواند به پله ی 30ام برسد؟

موفق باشین.
88/4/14


بازم یه سوال دیگه تو همون مایه ها:
یه جدول m در n داریم. تعداد کل مستطیلهای موجود در این جدول چند تاس؟
راهنمایی: مربع ها هم نوعی مستطیل محسوب میشوند.

فعلا تا فردا نمیتونم بیام.
خداحافظ.

موفق باشین.
88/4/14

ظاهرا کسی رو اینا هنوز فکر نکرده!
اگه بازم خبری نشد، تا فردا خودم جوابشو میذارم.

موفق باشین.
88/4/20

davy jones
11-07-2009, 11:21
یه سوال دیگه:
فرض کنید 9 تا نقطه به صورت زیر چیده شده باشن:
[ برای مشاهده لینک ، لطفا با نام کاربری خود وارد شوید یا ثبت نام کنید ]

فرض کنیم برای اینکه این نقاط رو به هم وصل کنیم فقط مجاز به استفاده از خطوط راست هستیم یعنی منحنی نداریم. اگر در خط راست ما شکستگی هم وجود داشته باشد به این معنی که راستای خط عوض شود آنوقت آن را 2 تا خط میشماریم.
حال حداقل با چند خط راست و بدون اینکه خودکار (یا مداد) را از روی کاغذ بلند کنیم میتونیم همه ی نقاط رو به هم وصل کنیم؟
هر کس هر عددی رو میگه لطفا با شکل بذاره تا همه متوجه منظورش بشن و اینکه چطوری به اون راه حل رسیده.

موفق باشین.
88/4/20

davy jones
11-07-2009, 11:38
nدوست عزیز بهتر بود راه حلتون رو ارئه میدادید به هر حال من این جوری اینو حل میکنم از روش جز به جز


[ برای مشاهده لینک ، لطفا با نام کاربری خود وارد شوید یا ثبت نام کنید ]


[ برای مشاهده لینک ، لطفا با نام کاربری خود وارد شوید یا ثبت نام کنید ]




دوست عزیز این راه حلی که نوشتی غلطه!!
تو دیفرانسیلگیری از تابع ln برای استفاده در جزء به جزء اشتباه کردی

اون شرايط صفر شدن و غيره اش را بيخيال شويم ميشه
cos(ln(cos))-cos

راه حل کاملش اینه ( البته با تشکر از zahedy2006 ):
[ برای مشاهده لینک ، لطفا با نام کاربری خود وارد شوید یا ثبت نام کنید ]

موفق باشین.
88/4/20

behnam_tr
11-07-2009, 12:03
دوست عزیز این راه حلی که نوشتی غلطه!!
تو دیفرانسیلگیری از تابع ln برای استفاده در جزء به جزء اشتباه کردی


راه حل کاملش اینه ( البته با تشکر از zahedy2006 ):
[ برای مشاهده لینک ، لطفا با نام کاربری خود وارد شوید یا ثبت نام کنید ]

موفق باشین.
88/4/20

جواب ها رو اگه نیگا کنی جواب ها یکی هستن ؟؟ نیستن ؟؟ (( بیشتر دقت کن عزیز دل ))

davy jones
11-07-2009, 12:17
جواب ها رو اگه نیگا کنی جواب ها یکی هستن ؟؟ نیستن ؟؟ (( بیشتر دقت کن عزیز دل ))

حالا که بیشتر دقت میکنم میبینم که هستن:31:
ولی خداییش راه حل من ساده تر و بهتره!:46:

موفق باشین.
88/4/20

amintnt
11-07-2009, 13:48
پلکانی شامل 30 پله است. قورباغه ای در پله اول ایستاده است و میخواهد به پله ی 30ام برود.
اگر قورباغه با هر بار پرش هر تعداد پله که بخواهد بتواند بالا بپرد در آن صورت به چند حالت میتواند به پله ی 30ام برسد؟
465
سیگما n




یه جدول m در n داریم. تعداد کل مستطیلهای موجود در این جدول چند تاس؟
راهنمایی: مربع ها هم نوعی مستطیل محسوب میشوند.
"ترکیب دو از M" ضرب در "ترکیب دو از N"

davy jones
11-07-2009, 14:45
465
سیگما n




"ترکیب دو از M" ضرب در "ترکیب دو از N"

امین جان ممنونم که به سوالها پاسخ دادی. ولی این که چی شد که به این جوابها رسیدی رو هم بذار چون جوابهای من با اینایی که گذاشتی فرق میکنه خصوصا جواب سوال اولی.

موفق باشین.
88/4/20

amintnt
11-07-2009, 17:13
امین جان ممنونم که به سوالها پاسخ دادی. ولی این که چی شد که به این جوابها رسیدی رو هم بذار چون جوابهای من با اینایی که گذاشتی فرق میکنه خصوصا جواب سوال اولی.

موفق باشین.
88/4/20
خواهش میکنم.
در مورد سوال اول باید بگم که روش فکر نکردم. در فیزیک پیش دانشگاهی، سوالی هست که تعداد حالت های گذار یک الکترون در تراز n به تراز یک رو میخواد، و توی کتاب آبی کانون فرمول سیگما n-1 رو داده بود. سوالی که شما مطرح کردین شبیه به همین بود، بنابراین من اون جواب رو ارائه دادم و کتاب در مورد اینکه چطور به این رابطه رسیده توضیحی نداده. اما جوابی که الآن داشتم بهش فکر میکردم این بود:
برای طی کردن پله ها، قوباغه یا باید بدون استفاده از پله ها مستقیما به پله ی 30 جهش کنه، یا باید از یک پله استفاده کنه، یا باید از دو پله استفاده کنه، یا سه پله یا... و به همین ترتیب. قورباغه روی پله ی اول قرار داره و مقصد هم پله ی 30 ام هست، پس از این دو پله صرف نظر می کنیم و به 28 پله ی میانی توجه می کنیم. حالا میگیم اگه از یک پله استفاده کنه، به تعداد "ترکیب 1 از 28" راه داره، اگه از دو پله استفاده کنه، به تعداد ترکیب 2 از 28 راه داره و ... . مجموع همه ی اینها میشه جواب، البته عدد یک رو هم باید اضافه کرد، چون صفر از 28 هم هست که در نظر نگرفتیم.

و این مجموع، همونطور که می دونید، تعداد زیر مجموعه های یک مجموعه ی n عضوی هست که برابر هست با دو به توان n. بنابراین جواب باید بشه دو به توان 28. امیدوارم درست باشه!

و اما سوال دوم:
هر مستطیل از 4 خط تشکیل شده، دو عمودی و دو افقی. باید ببینیم به چند طریق میشه دو خط از n+1 خط رو انتخاب کرد برای عرض مستطیل و به چند طریق دو خط از m+1 خط رو برای طول مستطیل. دلیل اون 1 هم اینه که تعداد خطوط عمودی یا افقی یکی از تعداد خانه های ردیف ها و ستون ها بیشتره.

SI-SA
11-07-2009, 17:53
یه سوال دیگه:
فرض کنید 9 تا نقطه به صورت زیر چیده شده باشن:
[ برای مشاهده لینک ، لطفا با نام کاربری خود وارد شوید یا ثبت نام کنید ]

فرض کنیم برای اینکه این نقاط رو به هم وصل کنیم فقط مجاز به استفاده از خطوط راست هستیم یعنی منحنی نداریم. اگر در خط راست ما شکستگی هم وجود داشته باشد به این معنی که راستای خط عوض شود آنوقت آن را 2 تا خط میشماریم.
حال حداقل با چند خط راست و بدون اینکه خودکار (یا مداد) را از روی کاغذ بلند کنیم میتونیم همه ی نقاط رو به هم وصل کنیم؟
هر کس هر عددی رو میگه لطفا با شکل بذاره تا همه متوجه منظورش بشن و اینکه چطوری به اون راه حل رسیده.
5 تا...
اینجوری:
[ برای مشاهده لینک ، لطفا با نام کاربری خود وارد شوید یا ثبت نام کنید ]

davy jones
12-07-2009, 07:44
خواهش میکنم.
در مورد سوال اول باید بگم که روش فکر نکردم. در فیزیک پیش دانشگاهی، سوالی هست که تعداد حالت های گذار یک الکترون در تراز n به تراز یک رو میخواد، و توی کتاب آبی کانون فرمول سیگما n-1 رو داده بود. سوالی که شما مطرح کردین شبیه به همین بود، بنابراین من اون جواب رو ارائه دادم و کتاب در مورد اینکه چطور به این رابطه رسیده توضیحی نداده. اما جوابی که الآن داشتم بهش فکر میکردم این بود:
برای طی کردن پله ها، قوباغه یا باید بدون استفاده از پله ها مستقیما به پله ی 30 جهش کنه، یا باید از یک پله استفاده کنه، یا باید از دو پله استفاده کنه، یا سه پله یا... و به همین ترتیب. قورباغه روی پله ی اول قرار داره و مقصد هم پله ی 30 ام هست، پس از این دو پله صرف نظر می کنیم و به 28 پله ی میانی توجه می کنیم. حالا میگیم اگه از یک پله استفاده کنه، به تعداد "ترکیب 1 از 28" راه داره، اگه از دو پله استفاده کنه، به تعداد ترکیب 2 از 28 راه داره و ... . مجموع همه ی اینها میشه جواب، البته عدد یک رو هم باید اضافه کرد، چون صفر از 28 هم هست که در نظر نگرفتیم.

و این مجموع، همونطور که می دونید، تعداد زیر مجموعه های یک مجموعه ی n عضوی هست که برابر هست با دو به توان n. بنابراین جواب باید بشه دو به توان 28. امیدوارم درست باشه!

و اما سوال دوم:
هر مستطیل از 4 خط تشکیل شده، دو عمودی و دو افقی. باید ببینیم به چند طریق میشه دو خط از n+1 خط رو انتخاب کرد برای عرض مستطیل و به چند طریق دو خط از m+1 خط رو برای طول مستطیل. دلیل اون 1 هم اینه که تعداد خطوط عمودی یا افقی یکی از تعداد خانه های ردیف ها و ستون ها بیشتره.

متشکرم. جوابها درسته!

موفق باشین.
88/4/21

davy jones
12-07-2009, 07:46
5 تا...
اینجوری:
[ برای مشاهده لینک ، لطفا با نام کاربری خود وارد شوید یا ثبت نام کنید ]

ممنونم. ولی فکر کنم با 4 تا خط هم میشه. یه کم روش فکر کن. اگه کس دیگه ای هم نظری داره لطفا بگه.

موفق باشین.
88/4/21

davy jones
12-07-2009, 07:58
یه سوال نسبتا ناراحت! :
فرض کنید که شما به یک مغازه نانوایی رفته اید و میخواهید از مغازه دار، آرد بخرید.
صاحب مغازه برای کشیدن وزن آردی که به مشتریان میفروشد فقط از یک ترازوی دو کفه ای معمولی استفاده میکند.
صاحب مغازه ایده ی جالبی به کار گرفته است به طوری که با کمترین تعداد ممکن از سنگ ترازو ، قادر است برای شما از بین اعداد صحیح بین 1 تا 40 کیلوگرم، هر مقدار که شما بخواهید را با یک بار استفاده از ترازو و تعدادی از آن سنگها برای شما بسته بندی و آماده کند.
حال سوال این است که صاحب مغازه ی نانوایی از چند سنگ ترازو استفاده کرده است و آن سنگها هر کدام چند کیلویی بوده است؟

موفق باشین.
88/4/21

SI-SA
12-07-2009, 10:49
یه سوال نسبتا ناراحت! :
فرض کنید که شما به یک مغازه نانوایی رفته اید و میخواهید از مغازه دار، آرد بخرید.
صاحب مغازه برای کشیدن وزن آردی که به مشتریان میفروشد فقط از یک ترازوی دو کفه ای معمولی استفاده میکند.
صاحب مغازه ایده ی جالبی به کار گرفته است به طوری که با کمترین تعداد ممکن از سنگ ترازو ، قادر است برای شما از بین اعداد صحیح بین 1 تا 40 کیلوگرم، هر مقدار که شما بخواهید را با یک بار استفاده از ترازو و تعدادی از آن سنگها برای شما بسته بندی و آماده کند.
حال سوال این است که صاحب مغازه ی نانوایی از چند سنگ ترازو استفاده کرده است و آن سنگها هر کدام چند کیلویی بوده است؟
باید اعدادی باشه که بتونه همهی 1 تا 40 رو پوشش بده...پس:
1.20.10.2.5.3.1.

davy jones
12-07-2009, 11:04
باید اعدادی باشه که بتونه همهی 1 تا 40 رو پوشش بده...پس:
1.20.10.2.5.3.1.

ممنونم دوست خوبم!
چند تا سوال از شما داشتم.
1- چرا سنگ 1 کیلویی رو 2بار به کار بردین؟
2- با این وزنه هایی که گفتین مطمئنا میشه از 1 تا 40 رو ساخت ولی آیا این حداقل تعداد سنگهای مورد نیازه؟ یعنی با کمتر از این تعداد نمیشه چنین کاری کرد؟
3- چه لزومی داره که بیش از 40 کیلو رو هم بتونیم یکضرب بکشیم. این وزنه هایی که شما گفتین تا 44 کیلو رو هم جواب میده. مطمئنا اگه میخوایم حداقل تعداد وزنه ها رو بدست بیاریم باید حالات بهینه رو مورد استفاده قرار بدیم.(این خودش یه راهنمایی برای حل این مساله بود.)

موفق باشین.
88/4/21

yugioh
12-07-2009, 19:11
یه سوال نسبتا ناراحت! :
فرض کنید که شما به یک مغازه نانوایی رفته اید و میخواهید از مغازه دار، آرد بخرید.
صاحب مغازه برای کشیدن وزن آردی که به مشتریان میفروشد فقط از یک ترازوی دو کفه ای معمولی استفاده میکند.
صاحب مغازه ایده ی جالبی به کار گرفته است به طوری که با کمترین تعداد ممکن از سنگ ترازو ، قادر است برای شما از بین اعداد صحیح بین 1 تا 40 کیلوگرم، هر مقدار که شما بخواهید را با یک بار استفاده از ترازو و تعدادی از آن سنگها برای شما بسته بندی و آماده کند.
حال سوال این است که صاحب مغازه ی نانوایی از چند سنگ ترازو استفاده کرده است و آن سنگها هر کدام چند کیلویی بوده است؟

موفق باشین.
88/4/21
معمولا نمیام اینجا ولی این جالب بود:
جواب 4 وزنه هست.
چند نوع ایده غلط هست که سریع به ذهن میرسه اگر این ایده ها به ذهنتون رسید مشکلی نیست یه خورده عجله کردید ولی ذهنتون روشهای خوبی رو امتحان می کنه :
1و2و4و8و16وو32 که میشه 6.
یا از یک روش پله ای استفاده می کنیم: برای تا 40 کیلو یک وزنه 20 کیلو + وزنه هایی فاقد 20 که تا 20 رو بگیره: برای 20: 10 + وزنه های فاقد 10 که تا 10 رو بگیره، برای10 : 5 + فاقد 5 برای تا 5.
حالا با 1و2و3و4 ، 5رو با چندتاش میشه ساخت؟ 3تا: مثلا با 1و2و3 (که میشه ثابت کرد با2تا وزنه نمیشه.) یا بعه عبارتی 3+ تا 2 فاقد 3. که میشه 1و 2
میشه 6تا.

این روش ها هیچ کدوم یک الگوریتم درست نیست چون هر دو در مثال 4 مشکل دارند و جواب 3 عدد 1و2و1 میدهند که جواب درست: 1و3 یعنی دو وزنه هست.
جالا چرا این روشها غلطه؟ چون به دو سمت بودن ترازو دقت نداره.
فرض کنیم وزنه های :
x1,x2,x3,...xn استفاده بشند، سمت راست ترازو رو مثبت وسمت چپ رو منفی میدونیم یعنی آرد سمت چپ هست: با این اعداد باید:
40-,39-,...,2-,1-, + 0 + 2,1,...40,39
که در اینجا میشه: 81 عدد
حالا در هر وزن کشی هر وزنه میتونه سمت چپ، راست یا خارج از ترازو باشه یعنی سه حالت داره:1+و 0 و 1- .
پس نیاز ما به حداقل( در مبنای 3) ( log(81
یعنی 4 وزنه.
حالا میسازیم ولی با وزنه های یکسان کار نمی کنیم ( چون در حالت وجود وزنه یکسان lمثلا x3=x4=1 دیگه 81 عدد ساخته نمیشه چون حالتهای x3-x4 , x4-x3 یا x3*0+x4, x4*0+x3 یکسان میشند وما نمی تونیم به 81 عدد برسیم.) از طرفی چون با 4 وزنه حداکثر ( در حالتی که هیچ دو نوع وزنه گذاری نتیجه یکسان نده ) به 81 می رسیم و 81 هم نیازه پس هیچ دو نوع وزنه گذاری نباید نتیجه یکسان بده:
x1+x2+x3+x4 (biggest number)=40
x1+x2+x3=39(second number)=39
پس:x4=1.
x1+x2+x4 or x1+x2+x3-x4=38 or 37
چون فرق این دوعدد x3-2x4هست و x4=1و x3>=2 پس حالت x1+x2+x3-x4=38 & x1+x2+x4=37 درسته.
پس به سادگی داریم: x3=3.
حالا حالتهای بعدی:
بخش قبل :x1+x2+x4=37 پس x1+x2=36 و پس x1+x2-x4=35 و پس x1+x2+x4-x3=34 و پس x1+x2-x3=33 و پسx1+x2-x3-x4=32.
حالا بزرگترین حالت موجود x1+x3+x4 هست پس: x1+x3+x4=31
پس x1=27 پس x2= 9.
x1=27,x2=9,x3=3,x4=1.
نکته مهم دیگه اینه که اگر تعداد بیشتر از 40 باشه لزوما تعداد هم از 4 بیشتره و اگر کمتر باشه لزوما جواب از 4 بیشتر نیست.
+ وزنه ها همیشه توان 3 هستند که با استقرا یا .... میشه ثابت کرد.

خورشید زمستان
12-07-2009, 19:40
با سلام خدمت davy jones عزیز

اگر ممکنه در مورد اعداد گویا به طور ساده یه توضیحی بدهید. خسته شدم این قدر تعریف تکراری و نامفهوم شنیدم . لطفا به صورت قابل فهم به من یاد بدهید . ضمنا تفاوت اعداد گویا به گنگ چیه؟
با تشکر. زنده باد

aran khan95
12-07-2009, 20:25
تین سوال ممکنه از من پرسیده نشده باشه ولی جواب من:
همه ی اعداد که بشود آنها رو به صورت کسری و غیر رادیکالی اعداد گویا هستند(مثل 7 ، 2/5 ، ... )، و اعداد گنگ هم اعدادی به غیر از گویا هستند(رادیکل ها...عدد P).
البته این نظر منه...ممکنه غلط باشه.

کاملا درسته و به عبارتی دیگه عدد گویا حاصل تقسیم دو عدد صحیح بر هم است

SI-SA
12-07-2009, 22:06
من یه سوال دارم:
عدد P از کجا اومده؟

mahsa1469
12-07-2009, 23:02
من یه سوال دارم:
عدد P از کجا اومده؟
برید اینجا فک کنم جوابتون رو پیدا کنید:11:

برای مشاهده محتوا ، لطفا وارد شوید یا ثبت نام کنید

خورشید زمستان
13-07-2009, 01:41
تین سوال ممکنه از من پرسیده نشده باشه ولی جواب من:
همه ی اعداد که بشود آنها رو به صورت کسری و غیر رادیکالی اعداد گویا هستند(مثل 7 ، 2/5 ، ... )، و اعداد گنگ هم اعدادی به غیر از گویا هستند(رادیکل ها...عدد P).
البته این نظر منه...ممکنه غلط باشه.

اختیار دارید دستتون درد نکنه


کاملا درسته و به عبارتی دیگه عدد گویا حاصل تقسیم دو عدد صحیح بر هم است
با تشکر
میتونیم بگیم اعداد اعشاری همه گویا هستند . درهر عدد گویا تعداد ارقام بعد از اعشار مشخص هست مثلا عدد 2/5 همان 2.5 یا 2/1 2 است و تعداد ارقام پشت ممیز یک هست . البته به تعداد نامشخص میتوان صفر اضافه نمود اما به نظر من برای اعداد گنگ اینطور نیست مثلا وقتی میگیم عدد پی 3.14 هست تعداد ارقام بعد از ممیز دو تا نیست بلکه تا بالای 30 رقم اعشار هم عدد پی رو تقریب زده اند !!!! برای اعداد رادیکالی مثل رادیکال 2 ، 3 و ...(غیر از اعداد مربع کامل) هم همینطوره .شاید به دلیل ناتمام بودشون گنگ نامیده شده اند حتی بعضی از اعداد رادیکالی با تعریف مشتق تقریب زده میشن!!!!!!!!!! به هر حال تا اونجا که شنیدم اجتماع دو مجموعه اعداد گویا و گنگ میشه مجموعه اعداد حقیقی . با تشکر از جواباتون

davy jones
13-07-2009, 07:37
معمولا نمیام اینجا ولی این جالب بود:
جواب 4 وزنه هست.
چند نوع ایده غلط هست که سریع به ذهن میرسه اگر این ایده ها به ذهنتون رسید مشکلی نیست یه خورده عجله کردید ولی ذهنتون روشهای خوبی رو امتحان می کنه :
1و2و4و8و16وو32 که میشه 6.
یا از یک روش پله ای استفاده می کنیم: برای تا 40 کیلو یک وزنه 20 کیلو + وزنه هایی فاقد 20 که تا 20 رو بگیره: برای 20: 10 + وزنه های فاقد 10 که تا 10 رو بگیره، برای10 : 5 + فاقد 5 برای تا 5.
حالا با 1و2و3و4 ، 5رو با چندتاش میشه ساخت؟ 3تا: مثلا با 1و2و3 (که میشه ثابت کرد با2تا وزنه نمیشه.) یا بعه عبارتی 3+ تا 2 فاقد 3. که میشه 1و 2
میشه 6تا.

این روش ها هیچ کدوم یک الگوریتم درست نیست چون هر دو در مثال 4 مشکل دارند و جواب 3 عدد 1و2و1 میدهند که جواب درست: 1و3 یعنی دو وزنه هست.
جالا چرا این روشها غلطه؟ چون به دو سمت بودن ترازو دقت نداره.
فرض کنیم وزنه های :
x1,x2,x3,...xn استفاده بشند، سمت راست ترازو رو مثبت وسمت چپ رو منفی میدونیم یعنی آرد سمت چپ هست: با این اعداد باید:
40-,39-,...,2-,1-, + 0 + 2,1,...40,39
که در اینجا میشه: 81 عدد
حالا در هر وزن کشی هر وزنه میتونه سمت چپ، راست یا خارج از ترازو باشه یعنی سه حالت داره:1+و 0 و 1- .
پس نیاز ما به حداقل( در مبنای 3) ( log(81
یعنی 4 وزنه.
حالا میسازیم ولی با وزنه های یکسان کار نمی کنیم ( چون در حالت وجود وزنه یکسان lمثلا x3=x4=1 دیگه 81 عدد ساخته نمیشه چون حالتهای x3-x4 , x4-x3 یا x3*0+x4, x4*0+x3 یکسان میشند وما نمی تونیم به 81 عدد برسیم.) از طرفی چون با 4 وزنه حداکثر ( در حالتی که هیچ دو نوع وزنه گذاری نتیجه یکسان نده ) به 81 می رسیم و 81 هم نیازه پس هیچ دو نوع وزنه گذاری نباید نتیجه یکسان بده:
x1+x2+x3+x4 (biggest number)=40
x1+x2+x3=39(second number)=39
پس:x4=1.
x1+x2+x4 or x1+x2+x3-x4=38 or 37
چون فرق این دوعدد x3-2x4هست و x4=1و x3>=2 پس حالت x1+x2+x3-x4=38 & x1+x2+x4=37 درسته.
پس به سادگی داریم: x3=3.
حالا حالتهای بعدی:
بخش قبل :x1+x2+x4=37 پس x1+x2=36 و پس x1+x2-x4=35 و پس x1+x2+x4-x3=34 و پس x1+x2-x3=33 و پسx1+x2-x3-x4=32.
حالا بزرگترین حالت موجود x1+x3+x4 هست پس: x1+x3+x4=31
پس x1=27 پس x2= 9.
x1=27,x2=9,x3=3,x4=1.
نکته مهم دیگه اینه که اگر تعداد بیشتر از 40 باشه لزوما تعداد هم از 4 بیشتره و اگر کمتر باشه لزوما جواب از 4 بیشتر نیست.
+ وزنه ها همیشه توان 3 هستند که با استقرا یا .... میشه ثابت کرد.

ممنونم!
جواب درسته و توضيحاتي هم كه داديد فوق العاده و عالي بود.

موفق باشين.
88/4/22

davy jones
13-07-2009, 07:46
اختیار دارید دستتون درد نکنه


با تشکر
میتونیم بگیم اعداد اعشاری همه گویا هستند . درهر عدد گویا تعداد ارقام بعد از اعشار مشخص هست مثلا عدد 2/5 همان 2.5 یا 2/1 2 است و تعداد ارقام پشت ممیز یک هست . البته به تعداد نامشخص میتوان صفر اضافه نمود اما به نظر من برای اعداد گنگ اینطور نیست مثلا وقتی میگیم عدد پی 3.14 هست تعداد ارقام بعد از ممیز دو تا نیست بلکه تا بالای 30 رقم اعشار هم عدد پی رو تقریب زده اند !!!! برای اعداد رادیکالی مثل رادیکال 2 ، 3 و ...(غیر از اعداد مربع کامل) هم همینطوره .شاید به دلیل ناتمام بودشون گنگ نامیده شده اند حتی بعضی از اعداد رادیکالی با تعریف مشتق تقریب زده میشن!!!!!!!!!! به هر حال تا اونجا که شنیدم اجتماع دو مجموعه اعداد گویا و گنگ میشه مجموعه اعداد حقیقی . با تشکر از جواباتون

يه نكته در مورد فرمايشاتتون قابل تامله. اگه عددي داراي ارقامي به صورت اعشاري باشه، در صورتي جزء اعداد گنگ محسوب ميشه كه تعداد ارقام بعد از مميز بينهايت باشه و ضمنا هيچ الگوريتم تكرار شونده اي هم در اين ارقام پيدا نشه. اگه تعداد ارقام اعشاري يك عدد ، بينهايت تا باشه ولي مثلا همشون 2 باشن يا رقم 24 مرتبا تكرار بشه، اين عدد گويا خواهد بود و ميتوان كسر گوياي مربوط به اون عدد رو بدست آورد. در زير با ارائه چند مثال چگونگي اين كار رو توضيح ميدم:
[ برای مشاهده لینک ، لطفا با نام کاربری خود وارد شوید یا ثبت نام کنید ]

یه مثال پیچیده تر :
[ برای مشاهده لینک ، لطفا با نام کاربری خود وارد شوید یا ثبت نام کنید ]

و یا حتی اعدادی شبیه این:
[ برای مشاهده لینک ، لطفا با نام کاربری خود وارد شوید یا ثبت نام کنید ]

امیدوارم تونسته باشم منظورم رو کامل برسونم.

موفق باشین.
88/4/22

davy jones
13-07-2009, 08:07
من یه سوال دارم:
عدد P از کجا اومده؟

در واقع به زبون خیلی ساده عدد pi حاصل تقسیم محیط هر دایره به قطر آن دایره است. این که این حاصل تقسیم همواره عدد ثابتی میشود رو اولین بار یونانیان باستان مشاهده و ثبت کرده اند.

موفق باشین.
88/4/22

خورشید زمستان
13-07-2009, 18:45
يه نكته در مورد فرمايشاتتون قابل تامله. اگه عددي داراي ارقامي به صورت اعشاري باشه، در صورتي جزء اعداد گنگ محسوب ميشه كه تعداد ارقام بعد از مميز بينهايت باشه و ضمنا هيچ الگوريتم تكرار شونده اي هم در اين ارقام پيدا نشه. اگه تعداد ارقام اعشاري يك عدد ، بينهايت تا باشه ولي مثلا همشون 2 باشن يا رقم 24 مرتبا تكرار بشه، اين عدد گويا خواهد بود و ميتوان كسر گوياي مربوط به اون عدد رو بدست آورد. در زير با ارائه چند مثال چگونگي اين كار رو توضيح ميدم:
[ برای مشاهده لینک ، لطفا با نام کاربری خود وارد شوید یا ثبت نام کنید ]

یه مثال پیچیده تر :
[ برای مشاهده لینک ، لطفا با نام کاربری خود وارد شوید یا ثبت نام کنید ]

و یا حتی اعدادی شبیه این:
[ برای مشاهده لینک ، لطفا با نام کاربری خود وارد شوید یا ثبت نام کنید ]

امیدوارم تونسته باشم منظورم رو کامل برسونم.

موفق باشین.
88/4/22
بله این نمونه عدد اعشاری اصطلاحا دارای مولد اعشای هست که بینهایت تکرار میشه ولی تعداد تکرار مشخصه :46: با یه حساب ساده فک کنم در سطح کتاب دوم دبیرستان میشه کسر مولد اعشاری رو حساب کرد . این حالت استثنائا گویاست (با وجود تکرار بینهایت چند عدد مشخص)چون به فرم کسری همونطور که اثبات کردید درمیاد
با تشکر فراوان

خورشید زمستان
13-07-2009, 19:00
من یه سوال دارم:
عدد P از کجا اومده؟


در واقع به زبون خیلی ساده عدد pi حاصل تقسیم محیط هر دایره به قطر آن دایره است. این که این حاصل تقسیم همواره عدد ثابتی میشود رو اولین بار یونانیان باستان مشاهده و ثبت کرده اند.

موفق باشین.
88/4/22

یک آزمایش ساده

اگر حوصله دارید ، بر روی یک کاغذ مقوایی نسبتا ضخیم(تقریبا 3 -4 میلیمتر ) چند دایره به شعاعهای مختلف و جداگانه بوسیله پرگار رسم کنید . حالا به دقت و با استفاده از یک چاقوی موکت بری یا قیچی متاسب دایره ها را از صفحه جدا کنید. و محیط پیرامون تک تک دایره ها را با نخ اندازه بگیرید و سپس طول نخ را به دقت توسط یک خط کش مدرج دقیق یا کولیس اندازه گرفته به همراه قطر دایره مذکور بر روی کاغذ یادداشت کنید . حالا اندازه های طول نخها که همان محیط دایره هاست را بر قطر (هردایره جداگانه) تقسیم کنید تا به عدد نسبتا ثابت 3.14... برسید . البته برای خطای کمتر میتوان از سیم خشک برقکاری به جای نخ استفاده نمود.

aran khan95
13-07-2009, 20:11
ممنونم!
جواب درسته و توضيحاتي هم كه داديد فوق العاده و عالي بود.

موفق باشين.
88/4/22
قبل از اینکه مصری ها ازش استفاده کنن؟

davy jones
14-07-2009, 08:41
یه سوال دیگه:
فرض کنید 9 تا نقطه به صورت زیر چیده شده باشن:
[ برای مشاهده لینک ، لطفا با نام کاربری خود وارد شوید یا ثبت نام کنید ]

فرض کنیم برای اینکه این نقاط رو به هم وصل کنیم فقط مجاز به استفاده از خطوط راست هستیم یعنی منحنی نداریم. اگر در خط راست ما شکستگی هم وجود داشته باشد به این معنی که راستای خط عوض شود آنوقت آن را 2 تا خط میشماریم.
حال حداقل با چند خط راست و بدون اینکه خودکار (یا مداد) را از روی کاغذ بلند کنیم میتونیم همه ی نقاط رو به هم وصل کنیم؟
هر کس هر عددی رو میگه لطفا با شکل بذاره تا همه متوجه منظورش بشن و اینکه چطوری به اون راه حل رسیده.

موفق باشین.
88/4/20


5 تا...
اینجوری:
[ برای مشاهده لینک ، لطفا با نام کاربری خود وارد شوید یا ثبت نام کنید ]


ممنونم. ولی فکر کنم با 4 تا خط هم میشه. یه کم روش فکر کن. اگه کس دیگه ای هم نظری داره لطفا بگه.

موفق باشین.
88/4/21



اینم جوابش:
[ برای مشاهده لینک ، لطفا با نام کاربری خود وارد شوید یا ثبت نام کنید ]

موفق باشین.
88/4/23

amintnt
14-07-2009, 09:19
اینم جوابش:
[ برای مشاهده لینک ، لطفا با نام کاربری خود وارد شوید یا ثبت نام کنید ]

موفق باشین.
88/4/23
به تست هوش بیشتر شبیه.... یادمه تو یکی از صفحات یکی پرسیده بود چطور پلی که تحمل 500 کیلو وزن رو داره یک ماشین 1 تنی از روش رد میشه؟ جوابای علمی داده میشد که یکشون که من یادمه این بود که پل اونقدر کوچیکه که در یک لحظه فقط یک طرف ماشین از روش رد میشه. بعد طراح سوال اومد جواب رو گفت و همه رو مبهوت کرد! چون بیمه ی حضرت عباس بوده(یه چیزی شبیه این).

ساختار شکنی ذهن سخته، ذهن خلاق میخواد.

شخصا فکر میکردم فقط میشه روی همون دایره ها نوسان داشت.

ممنون!

davy jones
14-07-2009, 09:58
یه سوال تو مایه های همون ذهن خلاق و ساختار شکن:

همونطور که در شکل زیر میبینین مثلا با چوب کبریت نوشته شده 11=71
با جابجا کردن ( یا برداشتن) تنها یک چوب کبریت کاری کنبد که تساوی برقرار شود.

[ برای مشاهده لینک ، لطفا با نام کاربری خود وارد شوید یا ثبت نام کنید ]

موفق باشین.
88/4/23

amintnt
14-07-2009, 11:20
یه سوال تو مایه های همون ذهن خلاق و ساختار شکن:

همونطور که در شکل زیر میبینین مثلا با چوب کبریت نوشته شده 11=71
با جابجا کردن ( یا برداشتن) تنها یک چوب کبریت کاری کنبد که تساوی برقرار شود.

[ برای مشاهده لینک ، لطفا با نام کاربری خود وارد شوید یا ثبت نام کنید ]

موفق باشین.
88/4/23
کردیمش نامساوی درست شد! :27:

[ برای مشاهده لینک ، لطفا با نام کاربری خود وارد شوید یا ثبت نام کنید ]

davy jones
14-07-2009, 11:35
کردیمش نامساوی درست شد! :27:

[ برای مشاهده لینک ، لطفا با نام کاربری خود وارد شوید یا ثبت نام کنید ]

خسته نباشی دوست عزیز!:31: تنهایی گفتی یا کسی هم کمکت کرد؟ (شوخی کردم)
نباید به مساوی دست بزنی.

موفق باشین.
88/4/23

mahsa1469
14-07-2009, 11:37
خسته نباشی دوست عزیز!:31: تنهایی گفتی یا کسی هم کمکت کرد؟ (شوخی کردم)
نباید به مساوی دست بزنی.

موفق باشین.
88/4/23
خوب به یه چیط درست رسید دیگه:31:

davy jones
14-07-2009, 11:42
خوب به یه چیط درست رسید دیگه:31:

با جابجا کردن ( یا برداشتن) تنها یک چوب کبریت کاری کنبد که تساوی برقرار شود.

موفق باشین.
88/4/23

yugioh
14-07-2009, 11:57
یه سوال تو مایه های همون ذهن خلاق و ساختار شکن:

همونطور که در شکل زیر میبینین مثلا با چوب کبریت نوشته شده 11=71
با جابجا کردن ( یا برداشتن) تنها یک چوب کبریت کاری کنبد که تساوی برقرار شود.

[ برای مشاهده لینک ، لطفا با نام کاربری خود وارد شوید یا ثبت نام کنید ]

موفق باشین.
88/4/23

مرا مشتاق نمودی، قشنگه.
v را به x تبدیل می کنیم:
xi=11
xi در عدد نویسی رومی. , 11 در عربی.

davy jones
14-07-2009, 12:03
مرا مشتاق نمودی، قشنگه.
v را به x تبدیل می کنیم:
xi=11
xi در عدد نویسی رومی. , 11 در عربی.

آفرین! ایده جالبی بود. درسته ولی من یه چیز دیگه تو ذهنم بود اونم اینه:

[ برای مشاهده لینک ، لطفا با نام کاربری خود وارد شوید یا ثبت نام کنید ]

در شکل بالا نشون میده که 1 مساوی با رادیکال یکه.

موفق باشین.
88/4/23

mahsa1469
14-07-2009, 12:07
مرا مشتاق نمودی، قشنگه.
v را به x تبدیل می کنیم:
xi=11
xi در عدد نویسی رومی. , 11 در عربی.


آفرین! ایده جالبی بود. درسته ولی من یه چیز دیگه تو ذهنم بود اونم اینه:

[ برای مشاهده لینک ، لطفا با نام کاربری خود وارد شوید یا ثبت نام کنید ]

در شکل بالا نشون میده که 1 مساوی با رادیکال یکه.

موفق باشین.
88/4/23
جفتشون جالب بود بازم از اینا بزار:11:

davy jones
14-07-2009, 12:17
جفتشون جالب بود بازم از اینا بزار:11:
پیشنهاد میکنم به این آدرس مراجعه کنین. اونجا پره از بازی هایی که ذهنتون رو خلاق و ساختارشکن میکنه!:27: البته بعضی از بازی هاش هم فقط جنبه ی سرگرمی داره.

برای مشاهده محتوا ، لطفا وارد شوید یا ثبت نام کنید

موفق باشین.
88/4/23

amintnt
14-07-2009, 15:53
راستی، اون قورباغه هه بود؟ فکر کردم باچه تعبیری میشه مستقیما به 2 به توان 28 رسید، دیدم ای بابا. اون همه راه حل رفتم خودمو خسته کردم! پله ی اول رو یا میره یا نمیره، 2 حالت، پله ی دوم هم یا میره یا نمیره، دو حالت و ... میشه 2 به توان 28!

راستی، بابت سوالا ممنون، الآن حس میکنم باید یه چیزی رو بشکنم، ساختار یا شیشه هم فرقی نمیکنه!

siavash_1147
14-07-2009, 22:38
دوستان اگه [x^2]=[x^3] باشه
بید نتیجه گرفت x در چه بازه ایه؟

davy jones
15-07-2009, 11:39
دوستان اگه [x^2]=[x^3] باشه
بید نتیجه گرفت x در چه بازه ایه؟

منظورت از ] و [ براکته؟
اگه منظورت اینه x بین بازه صفر تا فرجه سوم 2 قرار میگیره.

موفق باشین.
88/4/24

siavash_1147
15-07-2009, 11:58
منظورت از ] و [ براکته؟
اگه منظورت اینه x بین بازه صفر تا فرجه سوم 2 قرار میگیره.

موفق باشین.
88/4/24


راه تشریحی نداره؟
منظورم فقط توان 3 نیست
هر توان فردی

سوالمو اینجوری صلاح میکنم

[x]=[x^2k-1]

davy jones
15-07-2009, 13:58
راه تشریحی نداره؟
منظورم فقط توان 3 نیست
هر توان فردی

سوالمو اینجوری صلاح میکنم

[x]=[x^2k-1]

میدونیم که برای به دست آوردن نمودار تابعی مانند [x^2k-1] از روی نمودار تابع x^2k-1 کافیه که نمودار x^2k-1 رو روی خطهای افقی y=c که c ها اعداد صحیح هستند رو به سمت پایین تصویر کنیم. برای مثال: y=x^3 و در x>0:
[ برای مشاهده لینک ، لطفا با نام کاربری خود وارد شوید یا ثبت نام کنید ]

که به ترتیب نقاط تو خالی عبارتست از : 1 ، فرجه سوم 2 ، فرجه سوم 3 ، فرجه سوم 4 و ....
حال در حالت کلی برای هر توان فردی مثل 2k-1 این نقاطی که در اونها گسستگی داریم عبارتند از:
1 ، فرجه 2k-1 ام 2 ، فرجه 2k-1 ام 3 و ...
حال در رابطه تساوی[x]=[x^2k-1] واضح است که بازه مشترک عبارت است از منفی فرجه 2k-1 ام 2 تا مثبت فرجه 2k-1 ام 2.
در حالت کلی تر و هنگامی که داریم :

[ برای مشاهده لینک ، لطفا با نام کاربری خود وارد شوید یا ثبت نام کنید ]

در صورتی که m از n بزرگتر باشد و هر دو فرد باشد بازه جواب از منفی فرجه m ام 2 تا مثبت فرجه m ام 2 است.

موفق باشین.
88/4/24

davy jones
15-07-2009, 14:13
جفتشون جالب بود بازم از اینا بزار:11:
بفرما اینم یه سوال دیگه از همین دست.
این سوال رو توی یه نظرسنجی علمی از بچه های پیش دبستانی پرسیده بودن و جالب بوده که تقریبا همه ی اونها درست جواب دادن.
سوال اینه:
اگه فرض کنین که شکل زیر مینی بوسی باشه که شماها رو تا مهد کودک میاره و الان هم در حال حرکت باشه به سمت چپ در حال حرکته یا سمت راست؟
[ برای مشاهده لینک ، لطفا با نام کاربری خود وارد شوید یا ثبت نام کنید ]

نظر شما چیه؟
لطفا با ذکر دلیل پست بدین.
نکته : اگه تو شکل کمی عدم تقارن میبینین اونو به حساب نقاشی ضعیف من بذارین و شکل رو کاملا متقارن در نظر بگیرین.

موفق باشین.
88/4/24

davy jones
15-07-2009, 15:04
یه سوال که جوابشو خودمم نمیدونم ( اگه کسی میدونه لطفا راهنمایی کنه.) :
در صفحه 8*8 شطرنج اگر فقط یه مهره ی اسب داشته باشیم آیا میتوان با حرکت L مانند اسب طوری در صفحه ی شطرنج حرکت کنیم که همه ی خانه های صفحه فقط و فقط یکبار مهره در درونشان قرار گیرد؟

[ برای مشاهده لینک ، لطفا با نام کاربری خود وارد شوید یا ثبت نام کنید ]

و

[ برای مشاهده لینک ، لطفا با نام کاربری خود وارد شوید یا ثبت نام کنید ]

و

[ برای مشاهده لینک ، لطفا با نام کاربری خود وارد شوید یا ثبت نام کنید ]

و ...

----------------------------------
سوال دوم: آیا جواب دادن به سوال قبل به اینکه اسب در حرکت اول در کدام خانه واقع شده است بستگی دارد یا خیر؟

موفق باشین.
88/4/24

yugioh
15-07-2009, 17:05
یه سوال که جوابشو خودمم نمیدونم ( اگه کسی میدونه لطفا راهنمایی کنه.) :
در صفحه 8*8 شطرنج اگر فقط یه مهره ی اسب داشته باشیم آیا میتوان با حرکت L مانند اسب طوری در صفحه ی شطرنج حرکت کنیم که همه ی خانه های صفحه فقط و فقط یکبار مهره در درونشان قرار گیرد؟

[ برای مشاهده لینک ، لطفا با نام کاربری خود وارد شوید یا ثبت نام کنید ]

و

[ برای مشاهده لینک ، لطفا با نام کاربری خود وارد شوید یا ثبت نام کنید ]

و

[ برای مشاهده لینک ، لطفا با نام کاربری خود وارد شوید یا ثبت نام کنید ]

و ...

----------------------------------
سوال دوم: آیا جواب دادن به سوال قبل به اینکه اسب در حرکت اول در کدام خانه واقع شده است بستگی دارد یا خیر؟

موفق باشین.
88/4/24

سوالات باحال بودن شاد شدم گفت بیام راهنماییت:31:

1.آره میشه برای هر مربع بزرگتر از 4*4 میشه. ولی اگر فرد باشه تعداد بوضوح به خونه اولش برنمی گرده چون مثلا تو 5در 5 اگر از سیاه شروع کنه در هر حرکت رنگ عوض میکنه و بعد از 24 حرکت دوباره میاد تو سیاه.
2. نه حداقل برای 8*8 لزومی نیست. چون تو خیلی از این راهها به خونه اصلیت ( در مورد تعداد زوج) بر می گردی. بنابراین یه دوره و فرق نداره تو از کجا یک دور رو شروع کنی. ولی در مورد فردها نمی دونم،
یه نکته اینکه این لینک رو هم ببین، تعداد راهها رو هم ببین:
[ برای مشاهده لینک ، لطفا با نام کاربری خود وارد شوید یا ثبت نام کنید ]


برای مشاهده محتوا ، لطفا وارد شوید یا ثبت نام کنید


برای مشاهده محتوا ، لطفا وارد شوید یا ثبت نام کنید
[ برای مشاهده لینک ، لطفا با نام کاربری خود وارد شوید یا ثبت نام کنید ]

amintnt
15-07-2009, 18:35
بفرما اینم یه سوال دیگه از همین دست.
این سوال رو توی یه نظرسنجی علمی از بچه های پیش دبستانی پرسیده بودن و جالب بوده که تقریبا همه ی اونها درست جواب دادن.
سوال اینه:
اگه فرض کنین که شکل زیر مینی بوسی باشه که شماها رو تا مهد کودک میاره و الان هم در حال حرکت باشه به سمت چپ در حال حرکته یا سمت راست؟
[ برای مشاهده لینک ، لطفا با نام کاربری خود وارد شوید یا ثبت نام کنید ]

نظر شما چیه؟
لطفا با ذکر دلیل پست بدین.
نکته : اگه تو شکل کمی عدم تقارن میبینین اونو به حساب نقاشی ضعیف من بذارین و شکل رو کاملا متقارن در نظر بگیرین.

موفق باشین.
88/4/24
من قبلا این سوال رو دیدم... چه کنم؟ بگم؟ فکر کنم روی اینترنتم به وفور یافت بشه! من خودم اینو توی همین سایت انجمن متفرقه دیدم :دی

Numb
15-07-2009, 22:06
بفرما اینم یه سوال دیگه از همین دست.
این سوال رو توی یه نظرسنجی علمی از بچه های پیش دبستانی پرسیده بودن و جالب بوده که تقریبا همه ی اونها درست جواب دادن.
سوال اینه:
اگه فرض کنین که شکل زیر مینی بوسی باشه که شماها رو تا مهد کودک میاره و الان هم در حال حرکت باشه به سمت چپ در حال حرکته یا سمت راست؟
[ برای مشاهده لینک ، لطفا با نام کاربری خود وارد شوید یا ثبت نام کنید ]

نظر شما چیه؟
لطفا با ذکر دلیل پست بدین.
نکته : اگه تو شکل کمی عدم تقارن میبینین اونو به حساب نقاشی ضعیف من بذارین و شکل رو کاملا متقارن در نظر بگیرین.

موفق باشین.
88/4/24

be samte chap chon ke daresh samte rasteshe

ask_bl
15-07-2009, 22:09
be samte chap chon ke daresh samte rasteshe
خوب شاید داره دنده عقب میره!

davy jones
16-07-2009, 09:02
خوب شاید داره دنده عقب میره!

نه ديگه جواب درسته. البته اگه سرويس مهدكودك از اين اتوبوسهاي BRT باشه اونوقت جواب فرق ميكنه.:27:

موفق باشين.
25/4/88

eh_mn
16-07-2009, 13:08
دوستان اگه [x^2]=[x^3] باشه
بید نتیجه گرفت x در چه بازه ایه؟

ميشه ثابت كرد كه اگر [x]=[y] آنگاه اختلاف x و y از يك كمتره.
بنابراين براي حل معادله [x^n]=[x^m] كافيه نامعادله [ برای مشاهده لینک ، لطفا با نام کاربری خود وارد شوید یا ثبت نام کنید ] رو حل كنيم. فرض كنيم m يك عدد فرد، n يك عدد زوج و m>n. قرار ميدهيم f(x)=x^m-x^n. اگر [ برای مشاهده لینک ، لطفا با نام کاربری خود وارد شوید یا ثبت نام کنید ] آنگاه واضحه كه [ برای مشاهده لینک ، لطفا با نام کاربری خود وارد شوید یا ثبت نام کنید ] . از طرفي ميشه نشون داد كه تابع f بر مجموعه [ برای مشاهده لینک ، لطفا با نام کاربری خود وارد شوید یا ثبت نام کنید ] صعوديه. حالا فرض كنيم a و b اعدادي باشند كه f(a)=-f(b)=1. در اينصورت جواب مساله بازه
[ برای مشاهده لینک ، لطفا با نام کاربری خود وارد شوید یا ثبت نام کنید ] است. حالا a و b رو چطور بدست بياريم؟

eh_mn
16-07-2009, 16:14
من یه سوال دارم:
عدد P از کجا اومده؟

يه مقاله در مورد اصم بودن عدد pi.


برای مشاهده محتوا ، لطفا وارد شوید یا ثبت نام کنید

davy jones
16-07-2009, 17:08
یه سوال جالب:
میتونید یه لیوانی رو تصور کنید که اگه اون رو کاملا از رنگ پر کنیم طوری که از سر لیوان سرازیر شود و بعد آن را بلافاصله خالی کنیم، مشاهده کنیم که جداره ی داخلی لیوان هنوز کاملا رنگ نشده ؟
روش یه کم فکر کنین. جوابشو بعدا میذارم

موفق باشین.
88/4/25

davy jones
17-07-2009, 09:00
سلام. جواب این انتگرال چی میشه؟

[ برای مشاهده لینک ، لطفا با نام کاربری خود وارد شوید یا ثبت نام کنید ]{dx}{tg(x)ln(sinx)}

موفق باشین.
88/4/26

davy jones
18-07-2009, 08:08
یه سوال جالب:
میتونید یه لیوانی رو تصور کنید که اگه اون رو کاملا از رنگ پر کنیم طوری که از سر لیوان سرازیر شود و بعد آن را بلافاصله خالی کنیم، مشاهده کنیم که جداره ی داخلی لیوان هنوز کاملا رنگ نشده ؟
روش یه کم فکر کنین. جوابشو بعدا میذارم

موفق باشین.
88/4/25

برای درک بهتر از اون چیزی که میخوام بگم ابتدا یه مقدمه ای میگم:
تابع معروف زیر رو در نظر بگیرید.

[ برای مشاهده لینک ، لطفا با نام کاربری خود وارد شوید یا ثبت نام کنید ]

[ برای مشاهده لینک ، لطفا با نام کاربری خود وارد شوید یا ثبت نام کنید ]

برای این تابع و به ازای x>1 ، انتگرال این تابع برابر است با:

[ برای مشاهده لینک ، لطفا با نام کاربری خود وارد شوید یا ثبت نام کنید ] 7Bdx%7D%7Bx%7D%5Cto&space;%5Cinfty

[ برای مشاهده لینک ، لطفا با نام کاربری خود وارد شوید یا ثبت نام کنید ]

حال اگر نمودار این تابع رو حول محور x دوران بدیم، شکلی شبیه شیپور بدست میآید:

[ برای مشاهده لینک ، لطفا با نام کاربری خود وارد شوید یا ثبت نام کنید ]

اگر بخواهیم حجم داخل آن را حساب کنیم خواهیم داشت:

[ برای مشاهده لینک ، لطفا با نام کاربری خود وارد شوید یا ثبت نام کنید ] 7B%5Cpi&space;%7D%7Bx%5E2%7Ddx=2%5Cpi

که در اینجا جزء دیفرانسیلی عبارت است از دیسکهایی به مرکز مبدا و شعاع y و قطر dx .

همانطور که مشاهده کردین، مساحت زیر نمدار اول به سمت بینهایت میل میکند ولی حجم تابع دوران یافته ی آن حول محور x عدد ثابتی است!

حال میخواهیم مساحت سطح جانبی شکل دوران یافته را بدست آوریم:

[ برای مشاهده لینک ، لطفا با نام کاربری خود وارد شوید یا ثبت نام کنید ] c%7B2%5Cpi&space;dx%7D%7Bx%7D=2%5Cpi&space;%5Cint_%7B1%7D%5E%7 B%5Cinfty&space;%7D%5Cfrac%7B1%7D%7Bx%7Ddx%5Cto&space;%5Cinfty

که در اینجا جزء دیفرانسیلی برابر است با نوارهایی مستطیلی که طول آنها برابر با محیط دایره ای به شعاع y و عرض dx است.

حال نکته ای که در ابتدا به آن اشاره کردیم قابل تامل است و آن این است که میتوانیم این شکل شیپور مانند رو به یک لیوان تشبیه کنیم که چون حجم داخل آن محدود است میتوان تمام آن را پر از رنگ کرد طوری که حتی از لب آن سر برود ولی وقتی که لیوان را خالی میکنیم میبینیم که هنوز تمام جداره آن رنگ نشده است! چون مساحت جانبی آن بینهایت است.

امیدوارم مثمر ثمر بوده باشد.

موفق باشین.
88/4/27

zahedy2006
18-07-2009, 11:18
سلام. جواب این انتگرال چی میشه؟

[ برای مشاهده لینک ، لطفا با نام کاربری خود وارد شوید یا ثبت نام کنید ] inx%29%7D

موفق باشین.
88/4/26 اگر u=sin باشد داريم

[ برای مشاهده لینک ، لطفا با نام کاربری خود وارد شوید یا ثبت نام کنید ]*ln%28u%29%7 D

انتگرال du تقسيم بر u*ln(u)d .

حال اگر y=ln(U) d باشد

[ برای مشاهده لینک ، لطفا با نام کاربری خود وارد شوید یا ثبت نام کنید ]

اينم كه ميشه همون ln(y)d پس داريم

[ برای مشاهده لینک ، لطفا با نام کاربری خود وارد شوید یا ثبت نام کنید ]

davy jones
18-07-2009, 11:37
اگر u=sin باشد داريم

[ برای مشاهده لینک ، لطفا با نام کاربری خود وارد شوید یا ثبت نام کنید ]*ln%28u%29%7 D

انتگرال du تقسيم بر u*ln(u)d .

حال اگر y=ln(U) d باشد

[ برای مشاهده لینک ، لطفا با نام کاربری خود وارد شوید یا ثبت نام کنید ]

اينم كه ميشه همون ln(y)d پس داريم

[ برای مشاهده لینک ، لطفا با نام کاربری خود وارد شوید یا ثبت نام کنید ]





راه حل درسته ولي جواب آخر ميشه :

[ برای مشاهده لینک ، لطفا با نام کاربری خود وارد شوید یا ثبت نام کنید ]

موفق باشين.
88/4/27

davy jones
18-07-2009, 11:49
يه سوال عشقي! :
كسي ميتونه با يك تابع يك ضابطه اي شكل يه قلب رو در بياره؟

موفق باشين.
88/4/27

amintnt
18-07-2009, 12:47
يه سوال عشقي! :
كسي ميتونه با يك تابع يك ضابطه اي شكل يه قلب رو در بياره؟

موفق باشين.
88/4/27
توی کتاب "مجانب ها و رسم منحنی" همین مسئله توجهمو جلب کرده بود! البته دیگه تابع نیست، بلکه یه ضابطه است فقط!
[ برای مشاهده لینک ، لطفا با نام کاربری خود وارد شوید یا ثبت نام کنید ]

zahedy2006
18-07-2009, 13:41
يه سوال عشقي! :
كسي ميتونه با يك تابع يك ضابطه اي شكل يه قلب رو در بياره؟

موفق باشين.
88/4/27 با يك تابع يك ضابطه اي؟
با توجه به شكل قلب كه فكر نكنم تابع بشه ولي اون منحني ها دل نما (سيكلوئيد فكر كنم ) هستند


راه حل درسته ولي جواب آخر ميشه :

[ برای مشاهده لینک ، لطفا با نام کاربری خود وارد شوید یا ثبت نام کنید ]

موفق باشين.
88/4/27

درسته
حواسم نبود من U را سينوس گرفته ام!!!

davy jones
18-07-2009, 14:45
چرا من همش بايد سوال طرح كنم؟:19:
يكم شما به خودتون زحمت بدين و سوال در بيارين.:13:

موفق باشين.
88/4/27

خورشید زمستان
18-07-2009, 19:09
چرا من همش بايد سوال طرح كنم؟:19:
يكم شما به خودتون زحمت بدين و سوال در بيارين.:13:

موفق باشين.
88/4/27

مادری برای تحویل گرفتن جسد فرزندش از میدان تجریش شروع به حرکت مینماید تا در نهایت جسد فرزندش را در سردخانه کشتارگاه تحویل بگیرد . اگر بخواهد به طور غیر مستقیم با استفاده از اتوبوس خط واحد به کشتارگاه برود تا میدان انقلاب 2 مسیر ممکن و از انقلاب به کشتارگاه 3 مسیر ممکن و بطور مستقیم از دو مسیر میتواند از تجریش به کشتارگاه برود . حالا :

a - به چند راه ممکن میتواند از تجریش به کشتارگاه برود؟
b - به چند راه ممکن میتواند مسیر رفت و برگشتی از مبدا به مقصد داشته باشد؟
c - چند تا از مسیرهای پیموده شده قسمت قبل به گونه ای هستند که در مسیر بازگشت از کشتارگاه به تجریش ، از جاده های قبلا پیموده شده استفاده نمی شود؟

amintnt
18-07-2009, 19:50
مادری برای تحویل گرفتن جسد فرزندش از میدان تجریش شروع به حرکت مینماید تا در نهایت جسد فرزندش را در سردخانه کشتارگاه تحویل بگیرد . اگر بخواهد به طور غیر مستقیم با استفاده از اتوبوس خط واحد به کشتارگاه برود تا میدان انقلاب 2 مسیر ممکن و از انقلاب به کشتارگاه 3 مسیر ممکن و بطور مستقیم از دو مسیر میتواند از تجریش به کشتارگاه برود . حالا :

a - به چند راه ممکن میتواند از تجریش به کشتارگاه برود؟
b - به چند راه ممکن میتواند مسیر رفت و برگشتی از مبدا به مقصد داشته باشد؟
c - چند تا از مسیرهای پیموده شده قسمت قبل به گونه ای هستند که در مسیر بازگشت از کشتارگاه به تجریش ، از جاده های قبلا پیموده شده استفاده نمی شود؟
A. به دو طریق مستقیم میتواند برود و به 2 در 3 طریق غیر مستقیم. بنابراین به هشت طریق میتواند به مقصد رود.
B. برای رفت به 8 طریق میتواند اقدام کند و برای برگشت هم همینطور. و به هشت در هشت طریق میتواند برود و بازگردد.
C. یا فقط از مسیر های مستقیم استفاده میکند که میشود 2 مسیر، یا از مسیر مستقیم و غیر مستقیم به صورت ترکیبی استفاده که به 2 در 3 در 2 مسیر امکان دارد(بدون در نظر گرفتن تفاوت در رفت و بازگشت)، یا تنها از مسیرهای غیر مستقیم استفاده میکند که به 2 در 3 در 2 در 1 طریق امکان پذیر است. و مجموعا به 26 طریق میتواند.


راستی، حالا می رفت سبزی هم بخره مسیرا تفاوتی نداشت! سوال ترسناک شده بود :دی

zahedy2006
18-07-2009, 19:54
a - به چند راه ممکن میتواند از تجریش به کشتارگاه برود؟

8


b - به چند راه ممکن میتواند مسیر رفت و برگشتی از مبدا به مقصد داشته باشد؟

c(2,8)0


c - چند تا از مسیرهای پیموده شده قسمت قبل به گونه ای هستند که در مسیر بازگشت از کشتارگاه به تجریش ، از جاده های قبلا پیموده شده استفاده نمی شود؟

مسير رفت 8
مسير يرگشت متشكل از 2 مسير فرعي و 2 مسير اصلي كه ميشه 4 يا 6 مسير فرعي +1 مسير اصلي كه ميشه 7

خورشید زمستان
18-07-2009, 21:31
با تشکر از دوستان عزیز:

قسمت a کاملا درسته بطور غیر مستقیم 2*3=6 مسیر و مستقیم هم که 2 تا پس مجموعا 8 مسیر
در مورد قسمت b برای رفتن به کشتارگاه 8 راه و همین مسیر هم برای برگشت صدق میکنه پس کل مسیرهای ممکن 8*8=64 تا هست :46:طبق اصل زیر :

اصل ضرب : اگر انجام کار w وابسته به انجام کار w1 و w2 و ....و wk باشد آنگاه کار w را میتوان به n1.n2....nk طریق میتوان نجام داد .
اصل جمع :اگر انجام کار w وابسته به انجام کار w1 یاw2 یا ....یا wk باشد آنگاه کار w را میتوان به n1+n2....+nk طریق میتوان نجام داد .
کلا کار wi (با اندیس i) با ni طریق میتوان انجام داد.
خب برای پیمودن مسیر رفت و برگشت وابسته به پیمودن 8 طریق رفت و 8 طریق برگشت هست و طبق اصل ضرب کلا 8*8=64
در مورد قسمت c مساله مثل تعداد کلمات دو حرفی است که با استفاده از 8 حرف میتوان نوشت به طوری که حرف دوم تکراری نباشه پس کلا 7*8=56 مسیر ممکن میتوان طی کرد .:46:

اما به نظرم حالی برای مادره نمیمونه که جسد فرزندشو تحویل بگیره و شاید به مسیر برگشت هم نرسه !!! :31:

amintnt
18-07-2009, 21:53
با تشکر از دوستان عزیز:

قسمت a کاملا درسته بطور غیر مستقیم 2*3=6 مسیر و مستقیم هم که 2 تا پس مجموعا 8 مسیر
در مورد قسمت b برای رفتن به کشتارگاه 8 راه و همین مسیر هم برای برگشت صدق میکنه پس کل مسیرهای ممکن 8*8=64 تا هست :46:طبق اصل زیر :

اصل ضرب : اگر انجام کار w وابسته به انجام کار w1 و w2 و ....و wk باشد آنگاه کار w را میتوان به n1.n2....nk طریق میتوان نجام داد .
اصل جمع :اگر انجام کار w وابسته به انجام کار w1 یاw2 یا ....یا wk باشد آنگاه کار w را میتوان به n1+n2....+nk طریق میتوان نجام داد .
کلا کار wi (با اندیس i) با ni طریق میتوان انجام داد.
خب برای پیمودن مسیر رفت و برگشت وابسته به پیمودن 8 طریق رفت و 8 طریق برگشت هست و طبق اصل ضرب کلا 8*8=64
در مورد قسمت c مساله مثل تعداد کلمات دو حرفی است که با استفاده از 8 حرف میتوان نوشت به طوری که حرف دوم تکراری نباشه پس کلا 7*8=56 مسیر ممکن میتوان طی کرد .:46:

اما به نظرم حالی برای مادره نمیمونه که جسد فرزندشو تحویل بگیره و شاید به مسیر برگشت هم نرسه !!! :31:
ببخشید در مورد قسمت C بیشتر توضیح می فرمایید؟ آیا سوال این هست که این خانم به چند طریق میتونه به مقصد بره و بازگرده در حالی که از هیچ کدوم از جاده های رفت هنگام برگشت استفاده نکنه؟

Davy Jones جان شما هم یه نگاهی بندازی ممنون میشم. (همچنین شما Yugio عزیز)

yugioh
18-07-2009, 23:03
ببخشید در مورد قسمت C بیشتر توضیح می فرمایید؟ آیا سوال این هست که این خانم به چند طریق میتونه به مقصد بره و بازگرده در حالی که از هیچ کدوم از جاده های رفت هنگام برگشت استفاده نکنه؟

Davy Jones جان شما هم یه نگاهی بندازی ممنون میشم. (همچنین شما Yugio عزیز)

دیگه گفتی "عزیز" جو منو برد. البته اسمم رو غلط نوشتی که مهم نیست اسپلش سخته.
این بخش C اش بستگی به اینکه آیا اگر بر فرض به ظریق الف بره با ب برگرده، با اینکه از ب بره از الف برگرده جواب یکی دوبرابر اون یکی میشه.
من با اینکه فرق نداره حل می کنم اون یکی اش هم ضربدر دوشه. (c(1,2
اگه منظورش این باشه که از دقیقا همون مسیر قبلی عینا نیاد:
(c(2,8
رفت از هر راهی و برگشت از هر راهی جز اون پی برای رفت 8 و برای برگشت 1-8 راه هست و اگه اینکه الف-ب =ب- الف یه تقسیم به 2 هم میاد میشه 28.
28 یا 56.
اگه منظورشاین باشه که از هسپ یک از جادهای قبل هم نیاد مثلا : اگر از مسیر 1 تجریش- انقلاب میره انقلاب از مسیر3 انقلاب- جنوب میره جنوب: برگشت از از مسیر 1 تجریش- انقلاب میره انقلاب از مسیر2 انقلاب- جنوب میره جنوب هم قبول نیست ( یجتمل همینه منظور)

اگر هر دوتا رفت و برگشت رو بدون انقلاب بره میشه: (c(2,2
چون فقط همین راه برای تکراری نبودن هست.
اگر هر دوتا رفت و برگشت رو از انقلاب بره میشه:6=2/(p(1,1)*p(1,2)*p(1,2)*p(1,3
در دوتایی که راهی نیست و در 3تایی باید دو تا انتخاب بشه. در حالیکه رفت آزاده تقسیم به دو هم برای تمایز اونهاست.
اگر یکی از رفت و برگشت رو بدون انقلاب بره، یکی رو با انقلاب میشه:
(c(1,2)*c(1,2)*c(1,3
عمرا جاده یکسان نمیشه باشه.
میشه: 1+6+12=19
در کل 19 یا 38 فرق رفت و برگشت.
حال کردید با این راه حل مسخره یه جواب دیگه در اومد. اگرم به نظرتون غلطه مهم نیست. نظره دیگه. :21.

amintnt
19-07-2009, 00:33
دیگه گفتی "عزیز" جو منو برد. البته اسمم رو غلط نوشتی که مهم نیست اسپلش سخته.
این بخش C اش بستگی به اینکه آیا اگر بر فرض به ظریق الف بره با ب برگرده، با اینکه از ب بره از الف برگرده جواب یکی دوبرابر اون یکی میشه.
من با اینکه فرق نداره حل می کنم اون یکی اش هم ضربدر دوشه. (c(1,2
اگه منظورش این باشه که از دقیقا همون مسیر قبلی عینا نیاد:
(c(2,8
رفت از هر راهی و برگشت از هر راهی جز اون پی برای رفت 8 و برای برگشت 1-8 راه هست و اگه اینکه الف-ب =ب- الف یه تقسیم به 2 هم میاد میشه 28.
28 یا 56.
اگه منظورشاین باشه که از هسپ یک از جادهای قبل هم نیاد مثلا : اگر از مسیر 1 تجریش- انقلاب میره انقلاب از مسیر3 انقلاب- جنوب میره جنوب: برگشت از از مسیر 1 تجریش- انقلاب میره انقلاب از مسیر2 انقلاب- جنوب میره جنوب هم قبول نیست ( یجتمل همینه منظور)

اگر هر دوتا رفت و برگشت رو بدون انقلاب بره میشه: (c(2,2
چون فقط همین راه برای تکراری نبودن هست.
اگر هر دوتا رفت و برگشت رو از انقلاب بره میشه: (c(2,3)*c(2,2
در دوتایی که راهی نیست و در 3تایی باید دو تا انتخاب بشه.
اگر یکی از رفت و برگشت رو بدون انقلاب بره، یکی رو با انقلاب میشه:
(c(1,2)*c(1,2)*c(1,3
عمرا جاده یکسان نمیشه باشه.
میشه: 1+3+12=16
در کل 16 (محتمل و فرض zahedi) یا 32 فرق رفت و برگشت.
حال کردید با این راه حل مسخره یه جواب دیگه در اومد. راه حل درسته شک نکنید :21.
دست شما درد نکنه....

راستش نحوه ی استدلالت درست مثل مال من بود. فقط تو محاسبه نمیدونم چرا یکی نیستن! خوب اگه از راههای مستقیم بره و برگرده یک راه داره(که من اول گفتم دو راه، چون مسیر رفت رو میشه به دو صورت انتخاب کرد. اما خوب نباید این طوری در نظر بگیریم). اگه هم رفت و هم برگشت رو به طور غیر مستقیم یا به قول شما از انقلاب بره اون وقت به دو طریق از تجریش تا انقلاب، به سه طریق از انقلاب تا مقصد و موقع بازگشت به دو طریق از مقصد به انقلاب و به 1 طریق از انقلاب به مبدا میتونه بره. که میشه 2*3*2=12. و اگه به صورت ترکیبی از انقلاب و مسیر های یکسره(مستقیم) میگیم بدون در نظر گرفتن تفاوت در رفت و برگشت، به دو طریق از مسیر های یکسره میره به مقصد، و به 3 طریق میتونه از مقصد به انقلاب بیاد، و به دو طریق از انقلاب به مبدا که بازم میشه 12. و 12+12+1=25 که همون جواب منه....

اگه اشتباهی میکنم محبت کنید بفرمایید که متوجه بشم...

yugioh
19-07-2009, 05:54
دست شما درد نکنه....

راستش نحوه ی استدلالت درست مثل مال من بود. فقط تو محاسبه نمیدونم چرا یکی نیستن! خوب اگه از راههای مستقیم بره و برگرده یک راه داره(که من اول گفتم دو راه، چون مسیر رفت رو میشه به دو صورت انتخاب کرد. اما خوب نباید این طوری در نظر بگیریم). اگه هم رفت و هم برگشت رو به طور غیر مستقیم یا به قول شما از انقلاب بره اون وقت به دو طریق از تجریش تا انقلاب، به سه طریق از انقلاب تا مقصد و موقع بازگشت به دو طریق از مقصد به انقلاب و به 1 طریق از انقلاب به مبدا میتونه بره. که میشه 2*3*2=12. و اگه به صورت ترکیبی از انقلاب و مسیر های یکسره(مستقیم) میگیم بدون در نظر گرفتن تفاوت در رفت و برگشت، به دو طریق از مسیر های یکسره میره به مقصد، و به 3 طریق میتونه از مقصد به انقلاب بیاد، و به دو طریق از انقلاب به مبدا که بازم میشه 12. و 12+12+1=25 که همون جواب منه....

اگه اشتباهی میکنم محبت کنید بفرمایید که متوجه بشم...

خوب فرق ما تو این بخش دومه از انقلاب بره و بیاد. گفتم اگه فرض شه منظور سوال اینه که این رفت و برگشت عینا از یک مسیر نباشه: 8 تا انتخاب برای رفت و 7 تا برای برگشت هست. اما:21: اگه بگه از یک جاده هم دو بار نره:
اونوقت چی؟ در مسیر تجریش انقلاب دوتا جاده هست پس اگر هر دو بار از این روش انقلابی بیاد، یه بار باید از مسیر 1 و یه با از مسیر دوم بین تجریش انقلاب بیاد. که میشه انتخاب 2 از 2 (c) . بین انقلاب و جنوب هم 3 مسیر داریم: هیچ جاده ای قراره تکرار نشه پس: باید از این 3 جاده در دوتاش رفت آمد کنه مثلا 1و2 یا 1و3 یا 2و3 . که همون انتخاب 2از 3 (c) هست. پس فقط 3 راه هست. اما:21: اصل قضیه اینه که من دقیقا درست هم نگفتم، چرا؟ چون در عضویت جاده ها حالت رو مثل رنگ کزدن جاده گرفتم که کلا خیلی ضایعست. راه حل دقیق اینه که ما بگیم :6= 2/2*3*2
یعنی چی؟ یعنی مسیر رفت6 حالت داریم: مسیر برگشت هم دو حالت (توی یه جاده معلومه اون یکی مساله هست.) تقسیم بر دو هم برای اینکه رفت و برگشت تمایز نکنه.
پس میگم نه حرفتو نه حرف من: 12+6+1=19.
یا دو برابرش:38.

davy jones
19-07-2009, 09:45
ببخشید در مورد قسمت C بیشتر توضیح می فرمایید؟ آیا سوال این هست که این خانم به چند طریق میتونه به مقصد بره و بازگرده در حالی که از هیچ کدوم از جاده های رفت هنگام برگشت استفاده نکنه؟

Davy Jones جان شما هم یه نگاهی بندازی ممنون میشم. (همچنین شما Yugio عزیز)


c - چند تا از مسیرهای پیموده شده قسمت قبل به گونه ای هستند که در مسیر بازگشت از کشتارگاه به تجریش ، از جاده های قبلا پیموده شده استفاده نمی شود؟اگه از مسیرهای مستقیم بره برای برگشت یک مسیر مستقیم و 6 مسیر غیر مستقیم داره.(7 انتخاب) و اگر از از یکی از مسیرهای غیر مستقیم بره ، برای یرگشت 2 مسیر مستقیم میمونه و چون از هر کدوم از مسیرهای غیر مستقیم (2تا تجریش تا انقلاب و 3تا انقلاب تا کشتارگاه) یک مسیر کم میشه، پس 1*2=2 مسیر غیر مستقیم میمونه.(4 انتخاب)
پس جواب کلی میشه 2*7+4*6=38 حالت ممکن.

البته شاید جواب من غلط باشه.

موفق باشین.
88/4/28

davy jones
19-07-2009, 10:08
یه سوال دیگه تو همین مایه های شمارش و آنالیز ترکیبی:

دایره ای رو فرض کنین که رو محیطش n تا نقطه قرار داره. اگه همه ی وترهای این دایره رو که با این n نقطه میشه کشید رو رسم کنیم و فرض کنیم که هیچ سه خطی در یک نقطه همرس نباشن، چند مثلث به وجود میآید؟

موفق باشین.
88/4/28

eh_mn
20-07-2009, 06:46
ميتونيد ثابت كنيد كه اگر a^3 مضرب 3 باشه آنگاه a هم مضرب 3 هست؟ (بدون استفاده از برهان خلف)

chessmathter
20-07-2009, 09:21
ميتونيد ثابت كنيد كه اگر a^3 مضرب 3 باشه آنگاه a هم مضرب 3 هست؟ (بدون استفاده از برهان خلف)
چون a^3 مکعب کاملهپس از هر عامل مضرب 3 تا داره و چون مضرب 3 هست پس عامل 3 به توان 3k داره که وقتی رادیکال به فرجه 3 ازش میگیریم تاa بدست بیاد a عامل 3 به توان k خواهد داشت!.:20:

هایده
20-07-2009, 17:25
tanx

[ برای مشاهده لینک ، لطفا با نام کاربری خود وارد شوید یا ثبت نام کنید ] ([ برای مشاهده لینک ، لطفا با نام کاربری خود وارد شوید یا ثبت نام کنید ])

خورشید زمستان
20-07-2009, 23:13
ميتونيد ثابت كنيد كه اگر a^3 مضرب 3 باشه آنگاه a هم مضرب 3 هست؟ (بدون استفاده از برهان خلف)
لزوما درست نیست مثلا اگه a^3=3 باشد ،( a =3^(1/3 که مضرب 3 نیست :41: شاید من سوالو متوجه نشدم

amintnt
21-07-2009, 01:01
لزوما درست نیست مثلا اگه a^3=3 باشد ،( a =3^(1/3 که مضرب 3 نیست :41: شاید من سوالو متوجه نشدم
احتمالا اعداد طبیعی مد نظر بوده...

eh_mn
21-07-2009, 09:21
لزوما درست نیست مثلا اگه a^3=3 باشد ،( a =3^(1/3 که مضرب 3 نیست :41: شاید من سوالو متوجه نشدم

سلام
وقتي صحبت از مضرب و مقسوم و ... به ميون مياد منظور اعداد طبيعي هستن. يه جاي سوال گفته شده "a هم مضرب 3 است" اين يعني اينكه ما فرض كرديم a يك عدد طبيعيه.

راه حل chessmathter درسته.


چون a^3 مکعب کاملهپس از هر عامل مضرب 3 تا داره و چون مضرب 3 هست پس عامل 3 به توان 3k داره که وقتی رادیکال به فرجه 3 ازش میگیریم تاa بدست بیاد a عامل 3 به توان k خواهد داشت!.

ولي يك راه جالب ديگه هم وجود داره. مي دانيم

[ برای مشاهده لینک ، لطفا با نام کاربری خود وارد شوید یا ثبت نام کنید ](a^3-a)=a(a^2-1)=(a-1)a(a+1)

عدد 3 عبارت سمت راست را عاد مي كند زيرا يكي از سه عدد متوالي بر 3 بخش پذير است. بنابراين سمت چپ هم بر 3 بخش پذير است. چون a^3 بر 3 بخش پذير است پس a هم، هم!

حالا با همين روش ميتونين ثابت كنين كه اگر a^2 زوح باشه a هم زوجه؟

davy jones
21-07-2009, 11:02
یه سوال دیگه تو همین مایه های شمارش و آنالیز ترکیبی:

دایره ای رو فرض کنین که رو محیطش n تا نقطه قرار داره. اگه همه ی وترهای این دایره رو که با این n نقطه میشه کشید رو رسم کنیم و فرض کنیم که هیچ سه خطی در یک نقطه همرس نباشن، چند مثلث به وجود میآید؟

موفق باشین.
88/4/28

اینم جوابش:

تعداد کل خطهایی که میشه کشید که دو سرش از این نقاط استفاده شده باشه برابر با انتخاب 2 از n هستش. حال چون هر 3 تا خطی که از این خطوط انتخاب کنیم، همرس نیستند پس حتما تشکیل یک مثلث خواهند داد. (البته با عرض پوزش اینو یادم رفت بگم که هیچ 2 وتری هم با هم موازی نیستند.:41:) پس جواب کلی میشه:

[ برای مشاهده لینک ، لطفا با نام کاربری خود وارد شوید یا ثبت نام کنید ]

موفق باشین.
88/4/30

davy jones
21-07-2009, 11:06
یه سوال معروف در آنالیز ترکیبی:
اگر N تا جعبه یکسان داشته باشیم و M تا توپ یکسان (M>N) به چند طریق میتوان همه ی توپها رو درون جعبه ها انداخت طوری که :
1- در هر جعبه حداقل 1 توپ قرار گیرد.
2- میتواند بعضی از جعبه ها هم خالی باشد. (یعنی شرط خاصی وجود ندارد.)

موفق باشین.
88/4/30

mahsa1469
21-07-2009, 12:36
سلام
یه سوال داشتم در مورد ریاضی سوم راهنمایی!!!!
8 تا کارگر کاری را 6 روزه به اتمام می رسونن پس از آنکه نیمی از کا ر انجام شد 2 نفر از کارگران اخراج می شوند کل کار چند روزه به اتمام می رسه؟؟؟

davy jones
21-07-2009, 12:44
سلام
یه سوال داشتم در مورد ریاضی سوم راهنمایی!!!!
8 تا کارگر کاری را 6 روزه به اتمام می رسونن پس از آنکه نیمی از کا ر انجام شد 2 نفر از کارگران اخراج می شوند کل کار چند روزه به اتمام می رسه؟؟؟

اینکه 8 تا کارگر در 6 روز کار رو تموم میکنن یعنی کلا اون پروژه 48 نفر_روز کار میبره. حال اگه نصف این پروژه انجام شده باشه پس یعنی 3 روز گذشته و به 24 نفر_روز کار دیگه احتیاج داریم که با وجود 6 نفر کار در 4 روز به اتمام میرسه. پس کل کار در 7 روز تموم میشه.

موفق باشین.
88/4/30

sherlockholmz
21-07-2009, 13:10
سلام
یه سوال داشتم در مورد ریاضی سوم راهنمایی!!!!
8 تا کارگر کاری را 6 روزه به اتمام می رسونن پس از آنکه نیمی از کا ر انجام شد 2 نفر از کارگران اخراج می شوند کل کار چند روزه به اتمام می رسه؟؟؟
با سلام

اول اين سوال را جواب مي دهيم.
8 تا کارگر کاری را 6 روزه به اتمام می رسونن 6 كارگر همان كاررا چند روزه تمام مي كنند؟
اين يك تناسب معكوس است كه جوابش ميشود:

x=6*8/6=8

پس 6 كارگر كل كار را در 8 روز تمام مي كنند.
حال كار دو قسمت مي شود نصف آنرا 8 كارگر انجام مي دهند(كه 3 روز بطول مي انجامد) و نصف ديگررا 6 كارگر(كه 4 روز بطول مي انجامد) پس كل كار در 7 روز بپايان مي رسد.
موفق باشيد.

jims band
21-07-2009, 13:36
سلام.این سؤالو یکی از دوستانم به من داده خیلی ساده هست من خودمم حلش کردم (البته یه هفته طول کشید.) دیدم سؤال خوبیه گفتم اینجا بذارم :
باید بین اعداد از اعمال ریاضی استفاده کنید مثلا" باید بگید سه تا 2 چطوری شش میشه :


6 = 1 1 1
6 = 2 2 2
6 = 3 3 3
6 = 4 4 4
6 = 5 5 5
6 = 6 6 6
6 = 7 7 7
6 = 8 8 8
6 = 9 9 9

davy jones
21-07-2009, 13:44
سلام.این سؤالو یکی از دوستانم به من داده خیلی ساده هست من خودمم حلش کردم (البته یه هفته طول کشید.) دیدم سؤال خوبیه گفتم اینجا بذارم :
باید بین اعداد از اعمال ریاضی استفاده کنید مثلا" باید بگید سه تا 2 چطوری شش میشه :


6 = 1 1 1
6 = 2 2 2
6 = 3 3 3
6 = 4 4 4
6 = 5 5 5
6 = 6 6 6
6 = 7 7 7
6 = 8 8 8
6 = 9 9 9



منظورت از اعمال ریاضی فقط 4 عمل اصلیه یا از هر چیزی میشه استفاده کرد؟

موفق باشین.
88/4/30

davy jones
21-07-2009, 14:20
با سلام

اول اين سوال را جواب مي دهيم.
8 تا کارگر کاری را 6 روزه به اتمام می رسونن 6 كارگر همان كاررا چند روزه تمام مي كنند؟
اين يك تناسب معكوس است كه جوابش ميشود:

x=6*8/6=8

پس 6 كارگر كل كار را در 8 روز تمام مي كنند.
حال كار دو قسمت مي شود نصف آنرا 8 كارگر انجام مي دهند(كه 3 روز بطول مي انجامد) و نصف ديگررا 6 كارگر(كه 4 روز بطول مي انجامد) پس كل كار در 7 روز بپايان مي رسد.
موفق باشيد.






من هم که همینو گفتم. این استدلال شما چه چیز جدیدی داشت؟

موفق باشین.
88/4/30

mahsa1469
21-07-2009, 15:55
سلام.این سؤالو یکی از دوستانم به من داده خیلی ساده هست من خودمم حلش کردم (البته یه هفته طول کشید.) دیدم سؤال خوبیه گفتم اینجا بذارم :
باید بین اعداد از اعمال ریاضی استفاده کنید مثلا" باید بگید سه تا 2 چطوری شش میشه :


6 = 1 1 1
6 = 2 2 2
6 = 3 3 3
6 = 4 4 4
6 = 5 5 5
6 = 6 6 6
6 = 7 7 7
6 = 8 8 8
6 = 9 9 9

من چند تا شو پیدا کردم یعنی تا نگاه کردم فهمیدم

amintnt
21-07-2009, 18:33
اینم جوابش:

تعداد کل خطهایی که میشه کشید که دو سرش از این نقاط استفاده شده باشه برابر با انتخاب 2 از n هستش. حال چون هر 3 تا خطی که از این خطوط انتخاب کنیم، همرس نیستند پس حتما تشکیل یک مثلث خواهند داد. (البته با عرض پوزش اینو یادم رفت بگم که هیچ 2 وتری هم با هم موازی نیستند.:41:) پس جواب کلی میشه:

[ برای مشاهده لینک ، لطفا با نام کاربری خود وارد شوید یا ثبت نام کنید ]

موفق باشین.
88/4/30

اگه همه ی وترهای این دایره رو که با این n نقطه میشه کشید رو رسم کنیم ... ... چند مثلث به وجود میآید؟سوال کژتابی داره... مثلا سه تا وتر رسم کنید که یه مثلت درون دایره درست کنه(رئوس روی دایره قرار نگیره!)... من فکر کردم اینطوری باشه :دی هرچند کمی ساده لوحانه بود :happy: اما به هر حال از صورت سوال بر می اومد ; نه؟
[ برای مشاهده لینک ، لطفا با نام کاربری خود وارد شوید یا ثبت نام کنید ]

mahsa1469
21-07-2009, 18:34
اینکه 8 تا کارگر در 6 روز کار رو تموم میکنن یعنی کلا اون پروژه 48 نفر_روز کار میبره. حال اگه نصف این پروژه انجام شده باشه پس یعنی 3 روز گذشته و به 24 نفر_روز کار دیگه احتیاج داریم که با وجود 6 نفر کار در 4 روز به اتمام میرسه. پس کل کار در 7 روز تموم میشه.

موفق باشین.
88/4/30


با سلام

اول اين سوال را جواب مي دهيم.
8 تا کارگر کاری را 6 روزه به اتمام می رسونن 6 كارگر همان كاررا چند روزه تمام مي كنند؟
اين يك تناسب معكوس است كه جوابش ميشود:


x=6*8/6=8



پس 6 كارگر كل كار را در 8 روز تمام مي كنند.



حال كار دو قسمت مي شود نصف آنرا 8 كارگر انجام مي دهند(كه 3 روز بطول مي انجامد) و نصف ديگررا 6 كارگر(كه 4 روز بطول مي انجامد) پس كل كار در 7 روز بپايان مي رسد.


موفق باشيد.







از هردوتون بابت پاسخ ممنونم
می تونم ازتون خواهش کنم چند سوال در این مورد برای من بدین :11:

eh_mn
21-07-2009, 20:10
كسي اين رو حل كرد؟


[ برای مشاهده لینک ، لطفا با نام کاربری خود وارد شوید یا ثبت نام کنید ]^\infty\frac{x^3}{e^x-1}dx=\frac{\pi^4}{15}

mahsa1469
21-07-2009, 21:39
سلام مجدد:D
بچه ها دیگه سوال نمی خوام می شه این سوال ها رو به روشی آسون حلشون کنید در حدی که یه بچه ی سوم راهنمایی بتونه اونا رو حل کنه نه در حد گسسته ی پیش (در اون حد می دونم)

باقیمانده ی عدد 5 به توان 1386 تقسیم بر1000 ؟
کدام عدد از همه بزرگ تر است؟(با دلیل)
2 به توان 2 به توان 2 به توان دو(توان ها برا توان هستش)
22 به توان 22
222 به توان 2



خواهش می کنم اگه جواب رو بلدین زود جواب بدین

davy jones
22-07-2009, 08:14
سوال کژتابی داره... مثلا سه تا وتر رسم کنید که یه مثلت درون دایره درست کنه(رئوس روی دایره قرار نگیره!)... من فکر کردم اینطوری باشه :دی هرچند کمی ساده لوحانه بود :happy: اما به هر حال از صورت سوال بر می اومد ; نه؟
[ برای مشاهده لینک ، لطفا با نام کاربری خود وارد شوید یا ثبت نام کنید ]

موافقم.

موفق باشين.
88/4/31

davy jones
22-07-2009, 08:32
سلام مجدد:D
بچه ها دیگه سوال نمی خوام می شه این سوال ها رو به روشی آسون حلشون کنید در حدی که یه بچه ی سوم راهنمایی بتونه اونا رو حل کنه نه در حد گسسته ی پیش (در اون حد می دونم)

باقیمانده ی عدد 5 به توان 1386 تقسیم بر1000 ؟
کدام عدد از همه بزرگ تر است؟(با دلیل)
2 به توان 2 به توان 2 به توان دو(توان ها برا توان هستش)
22 به توان 22
222 به توان 2



خواهش می کنم اگه جواب رو بلدین زود جواب بدین

باقيمانده 5 به توان 1 ميشه 005 (در تقسيم به 1000)
باقيمانده 5 به توان 2 ميشه 025
باقيمانده 5 به توان 3 ميشه 125
باقيمانده 5 به توان 4 ميشه 625
باقيمانده 5 به توان 5 ميشه 125
باقيمانده 5 به توان 6 ميشه 625
.
.
.
از اينجا به بعد ديگه باقيمانده ها فقط بين دو عدد 125 و 625 تغيير ميكنه. باقيمانده توانهايي كه زوجه (مث 4 و 6 و ...) 625 هستش و توانهاي فرد 125.
چون 1386 زوجه پس باقيمانده ي تقسيم 5 به توان 1386 برابر با 625 ميشه.


در مورد سوال بعدي هم بايد بگم كه "2 به توان 2 به توان 2 به توان 2 " ميشه 2 به توان 16 كه چيزي نزديك به 80000 ميشه. 222 به توان 2 هم چيزي در اوردر 50000 ميشه. ولي در مورد 22 به توان 22 ، با يك نگاه ساده ميشه فهميد اگه پايه ي توان رو به جاي 22 و براي راحتي بيشتر 10 در نظر بگيريم جواب ميشه يه دونه 1 با 22 تا صفر جلوش! تازه اين از جواب اصلي به مراتب كوچيك تره! پس واضحه كه 22 به توان 22 از همه بزرگتره.


موفق باشين.
88/4/31

davy jones
22-07-2009, 08:36
یه سوال معروف در آنالیز ترکیبی:
اگر N تا جعبه یکسان داشته باشیم و M تا توپ یکسان (M>N) به چند طریق میتوان همه ی توپها رو درون جعبه ها انداخت طوری که :
1- در هر جعبه حداقل 1 توپ قرار گیرد.
2- میتواند بعضی از جعبه ها هم خالی باشد. (یعنی شرط خاصی وجود ندارد.)

موفق باشین.
88/4/30

نبووووود؟؟:19:

باشه حالا كه اينقدر اصرار ميكنين بازم صبر ميكنم و جوابش رو حالا نميذارم. روش فكر كنين ديگه:27:

موفق باشين.
88/4/31

sherlockholmz
22-07-2009, 08:37
سلام مجدد:D
بچه ها دیگه سوال نمی خوام می شه این سوال ها رو به روشی آسون حلشون کنید در حدی که یه بچه ی سوم راهنمایی بتونه اونا رو حل کنه نه در حد گسسته ی پیش (در اون حد می دونم)

باقیمانده ی عدد 5 به توان 1386 تقسیم بر1000 ؟



خواهش می کنم اگه جواب رو بلدین زود جواب بدین\

باسلام
روش اصلي حل اينگونه سوالات، استفاده از همنهشتي است.البته ما همنهشتي را در دبيرستان خوانديم. پس:




[ برای مشاهده لینک ، لطفا با نام کاربری خود وارد شوید یا ثبت نام کنید ] Cfrac%7B5%5E%7B1386%7D%7D%7B5%5E%7B3%7D2%5E%7B3%7D %7D=%5Cfrac%7B5%5E%7B1383%7D%7D%7B2%5E%7B3%7D%7D=% 5Cfrac%7B5%5E%7B2%7D5%5E%7B1381%7D%7D%7B2%5E%7B3%7 D%7D=5%5E%7B1381%7D%5Cfrac%7B25%7D%7B8%7D


پس باقيمانده 5 بتوان 1386 بر 1000 با باقيمانده 25 بر 8 برابر است.پس باقيمانده مطلوب 1 است.
موفق باشيد.

davy jones
22-07-2009, 09:00
كسي اين رو حل كرد؟


[ برای مشاهده لینک ، لطفا با نام کاربری خود وارد شوید یا ثبت نام کنید ] e%5Ex-1%7Ddx=%5Cfrac%7B%5Cpi%5E4%7D%7B15%7D


اين انتگرال رو بايد تو اعداد مختلط گرفت. يادش بخير تو دانشگاه يه چيزايي راجع بهش ميگفت اون استاد خدابيامرزمون. فكر نكنم با روشهاي معمول حل بشه. خودت جوابش رو ميدوني؟

موفق باشين.
88/4/31

jims band
22-07-2009, 09:13
من چند تا شو پیدا کردم یعنی تا نگاه کردم فهمیدم





نه میتونی از هر عملی استفاده کنی . بعضی هاشون خیلی سادن و یکیشون خیلی وقتگیره .

eh_mn
22-07-2009, 09:16
اين انتگرال رو بايد تو اعداد مختلط گرفت. يادش بخير تو دانشگاه يه چيزايي راجع بهش ميگفت اون استاد خدابيامرزمون. فكر نكنم با روشهاي معمول حل بشه. خودت جوابش رو ميدوني؟

موفق باشين.
88/4/31

نه نمي‌دونم.

mahsa1469
22-07-2009, 13:48
\

باسلام
روش اصلي حل اينگونه سوالات، استفاده از همنهشتي است.البته ما همنهشتي را در دبيرستان خوانديم. پس:




[ برای مشاهده لینک ، لطفا با نام کاربری خود وارد شوید یا ثبت نام کنید ] Cfrac%7B5%5E%7B1386%7D%7D%7B5%5E%7B3%7D2%5E%7B3%7D %7D=%5Cfrac%7B5%5E%7B1383%7D%7D%7B2%5E%7B3%7D%7D=% 5Cfrac%7B5%5E%7B2%7D5%5E%7B1381%7D%7D%7B2%5E%7B3%7 D%7D=5%5E%7B1381%7D%5Cfrac%7B25%7D%7B8%7D



پس باقيمانده 5 بتوان 1386 بر 1000 با باقيمانده 25 بر 8 برابر است.پس باقيمانده مطلوب 1 است.



موفق باشيد.



مرسی از پاسختون ولی جواب می شه625

davy jones
22-07-2009, 14:00
جواب اين انتگرال چي ميشه: (البته فكر كنم ساده باشه)


[ برای مشاهده لینک ، لطفا با نام کاربری خود وارد شوید یا ثبت نام کنید ]

موفق باشين.
88/4/31

davy jones
22-07-2009, 14:02
همچنين جواب اين رو هم بگين:


[ برای مشاهده لینک ، لطفا با نام کاربری خود وارد شوید یا ثبت نام کنید ] 5Csqrt%7B...%7D%7D%7D%7D

موفق باشين.
88/4/31

mahsa1469
22-07-2009, 14:42
همچنين جواب اين رو هم بگين:


[ برای مشاهده لینک ، لطفا با نام کاربری خود وارد شوید یا ثبت نام کنید ] 5Csqrt%7B...%7D%7D%7D%7D

موفق باشين.
88/4/31
کل عبارت رو مساوی x قرار می دیم در نتیجه طxبرابر می شه رادیکالx+1 با به توان 2 رسیدن دو طرف یه معادله درجه دو به دست می یاد که راحت با یه دلتا به دست می یاد جوابش رو بصورن ریاضی یراتون می نویسم

amintnt
22-07-2009, 15:52
یه سوال معروف در آنالیز ترکیبی:
اگر N تا جعبه یکسان داشته باشیم و M تا توپ یکسان (M>N) به چند طریق میتوان همه ی توپها رو درون جعبه ها انداخت طوری که :
1- در هر جعبه حداقل 1 توپ قرار گیرد.
2- میتواند بعضی از جعبه ها هم خالی باشد. (یعنی شرط خاصی وجود ندارد.)

موفق باشین.
88/4/30
1. اول به هر کدوم یه توپ میدیم، بعد واسه باقیمونده ی توپا از قضیه ی آماره ی انیشتین استفاده میکنیم! (جواب معادله ی معروف N مجهوله، به شرطی که دامنه جز اعداد طبیعی باشه)
2. استفاده از قضیه ی یاد شده، بدون شرط(معادله ی N مجهوله، با جواب صحیح بزرگتر مساوی صفر)

اشتباه که نکردم؟:happy:

هایده
22-07-2009, 22:56
كسي اين رو حل كرد؟



[ برای مشاهده لینک ، لطفا با نام کاربری خود وارد شوید یا ثبت نام کنید ]^\infty\frac{x^3}{e^x-1}dx=\frac{\pi^4}{15}



بچه ها من شنبه حل تمرین دارم
خواهشا کمکم کنید و جواب این سوال رو بدین

eh_mn
23-07-2009, 06:44
اين انتگرال رو بايد تو اعداد مختلط گرفت. يادش بخير تو دانشگاه يه چيزايي راجع بهش ميگفت اون استاد خدابيامرزمون. فكر نكنم با روشهاي معمول حل بشه. خودت جوابش رو ميدوني؟

موفق باشين.
88/4/31

با سلام

خوشبختانه حل شد. البته به نظر ميرسه راه حل ساده‏تر هم داشته باشه!

اول رابطه‏‍‌هايي رو كه لازم داريم مي‏نويسم

(1)

[ برای مشاهده لینک ، لطفا با نام کاربری خود وارد شوید یا ثبت نام کنید ]{n&space;=&space;1}^\infty&space;{\frac{1}{{n^ 4&space;}}}&space;=&space;\frac{{\pi&space;^4&space;}}{{90}}

اين تساوي با استفاده از اتحاد پارسوال با فرض [ برای مشاهده لینک ، لطفا با نام کاربری خود وارد شوید یا ثبت نام کنید ](x)&space;=&space;x(\pi&space;-&space;x) براي [ برای مشاهده لینک ، لطفا با نام کاربری خود وارد شوید یا ثبت نام کنید ][0,\pi]

بدست مياد.

(2)

[ برای مشاهده لینک ، لطفا با نام کاربری خود وارد شوید یا ثبت نام کنید ]^\infty&space;{x^k&space;e^{&space;-&space;ax}&space;dx}&space;=&space;\frac{{k!}}{{a^{k&space;+&space;1}&space;}}

اين رابطه با استفاده از انتگرال‏گيري جزء به جزء ثابت مي‏شه.

مي‏دانيم

[ برای مشاهده لینک ، لطفا با نام کاربری خود وارد شوید یا ثبت نام کنید ]{1}{{1&space;-&space;x}}&space;=&space;\sum\limits_{n&space;=&space;0}^\infty&space;{x^n&space;}&space;,\left|&space;x &space;\right|&space;<&space;1.
بنابراين


[ برای مشاهده لینک ، لطفا با نام کاربری خود وارد شوید یا ثبت نام کنید ]{1}{{1&space;-&space;\frac{1}{e^x}}}&space;=&space;\sum\limits_{n&space;=&space;0}^\infty &space;{e^{-nx}&space;}&space;,&space;x>0.

پس


[ برای مشاهده لینک ، لطفا با نام کاربری خود وارد شوید یا ثبت نام کنید ]{{x^3&space;}}{{e^x&space;-&space;1}}&space;=&space;\frac{{x^3&space;}}{{e^x&space;}}\left( \frac{1}{{1&space;-&space;\frac{1}{{e^x&space;}}}}\right)&space;=&space;\frac{{x^3&space;}}{{e^x&space;}} \sum\limits_{n&space;=&space;0}^\infty&space;{e^{&space;-&space;nx}&space;}&space;=&space;\sum\limits_{n&space;=&space;0}^\infty&space;{x^3&space;e^{&space;-&space;(n&space;&plus;&space;1)x}&space;}

حالا ميريم سراغ اصل مطلب


[ برای مشاهده لینک ، لطفا با نام کاربری خود وارد شوید یا ثبت نام کنید ]{align}\int_0^\infty%20{\frac{{x ^3}}{{e^x-1}}dx}%20&%20=%20&%20\int_0^\infty%20{\sum\limits_{n%20=0}^\infty{x^ 3%20e^{-(n+1)x}}dx}%20\\%20&%20=%20&%20\sum\limits_{n=0}^\infty{\int_0^\infty{x^3e^{-(n+1)x}dx}}%20\\%20&%20=%20&\sum\limits_{n=0}^\infty{\frac{{3!}}{{(n+1)^4}}}%2 0\\%20&%20=%20&\frac{{\pi^4}}{15}%20\end{align}


توجه كنيد كه با استفاده از قضيه‏اي مي‏توان نشان داد كه تعويض جاي انتگرال در رابطه‏ي اخير مجاز است.

jims band
24-07-2009, 18:09
سلام.این سؤالو یکی از دوستانم به من داده خیلی ساده هست من خودمم حلش کردم (البته یه هفته طول کشید.) دیدم سؤال خوبیه گفتم اینجا بذارم :
باید بین اعداد از اعمال ریاضی استفاده کنید مثلا" باید بگید سه تا 2 چطوری شش میشه :


6 = 1 1 1
6 = 2 2 2
6 = 3 3 3
6 = 4 4 4
6 = 5 5 5
6 = 6 6 6
6 = 7 7 7
6 = 8 8 8
6 = 9 9 9



کسی جواب اینو پیدا نکرد ؟ :41: :21: :18::19:

davy jones
25-07-2009, 14:02
کسی جواب اینو پیدا نکرد ؟ :41: :21: :18::19:

یه چند تاشو پیدا کردم:

[ برای مشاهده لینک ، لطفا با نام کاربری خود وارد شوید یا ثبت نام کنید ]*3-3=6%5C%5C&space;%5Cleft&space;%5Clceil&space;%5Csqrt%5B4%5D%7B4%7D*4 &space;%5Cright&space;%5Crceil=6%5C%5C&space;%5Cleft&space;%5Clfloor&space;%5Csq rt%5B5%5D%7B5%7D*5&space;%5Cright&space;%5Crfloor=6%5C%5C&space;6&plus;6-6=6%5C%5C&space;%5Cleft&space;%5Clfloor&space;ln%287%5E7%29-7&space;%5Cright&space;%5Crfloor=6%5C%5C

موفق باشین.
88/5/3

davy jones
25-07-2009, 14:04
یه چند تاشو پیدا کردم:

[ برای مشاهده لینک ، لطفا با نام کاربری خود وارد شوید یا ثبت نام کنید ]

[ برای مشاهده لینک ، لطفا با نام کاربری خود وارد شوید یا ثبت نام کنید ]*3-3=6

[ برای مشاهده لینک ، لطفا با نام کاربری خود وارد شوید یا ثبت نام کنید ]*4 &space;%5Cright&space;%5Crceil=6

[ برای مشاهده لینک ، لطفا با نام کاربری خود وارد شوید یا ثبت نام کنید ]* 5&space;%5Cright&space;%5Crfloor=6

[ برای مشاهده لینک ، لطفا با نام کاربری خود وارد شوید یا ثبت نام کنید ]

[ برای مشاهده لینک ، لطفا با نام کاربری خود وارد شوید یا ثبت نام کنید ]

[ برای مشاهده لینک ، لطفا با نام کاربری خود وارد شوید یا ثبت نام کنید ]*9%7D-%5Csqrt%7B9%7D=6

موفق باشین.
88/5/3

mahsa1469
25-07-2009, 14:16
davy jones
چی نوشتی؟؟؟
2+2+2=6
6-6+6=6
5/5+5=6
7/7-7=6
3*3-3=6
یه کم قاطی شد!

davy jones
25-07-2009, 14:45
davy jones
چی نوشتی؟؟؟
2+2+2=6
6-6+6=6
5/5+5=6
7/7-7=6
3*3-3=6
یه کم قاطی شد!

نمیدونم چرا اینطوری میشه. طبق دستورالعملی که مدیران محترم انجمن ریاضیات گفته بودن از کد لاتکس استفاده کردم. هر کاری میکنم درست نمیشه.:41:

موفق باشین.
88/5/3

yugioh
25-07-2009, 16:08
سلام.این سؤالو یکی از دوستانم به من داده خیلی ساده هست من خودمم حلش کردم (البته یه هفته طول کشید.) دیدم سؤال خوبیه گفتم اینجا بذارم :
باید بین اعداد از اعمال ریاضی استفاده کنید مثلا" باید بگید سه تا 2 چطوری شش میشه :


6 = 1 1 1
6 = 2 2 2
6 = 3 3 3
6 = 4 4 4
6 = 5 5 5
6 = 6 6 6
6 = 7 7 7
6 = 8 8 8
6 = 9 9 9


اصولا این سوالات جالبتره.
احتمالا غلطه چون رادیکال داره.

!(1+1+1)
!(2+2/2)
!(3+3-3)
!(4-4/4)
5/5+5
6+6-6
7-7/7
!((ln(8) +ln (8^1/2) )/ ln (8^1/2))
(1/2^9)/(9+9)

چون غلط چاپ می شه بگم : واضحه که قراره داخل ! مثبت باشه.
8: لگاریتم 8+ لگاریتم رادیکال 8 تقسیم بر لگاریتم رادیکال 8 هست. همش فاکتوریل
9: 9+9 تقسیم بر رادیکال 9. همش فاکتوریل
به نظرم باید غلط حسابشه چون رادیکال هست که هرچند فرجه اش نیست ولی فرجه اش 2 هست. از طرفی خوب 2 هم مساوی 2/1 هست حالا اگه از این ور بگیم یا نباید به رادیکال گیر داد یا جواب نداره.

davy jones
25-07-2009, 16:09
جواب اين انتگرال چي ميشه: (البته فكر كنم ساده باشه)


[ برای مشاهده لینک ، لطفا با نام کاربری خود وارد شوید یا ثبت نام کنید ]

موفق باشين.
88/4/31

اینم جواب آخرش (حوصله ندارم تایپ کنم:5:):

[ برای مشاهده لینک ، لطفا با نام کاربری خود وارد شوید یا ثبت نام کنید ]

موفق باشین.
88/5/3

davy jones
25-07-2009, 16:34
احتمالا غلطه چون رادیکال داره.

!(1+1+1)
!(2+2/2)
!(3+3-3)
!(4-4/4)
5/5+5
6+6-6
7-7/7
!((ln(8) +ln (8^1/2) )/ ln (8^1/2))
(1/2^9)/(9+9)

چون غلط چاپ می شه بگم : واضحه که قراره داخل ! مثبت باشه.
8: لگاریتم 8+ لگاریتم رادیکال 8 تقسیم بر لگاریتم رادیکال 8 هست. همش فاکتوریل
9: 9+9 تقسیم بر رادیکال 9. همش فاکتوریل
به نظرم باید غلط حسابشه چون رادیکال هست که هرچند فرجه اش نیست ولی فرجه اش 2 هست. از طرفی خوب 2 هم مساوی 2/1 هست حالا اگه از این ور بگیم یا نباید به رادیکال گیر داد یا جواب نداره.

شما خیلی علاقه به استفاده از فاکتوریل دارین. مثلا در مورد 2 و 3 راههای بسیار ساده تری هم وجود داره:
2+2+2=6
3*3-3=6

موفق باشین.
88/5/3

sherlockholmz
25-07-2009, 16:47
جواب اين انتگرال چي ميشه: (البته فكر كنم ساده باشه)


[ برای مشاهده لینک ، لطفا با نام کاربری خود وارد شوید یا ثبت نام کنید ]

موفق باشين.
88/4/31


[ برای مشاهده لینک ، لطفا با نام کاربری خود وارد شوید یا ثبت نام کنید ] 4-ln%28x%29%7D%7D=?%20u=4-ln%28x%29%5CRightarrow%20du=-%5Cfrac%7Bdx%7D%7Bx%7D%20%5CRightarrow%20I=-%5Cint%5Cfrac%7Bdu%7D%7B%5Csqrt%7Bu%7D%7D%20%5CRig htarrow%20I=-2%5Cint%5Cfrac%7Bdu%7D%7B2%5Csqrt%7Bu%7D%7D%20=-2%5Csqrt%7Bu%7D%5CRightarrow%20I=-2%5Csqrt%7B4-ln%28x%29%7D

موفق باشيد.

sarsam
07-08-2009, 16:05
با سلام.

لطفا اساتید به سوال منی که میخواهم ریاضی را درک کنم نه صرفا سوال ها را جواب بدم کمک کنید.(اگه میشه با زبانی ساده چون مبتدی ام)

ما قبول داریم که مشتق یک تابع در نقطه ای مانند c برابر است با شیب خط مماس بر آن نقطه و برابر f'(c). حال سوال من این است که مشتق مرتبه دوم آن تابع در آن نقطه را به صورت نموداری چطور نمایش میدهیم یعنی مشتق مرتبه دوم آن تابع در آن نقطه اصولا بر کجا مماس است ؟؟

amintnt
07-08-2009, 17:49
با سلام.

لطفا اساتید به سوال منی که میخواهم ریاضی را درک کنم نه صرفا سوال ها را جواب بدم کمک کنید.(اگه میشه با زبانی ساده چون مبتدی ام)

ما قبول داریم که مشتق یک تابع در نقطه ای مانند c برابر است با شیب خط مماس بر آن نقطه و برابر f'(c). حال سوال من این است که مشتق مرتبه دوم آن تابع در آن نقطه را به صورت نموداری چطور نمایش میدهیم یعنی مشتق مرتبه دوم آن تابع در آن نقطه اصولا بر کجا مماس است ؟؟
تعبیر هندسی مشتق دوم یک نمودار، تقعر اون تابع هست...

Parnyan
08-08-2009, 21:43
در یک ذوزنقه قائم الزاویه اگر اقطار آن بر هم عمود باشند، کدام گزینه همواره درست است (میدونم جواب گزینه 1 هست.)
1- ارتفاع آن واسطه هندسی بین دو قاعده است.
2- ارتفاع آن واسطه هندسی بین دو قطر است.
3-ارتفاع آن واسطه هندسی بین دو ساق است.
4-ارتفاع آن واسطه هندسی بین یک ساق و یک قاعده است.


سلام

میشه یکی این سوالو برام گزینه 1شو اثبات کنه ! ؛ با تشابه حل میشه ولی نمیدونم چرا دو تا مثلی که روی ساقه ذوذنقه درست میشه باهم متشابه اند !

chessmathter
08-08-2009, 23:18
در یک ذوزنقه قائم الزاویه اگر اقطار آن بر هم عمود باشند، کدام گزینه همواره درست است (میدونم جواب گزینه 1 هست.)
1- ارتفاع آن واسطه هندسی بین دو قاعده است.
2- ارتفاع آن واسطه هندسی بین دو قطر است.
3-ارتفاع آن واسطه هندسی بین دو ساق است.
4-ارتفاع آن واسطه هندسی بین یک ساق و یک قاعده است.


سلام

میشه یکی این سوالو برام گزینه 1شو اثبات کنه ! ؛ با تشابه حل میشه ولی نمیدونم چرا دو تا مثلی که روی ساقه ذوذنقه درست میشه باهم متشابه اند !


ذوزنقه abcd رو در نظر تشابه این دو مثلث abc و acd به جواب میرسوندت

Parnyan
10-08-2009, 22:37
ذوزنقه abcd رو در نظر تشابه این دو مثلث abc و acd به جواب میرسوندت

بله

زحمت کشیدین ممنون ...ولی من که مییییدووووووووووووووووووو وونم با تشابه باید حل کنم فقط نمیدونم چرااااا ! چه طوری زاویه ها شو برابر در بیارم ! میدونم رو مثلث هایی که رو ساق ذذنقه تشکیل میشه هست و یکی از دو زاویه برا تشابه هم زاویه قاپمه اس !
:41::41::41::41::41::41:

حلللللل کنییییییییییییییییییییییی ین

amintnt
11-08-2009, 12:12
بله

زحمت کشیدین ممنون ...ولی من که مییییدووووووووووووووووووو وونم با تشابه باید حل کنم فقط نمیدونم چرااااا ! چه طوری زاویه ها شو برابر در بیارم ! میدونم رو مثلث هایی که رو ساق ذذنقه تشکیل میشه هست و یکی از دو زاویه برا تشابه هم زاویه قاپمه اس !
:41::41::41::41::41::41:

حلللللل کنییییییییییییییییییییییی ین
سلام...

اول دو قطر عمود بر هم رسم میکنیم... حالا دو قاعده ذوزنقه رو با دو خط موازی در دو طرف نقطه تقاطع قطر ها رسم میکنیم... اگه فاصله ی نقطه ی تقاطع از یکی از خطوط موازی برابر نباشه با فاصله ی نقطه ی تقاطع با خط دیگه، شکل زیر به وجود میاد...
[ برای مشاهده لینک ، لطفا با نام کاربری خود وارد شوید یا ثبت نام کنید ]

که مشخصه نمیتونه ذوزنقه ی قائم الزاویه درست کنه... برای اینکه قائمه بشه...فاصله ی خطوط موازی رو از نقطه ی تقاطع مساوی در نظر میگیریم.... که در این صورت نه فقط یک ساق، بلکه هر دو ساق قائمه میشن... و اندازه ی ساق ها با اندازه ی قاعده ها هم برابره... به خاطر عمود بودن دو قطر... یعنی هرچه فاصله ی دو خط موازی بیشتر بشه، طولشون بیشتر میشه و طول ساق ها هم بیشتر میشه...
[ برای مشاهده لینک ، لطفا با نام کاربری خود وارد شوید یا ثبت نام کنید ]

و در واقع شکل مربع میشه... که همون رابطه هم بر قراره واسش...

این استدلال شاید دقیق نباشه اما به صورت شهودی چندان بد نیست!

mohsen1472
11-08-2009, 23:53
با سلام
لطفا حد را محاسبه نمایید.

[ برای مشاهده لینک ، لطفا با نام کاربری خود وارد شوید یا ثبت نام کنید ]{x\to&space;0}\frac{1&space;-&space;\cos&space;x\sqrt{\cos&space;2x}}{x^{2}}

mohsen1472
12-08-2009, 17:35
لطفا حدهای زیر را بدون استفاده از هوپیتال و هم ارزی حل کنید:


[ برای مشاهده لینک ، لطفا با نام کاربری خود وارد شوید یا ثبت نام کنید ]{x&space;\to&space;0}\frac{(1&space;&plus;&space;mx)^{n}-&space;(1&space;-&space;nx)^{m}}{x^{2}}



[ برای مشاهده لینک ، لطفا با نام کاربری خود وارد شوید یا ثبت نام کنید ]{x\to&space;0}\frac{\sqrt{1&space;-&space;\cos&space;x^{2}}}{\tan&space;^{2}x}

eh_mn
12-08-2009, 21:09
با سلام
لطفا حد را محاسبه نمایید.

[ برای مشاهده لینک ، لطفا با نام کاربری خود وارد شوید یا ثبت نام کنید ]{x\to&space;0}\frac{1&space;-&space;\cos&space;x\sqrt{\cos&space;2x}}{x^{2}}


[ برای مشاهده لینک ، لطفا با نام کاربری خود وارد شوید یا ثبت نام کنید ]{align}\nonumber&space;\frac{1-\cos&space;x\sqrt{\cos&space;2x}}{x^{2}}&space;&&space;=\frac{(1-\cos&space;x\sqrt{\cos&space;2x})(1&plus;\cos&space;x\sqrt{\cos&space;2x})}{x^{ 2}(1&plus;\cos&space;x\sqrt{\cos&space;2x})}&space;\\\nonumber&space;&&space;=&space;\frac{1-\cos^2&space;x\cos&space;2x}{x^{2}(1&plus;\cos&space;x\sqrt{\cos&space;2x})}&space;\\ \nonumber&space;&&space;=&space;\frac{1-\cos&space;2x&space;&plus;&space;\sin^2x\cos&space;2x}{x^{2}(1&plus;\cos&space;x\sqrt{\cos &space;2x})}&space;\\\nonumber&space;&&space;=&space;\frac{\sin^2x(2&plus;\cos&space;2x)}{x^{2}(1&plus;\cos&space;x\sqrt{\ cos&space;2x)}}&space;\\\nonumber&space;\end{align}

پس


[ برای مشاهده لینک ، لطفا با نام کاربری خود وارد شوید یا ثبت نام کنید ]{x\to&space;0}\frac{1-\cos&space;x\sqrt{\cos&space;2x}}{x^{2}}&space;=&space;\frac{3}{2}

eh_mn
13-08-2009, 10:09
لطفا حدهای زیر را بدون استفاده از هوپیتال و هم ارزی حل کنید:



[ برای مشاهده لینک ، لطفا با نام کاربری خود وارد شوید یا ثبت نام کنید ]{x\to&space;0}\frac{\sqrt{1&space;-&space;\cos&space;x^{2}}}{\tan&space;^{2}x}

داريم


[ برای مشاهده لینک ، لطفا با نام کاربری خود وارد شوید یا ثبت نام کنید ]{\sqrt{1-\cos&space;x^{2}}}{\tan^{2}x}&space;=&space;\frac{\sqrt{1-\cos^2&space;x^{2}}}{\tan^{2}x\sqrt{1&plus;\cos&space;x^{2}}}&space;=\fra c{\sin&space;x^2}{\tan^{2}x\sqrt{1&plus;\cos&space;x^{2}}}

پس


[ برای مشاهده لینک ، لطفا با نام کاربری خود وارد شوید یا ثبت نام کنید ]{x\to&space;0}\frac{\sqrt{1-\cos&space;x^{2}}}{\tan^{2}x}&space;=&space;\frac{1}{\sqrt{2}}

ask_bl
13-08-2009, 11:05
داريم


[ برای مشاهده لینک ، لطفا با نام کاربری خود وارد شوید یا ثبت نام کنید ]{\sqrt{1-\cos&space;x^{2}}}{\tan^{2}x}&space;=&space;\frac{\sqrt{1-\cos^2&space;x^{2}}}{\tan^{2}x\sqrt{1&plus;\cos&space;x^{2}}}&space;=\fra c{\sin&space;x^2}{\tan^{2}x\sqrt{1&plus;\cos&space;x^{2}}}

پس



[ برای مشاهده لینک ، لطفا با نام کاربری خود وارد شوید یا ثبت نام کنید ]{x\to&space;0}\frac{\sqrt{1-\cos&space;x^{2}}}{\tan^{2}x}&space;=&space;\frac{1}{\sqrt{2}}


ممنون ولی از هم ارزی استفاده شده!

eh_mn
13-08-2009, 11:28
ممنون ولی از هم ارزی استفاده شده!

با استفاده از

[ برای مشاهده لینک ، لطفا با نام کاربری خود وارد شوید یا ثبت نام کنید ]{x\to&space;0}\frac{\sin&space;x}{x}=1

و قضاياي حد مي‏شه ثابت كرد كه

[ برای مشاهده لینک ، لطفا با نام کاربری خود وارد شوید یا ثبت نام کنید ]{x\to&space;0}\frac{\sin&space;x^2}{x^2}=1
و

[ برای مشاهده لینک ، لطفا با نام کاربری خود وارد شوید یا ثبت نام کنید ]{x\to&space;0}\frac{\tan^2&space;x}{x^2}=1
پس [ برای مشاهده لینک ، لطفا با نام کاربری خود وارد شوید یا ثبت نام کنید ]{x\to&space;0}\frac{\sin&space;x^2}{\tan^2&space;x}=1

ask_bl
13-08-2009, 11:31
تا جایی که من اطلاع دارم به

[ برای مشاهده لینک ، لطفا با نام کاربری خود وارد شوید یا ثبت نام کنید ]{x\to&space;0}\frac{\sin&space;x}{x}=1


میگن هم ارزی.(شایدم من اشتباه میکنم!)

eh_mn
13-08-2009, 12:43
تا جایی که من اطلاع دارم به

[ برای مشاهده لینک ، لطفا با نام کاربری خود وارد شوید یا ثبت نام کنید ]{x\to&space;0}\frac{\sin&space;x}{x}=1


میگن هم ارزی.(شایدم من اشتباه میکنم!)

من فكر مي‏كردم كه چون اين حد رو بطور تحليلي (با قضيه‏ي فشار ) تو دبيرستان ثابت مي‏كنن ديگه هم ارزي به حساب نمياد.!! :20:

ask_bl
13-08-2009, 12:59
من فكر مي‏كردم كه چون اين حد رو بطور تحليلي (با قضيه‏ي فشار ) تو دبيرستان ثابت مي‏كنن ديگه هم ارزي به حساب نمياد.!! :20:
بله اثبات میکنن ،ولی اصل سوالی که با استفاده از قضیه فشار اثبات شده از هم ارزی بدست اومده.
همونطور که شما هم میدونین سری تیلور Sinx میشه :

[ برای مشاهده لینک ، لطفا با نام کاربری خود وارد شوید یا ثبت نام کنید ]

که با صرفنظر کردن از جملات دوم به بعد وقتی x به سمت 0 میل میکنه جواب آن حد برابر 1 میشه.


در حقیقت شما یک سوال رو با هم ارزی حل کردید و با یه قضیه اثبات کردید که جواب درست است !!!
درسته ؟

eh_mn
13-08-2009, 14:17
بله اثبات میکنن ،ولی اصل سوالی که با استفاده از قضیه فشار اثبات شده از هم ارزی بدست اومده.
همونطور که شما هم میدونین سری تیلور Sinx میشه :

[ برای مشاهده لینک ، لطفا با نام کاربری خود وارد شوید یا ثبت نام کنید ]

که با صرفنظر کردن از جملات دوم به بعد وقتی x به سمت 0 میل میکنه جواب آن حد برابر 1 میشه.


در حقیقت شما یک سوال رو با هم ارزی حل کردید و با یه قضیه اثبات کردید که جواب درست است !!!
درسته ؟

تو دبيرستان سري تيلور رو نميگن. اول اثبات ميكنن كه براي x بين 0 و نصف pi داريم sin x< x < tan x . بعد طرفين رو بر sin x تقسيم ميكنن تا به نامساوي cos x / ‏1 > ‏x/sin x ‏> 1 برسيم و بعد با استفاده از قضيه فشار حكم مورد نظر بدست مياد. :20:

sinasalar2007
14-08-2009, 19:05
امکانش هست یکی این حد رو حل کنه ؟

[ برای مشاهده لینک ، لطفا با نام کاربری خود وارد شوید یا ثبت نام کنید ]

************

اگر کسی راهی رو بلده که بشه از اون طریق عبارت های ریاضی رو که داخل math type مینویسیم بشه داخل این سایت کپی پیست کرد ممنون میشم اگر راهنماییم کنه


برای مشاهده محتوا ، لطفا وارد شوید یا ثبت نام کنید

eh_mn
15-08-2009, 13:50
امکانش هست یکی این حد رو حل کنه ؟

[ برای مشاهده لینک ، لطفا با نام کاربری خود وارد شوید یا ثبت نام کنید ]



داريم

[ برای مشاهده لینک ، لطفا با نام کاربری خود وارد شوید یا ثبت نام کنید ]{align}\nonumber&space;\frac{xf(2)-2f(x)}{x-2}&space;&&space;=\frac{(xf(2)-2f(2))&plus;2f(2)&plus;(xf(x)-2f(x))-xf(x)}{x-2}\\\nonumber&space;&&space;=\frac{(x-2)(f(2)&plus;f(x))&plus;2f(2)-xf(x)}{x-2}&space;\\\nonumber&space;&&space;=&space;f(2)&plus;f(x)&plus;\frac{2f(2)-xf(x)}{x-2}&space;\end{align}

بنابراين اگر f در 2 مشتق‏پذير باشد آنگاه


[ برای مشاهده لینک ، لطفا با نام کاربری خود وارد شوید یا ثبت نام کنید ]{align}{\nonumber&space;\lim_{x\to&space;2}\fr ac{xf(2)-2f(x)}{x-2}&space;&&space;=&space;\lim_{x\to&space;2}\left(&space;f(2)&plus;f(x)&plus;\frac{2f(2)-xf(x)}{x-2}\right&space;)&space;\cr&space;&&space;=&space;2f(2)&space;-&space;\frac{d}{dx}(xf(x))_{x=2}&space;\cr&space;&&space;=&space;f(2)&space;-&space;2f'(2)&space;}\end{align}

eh_mn
15-08-2009, 20:54
به ازای کدام مقدار m در معادله ی درجه دوم [ برای مشاهده لینک ، لطفا با نام کاربری خود وارد شوید یا ثبت نام کنید ](m&plus;1)x^2-3x&plus;m=0 یکی از ریشه ها دو برابر ریشه ی دیگر است ؟

جواب: 1 و -2


اگه [ برای مشاهده لینک ، لطفا با نام کاربری خود وارد شوید یا ثبت نام کنید ] و [ برای مشاهده لینک ، لطفا با نام کاربری خود وارد شوید یا ثبت نام کنید ] ريشه‏هاي عبارت درجه دوي [ برای مشاهده لینک ، لطفا با نام کاربری خود وارد شوید یا ثبت نام کنید ]^2&plus;bx&plus;c باشند آنگاه

[ برای مشاهده لینک ، لطفا با نام کاربری خود وارد شوید یا ثبت نام کنید ]{align}{\alpha&plus;\beta&space;&&space;=&space;-\frac{b}{a}&space;\\&space;\alpha\beta&space;&&space;=&space;\frac{c}{a}&space;}\end{align}

در اين مسأله داريم [ برای مشاهده لینک ، لطفا با نام کاربری خود وارد شوید یا ثبت نام کنید ] بنابراين از رابطه‏ي (1) داريم

[ برای مشاهده لینک ، لطفا با نام کاربری خود وارد شوید یا ثبت نام کنید ]{1}{m&plus;1}
با جايگزاري در رابطه‏ي (2) يك معادله‏ي درجه دو بر حسب m بدست مياد كه حل اون جواب مساله رو بما ميده.



mohsen1472
16-08-2009, 19:13
با سلام و تشکر از پاسخ به سوال های قبلی من
لطفا دو حد زیر را نیز بدون استفاده از هوپیتال و هم ارزی حل کنید :

[ برای مشاهده لینک ، لطفا با نام کاربری خود وارد شوید یا ثبت نام کنید ]{x\to&space;\frac{\pi&space;}{3}}\frac{\sqrt{\s in&space;x}-\sqrt{\sin&space;2x}}{2\sin&space;x-\sqrt{3}}



[ برای مشاهده لینک ، لطفا با نام کاربری خود وارد شوید یا ثبت نام کنید ]{x\to&space;\pi&space;}\frac{1&space;-\cos&space;(1&space;&plus;&space;\cos&space;x)}{\sin&space;^{4}x}

sherlockholmz
17-08-2009, 12:01
با سلام و تشکر از پاسخ به سوال های قبلی من
لطفا دو حد زیر را نیز بدون استفاده از هوپیتال و هم ارزی حل کنید :

[ برای مشاهده لینک ، لطفا با نام کاربری خود وارد شوید یا ثبت نام کنید ] %7D%7D%5Cfrac%7B%5Csqrt%7B%5Csin&space;x%7D-%5Csqrt%7B%5Csin&space;2x%7D%7D%7B2%5Csin&space;x-%5Csqrt%7B3%7D%7D



[ برای مشاهده لینک ، لطفا با نام کاربری خود وارد شوید یا ثبت نام کنید ] 7D


با سلام
قدم اول-راهنمائي:
براي حد اول صورت كسر را گويا كن، سپس با معادله سينوس در برابر قوس صورت را ساده كن و سر انجام تغيير متغير u=x-pi/3 بده، ...قضيه حل است.

sherlockholmz
17-08-2009, 13:08
با سلام و تشکر از پاسخ به سوال های قبلی من
لطفا دو حد زیر را نیز بدون استفاده از هوپیتال و هم ارزی حل کنید :

[ برای مشاهده لینک ، لطفا با نام کاربری خود وارد شوید یا ثبت نام کنید ] %7D%7D%5Cfrac%7B%5Csqrt%7B%5Csin&space;x%7D-%5Csqrt%7B%5Csin&space;2x%7D%7D%7B2%5Csin&space;x-%5Csqrt%7B3%7D%7D



[ برای مشاهده لینک ، لطفا با نام کاربری خود وارد شوید یا ثبت نام کنید ] 7D


راهنمائي در مورد حد دوم:
1+cosx را u بگير، بعد ازآن سينوس ايكس بتوان 4 را محاسبه كن، يك منهاي كسينوس u را به سينوس نصف قوس تبديل كن، مخرج را ساده كن و با استفاده از قضيه افشردگي عاملهاي صفر صورت و مخرج را حذف كن.
روسا گفته اند بهتر است در اين موارد راهنمائي كني تا حل، با اين وجود اگر مشكلي بود بگو تا در حل كمكت كنم.
موفق باشيد.

mohsen1472
17-08-2009, 16:57
با سلام و تشکر از راهنماییتان
در مورد حد اول من متوجه منظورتان از "با معادله سينوس در برابر قوس صورت را ساده كن " نشدم و به اینجا رسیدم :



[ برای مشاهده لینک ، لطفا با نام کاربری خود وارد شوید یا ثبت نام کنید ]{x&space;\to&space;\frac{\pi&space;}{3}}\frac{\sin&space;x( 1-2\cos&space;x)}{(2\sin&space;x&space;-&space;\sqrt{3})(\sqrt{\sin&space;x}&plus;&space;\sqrt{\sin&space;2x})}


در مورد حد دوم من به اینجا رسیدم و بقیه را متوجه نشدم :


[ برای مشاهده لینک ، لطفا با نام کاربری خود وارد شوید یا ثبت نام کنید ]{x&space;\to&space;\pi&space;}\frac{1&space;-&space;\cos&space;u}{(1&space;-&space;\cos&space;x)^{2}(1&space;&plus;&space;\cos&space;x)^{2}}
با تشکر

sharif2011
19-08-2009, 21:08
با عرض سلام. یه سوال داشتم
در مثلث ABCزاویه A=30وAC=2BCایا می توان گفت زاویه B=90؟چرا؟

mohsen1472
22-08-2009, 08:40
با سلام
اگر [ برای مشاهده لینک ، لطفا با نام کاربری خود وارد شوید یا ثبت نام کنید ]{x&space;\to&space;1}\frac{x^{3}-&space;1}{x^{3}&plus;&space;ax^{2}&plus;&space;bx&space;&plus;&space;c}=&space;&plus;\infty
انگاه مقادیر a , b , c را حساب کنید.

mohsen1472
22-08-2009, 08:43
به ازای چه مقادیر a , b :

[ برای مشاهده لینک ، لطفا با نام کاربری خود وارد شوید یا ثبت نام کنید ]{x&space;\to&space;0}\frac{\cos&space;x}{a&space;-&space;b\cos&space;x}=&space;-\infty

eh_mn
22-08-2009, 20:24
با عرض سلام. یه سوال داشتم
در مثلث ABCزاویه A=30وAC=2BCایا می توان گفت زاویه B=90؟چرا؟

با رابطه‏ي سينوس‏ها در يك مثلث آشنايي دارين؟

اين رابطه ميگه كه اگر a، ‏b و c ضلع مقابل به زاويه‏هاي A، ‏B و C باشند آنگاه


[ برای مشاهده لینک ، لطفا با نام کاربری خود وارد شوید یا ثبت نام کنید ]{a}{\sin&space;A}=\frac{b}{\sin&space;B}=\frac{ c}{\sin&space;C} 

با استفاده از اين رابطه ميشه حكمي رو كه گفتين بررسي كرد.

eh_mn
22-08-2009, 20:26
با سلام
اگر [ برای مشاهده لینک ، لطفا با نام کاربری خود وارد شوید یا ثبت نام کنید ]{x&space;\to&space;1}\frac{x^{3}-&space;1}{x^{3}&plus;&space;ax^{2}&plus;&space;bx&space;&plus;&space;c}=&space;&plus;\infty
انگاه مقادیر a , b , c را حساب کنید.

آيا امكان داره كه هر سه برابر صفر باشند؟ آيا هر سه ميتونن 1 باشن؟ چرا؟
توجه كنين كه

[ برای مشاهده لینک ، لطفا با نام کاربری خود وارد شوید یا ثبت نام کنید ]{x^3-1}{x^3&plus;ax^2&plus;bx&plus;c}=\frac{(x-1)(x^2&plus;x&plus;1)}{x^3&plus;ax^2&plus;bx&plus;c}

mehdi_7070
24-08-2009, 10:35
آيا امكان داره كه هر سه برابر صفر باشند؟ آيا هر سه ميتونن 1 باشن؟ چرا؟
اگه a=-1 و b=-1 و c=1 باشه، درسته؟

panizir
24-08-2009, 12:22
لطفا این سوالو برام حل کنین:
تابع غیر ثابت f در x=0 مشتق پذیر است و f'(0)=2 , fi hchd iv و به ازای هر x و y حقیقی (f(x+y)=f(x)f(y در این صورت (f'(x دارای کدام علامت است؟
1) همواره منفی
2)هم علامت با(f(x
3)هم مثبت هم منفی
4)همواره برابر صفر است

sherlockholmz
25-08-2009, 10:04
با سلام و تشکر از راهنماییتان
در مورد حد اول من متوجه منظورتان از "با معادله سينوس در برابر قوس صورت را ساده كن " نشدم و به اینجا رسیدم :




با تشکر

باسلام
منظورم دو برابر قوس بود!!حال جهت جبران اين حد را برايت حل مي كنم.(اين است مجازات عدم دقت!)
نخست داريم:

[ برای مشاهده لینک ، لطفا با نام کاربری خود وارد شوید یا ثبت نام کنید ] 9+%5Csqrt%28sin2x%29%7D%7B%5Csqrt%28sinx%29+%5Csqr t%28sin2x%29%7D=%5Cfrac%7Bsinx-sin2x%7D%7B%282sinx-%5Csqrt3%29%28%5Csqrt%28sinx%29+%5Csqrt%28sin2x%29 %29%7D=%5Cfrac%7Bsinx-2sinxcosx%7D%7Bdenominator%7D=%5Cfrac%7Bsinx%281-2cosx%29%7D%7Bdenominator%7D

حال تغيير متغير u=x-pi/3 ميدهيم.

sherlockholmz
25-08-2009, 10:22
داريم:

[ برای مشاهده لینک ، لطفا با نام کاربری خود وارد شوید یا ثبت نام کنید ]

و مي دانيم:




[ برای مشاهده لینک ، لطفا با نام کاربری خود وارد شوید یا ثبت نام کنید ] siny

وهمچنين:


[ برای مشاهده لینک ، لطفا با نام کاربری خود وارد شوید یا ثبت نام کنید ] siny


با اين توضيحات كسر را ساده مي كنيم:


[ برای مشاهده لینک ، لطفا با نام کاربری خود وارد شوید یا ثبت نام کنید ]

بدين ترتيب ما كسر را بدو كسر معين و مبهم تقسيم كرديم.اگر ما از كسر دوم (كه باعث مبهم شدن حد مي شود )رفع ابهام كنيم، قضيه حل است.قبول داري؟

sherlockholmz
25-08-2009, 10:46
داريم:


[ برای مشاهده لینک ، لطفا با نام کاربری خود وارد شوید یا ثبت نام کنید ]

پس كسركلي ما به صورت ذيل تبديل مي شود:


[ برای مشاهده لینک ، لطفا با نام کاربری خود وارد شوید یا ثبت نام کنید ]

sherlockholmz
25-08-2009, 10:56
فكر كنم ديگه حل شد.نه؟
[ برای مشاهده لینک ، لطفا با نام کاربری خود وارد شوید یا ثبت نام کنید ] o%200%7D%5Cfrac%7Bsin%28u+%5Cpi/3%29%7D%7B%5Csqrt%28sin%28u+%5Cpi/3%29%29+%5Csqrt%28sin2%28u+%5Cpi/3%29%29%7D%5Ctimes%20%5Cfrac%7Bsinu/2+%5Csqrt3cosu/2%7D%7Bcosu/2-%5Csqrt3sinu/2%7D=%5Cfrac%7Bsin%5Cpi/3%5Ctimes%20%5Csqrt3%7D%7B%28%5Csqrt%20sin%5Cpi/3+%5Csqrt%20sin2%5Cpi/3%29%5Ctimes%201%7D=...=%5Cfrac%7B%5Csqrt%203%7D%7 B2%7D%5Ctimes%20%5Cfrac%7B%5Csqrt%5Csqrt3%20%7D%7B %5Csqrt2%7D



انشاا... كه در جاگذاري عددي اشتباهي نشده باشد.
موفق باشي

sharif2011
25-08-2009, 21:28
با سلام
یک سوال فوری داشتم
در مثلث abcزاویه a=30وac=2bcآ؟یا می توان گفت b=90؟چرا؟

sharif2011
25-08-2009, 21:34
در مثلث abc زاویه a=30و ac=2bcآیا می توان گفت b=90؟چرا؟

mehrdad_ati
26-08-2009, 06:48
سلام بر استادان گرامی

من دانش آموز پیش تجربی هستم (از مهر ان شا الله)

الان دارم برای کنکور 89 میخونم . دانش آموز سمپاد هم هستم

مشکل من و سایر همکلاسی هام اینه که معلم ما تعریف مشتق رو خیلی ساده از کنارش گذشت و ما هیچ مثالی و ... در این مورد کار نکردیم اما مشتقمون خیلی قویه !

من فعلا با برنامه مطالعاتی کانون پیش میرم و الان ریاضیاتش به تعریف مشتق رسیده و من در حال گریه کردنم !

چیز هایی که میدونم اینهاست :

فرمولهای تعریف مشتق (از اینترنت گرفتم) و روابط مشتق گیری

اینجوری بهتون بگم که تعریف مشتق ما در حدی ضعیفه که خود مسئله های بدون پاسخ کتاب کار کانون رو هم نمیتونیم حل کنیم ! حالا من فکر کردم که یکی دو تا سوال از تعریف مشتق اینجا بپرسم تا با دیدن حلشون دیگه روی غلتک بیفتم و تعریف مشتق رو متوجه بشم !

فقط چیزی که برام مهمه اینه که برنامه مطالعاتیم بهم نخوره آخه امروز رو برای ریاضی گذاشتم و میخوام همین امروز از شر تعریف مشتق خلاص شم و برم سراغ درس های دیگه

فقط اگر ممکن هست شمرده به شمرده و برای کسی که هیچی از تعریف مشتق نمیدونه توضیح بدید (البته ریاضیم بد نیست ) (نمره نهایی 18.5)

و اما سوالها (از خود کتاب کار کانون هست)

[ برای مشاهده لینک ، لطفا با نام کاربری خود وارد شوید یا ثبت نام کنید ]

و

[ برای مشاهده لینک ، لطفا با نام کاربری خود وارد شوید یا ثبت نام کنید ]


پیشاپیش از کمکتون ممنونم

maziar92142
26-08-2009, 07:48
سلام بر استادان گرامی

من دانش آموز پیش تجربی هستم (از مهر ان شا الله)

الان دارم برای کنکور 89 میخونم . دانش آموز سمپاد هم هستم

مشکل من و سایر همکلاسی هام اینه که معلم ما تعریف مشتق رو خیلی ساده از کنارش گذشت و ما هیچ مثالی و ... در این مورد کار نکردیم اما مشتقمون خیلی قویه !

من فعلا با برنامه مطالعاتی کانون پیش میرم و الان ریاضیاتش به تعریف مشتق رسیده و من در حال گریه کردنم !

چیز هایی که میدونم اینهاست :

فرمولهای تعریف مشتق (از اینترنت گرفتم) و روابط مشتق گیری

اینجوری بهتون بگم که تعریف مشتق ما در حدی ضعیفه که خود مسئله های بدون پاسخ کتاب کار کانون رو هم نمیتونیم حل کنیم ! حالا من فکر کردم که یکی دو تا سوال از تعریف مشتق اینجا بپرسم تا با دیدن حلشون دیگه روی غلتک بیفتم و تعریف مشتق رو متوجه بشم !

فقط چیزی که برام مهمه اینه که برنامه مطالعاتیم بهم نخوره آخه امروز رو برای ریاضی گذاشتم و میخوام همین امروز از شر تعریف مشتق خلاص شم و برم سراغ درس های دیگه

فقط اگر ممکن هست شمرده به شمرده و برای کسی که هیچی از تعریف مشتق نمیدونه توضیح بدید (البته ریاضیم بد نیست ) (نمره نهایی 18.5)

و اما سوالها (از خود کتاب کار کانون هست)

[ برای مشاهده لینک ، لطفا با نام کاربری خود وارد شوید یا ثبت نام کنید ]

و

[ برای مشاهده لینک ، لطفا با نام کاربری خود وارد شوید یا ثبت نام کنید ]


پیشاپیش از کمکتون ممنونم
خيلي راحت اين همون ضرب F G هستش شما ضرب رو با متغير X مي نويسي و مشتق خودتو مي گيزي و در نهايت به جاي X عدد 2 رو ميزاري
فهمش اين طوريه كه وقتي دلتا رو بزاري كنار صفر روي صفر به دست مياد و مي فهمي مشتق راحته

mehrdad_ati
26-08-2009, 08:33
یک .رده بیشتر توضیح بدید بابا ناسلامتی تجربی هستم ها ! :D

ما اصلا تعریف مشتق برای ترکیب توابع تو ریاضی 3 نداریم k

mir@
26-08-2009, 19:37
سلام بر استادان گرامی

من دانش آموز پیش تجربی هستم (از مهر ان شا الله)

الان دارم برای کنکور 89 میخونم . دانش آموز سمپاد هم هستم

مشکل من و سایر همکلاسی هام اینه که معلم ما تعریف مشتق رو خیلی ساده از کنارش گذشت و ما هیچ مثالی و ... در این مورد کار نکردیم اما مشتقمون خیلی قویه !

من فعلا با برنامه مطالعاتی کانون پیش میرم و الان ریاضیاتش به تعریف مشتق رسیده و من در حال گریه کردنم !

چیز هایی که میدونم اینهاست :

فرمولهای تعریف مشتق (از اینترنت گرفتم) و روابط مشتق گیری

اینجوری بهتون بگم که تعریف مشتق ما در حدی ضعیفه که خود مسئله های بدون پاسخ کتاب کار کانون رو هم نمیتونیم حل کنیم ! حالا من فکر کردم که یکی دو تا سوال از تعریف مشتق اینجا بپرسم تا با دیدن حلشون دیگه روی غلتک بیفتم و تعریف مشتق رو متوجه بشم !

فقط چیزی که برام مهمه اینه که برنامه مطالعاتیم بهم نخوره آخه امروز رو برای ریاضی گذاشتم و میخوام همین امروز از شر تعریف مشتق خلاص شم و برم سراغ درس های دیگه

فقط اگر ممکن هست شمرده به شمرده و برای کسی که هیچی از تعریف مشتق نمیدونه توضیح بدید (البته ریاضیم بد نیست ) (نمره نهایی 18.5)

و اما سوالها (از خود کتاب کار کانون هست)

[ برای مشاهده لینک ، لطفا با نام کاربری خود وارد شوید یا ثبت نام کنید ]

و

[ برای مشاهده لینک ، لطفا با نام کاربری خود وارد شوید یا ثبت نام کنید ]


پیشاپیش از کمکتون ممنونم

تعریف مشتق به این صورته

[ برای مشاهده لینک ، لطفا با نام کاربری خود وارد شوید یا ثبت نام کنید ]'(x)=\lim_{h\to&space;0}\frac{f(x&plus;h)-f(x)}{h}=\lim_{h\to&space;0}\frac{f(x)-f(x-h)}{h}

حالا مسئله اول رو به شکل زیر می نویسیم و یک f(x)l اضافه و کم می کنیم و چون h به سمت صفر میره، پس 2h هم به سمت صفر میره و اسم 2h رو میذاریم t یعنی تغییر متغیر میدیم:

[ برای مشاهده لینک ، لطفا با نام کاربری خود وارد شوید یا ثبت نام کنید ]'(x)=\lim_{h\to&space;0}\frac{f(x&plus;2h)-f(x-2h)}{3h}=\frac{2}{3}\lim_{h\to&space;0}\frac{f(x&plus;2h)-f(x-2h)}{2h}\\&space;=\frac{2}{3}\lim_{t\to&space;0}\frac{\big(f(x &plus;t)-f(x)\big)&plus;\big(f(x)-f(x-t)\big)}{t}\\&space;=\frac{2}{3}\left(&space;\lim_{t\to&space;0}\fra c{\big(f(x&plus;t)-f(x)\big)}{t}&plus;\lim_{t\to&space;0}\frac{\big(f(x)-f(x-t)\big)}{t}\right&space;)=\\&space;=\frac{2}{3}\big(f'(x)&plus;f'(x )&space;\big)=\frac{4}{3}f'(x)=2\sqrt{x}\&space;\Rightarrow&space;f' (x)=\frac{3}{2}\sqrt{x}&space;\Rightarrow&space;f'(9)=\frac{9} {2}


و اما مسئله دوم هم که تعریف مشتق حاصل ضرب دو تابع هست که در نقطه 2 محاسبه میشه


[ برای مشاهده لینک ، لطفا با نام کاربری خود وارد شوید یا ثبت نام کنید ](FG)'=F'G&plus;G'F

بنابراین


[ برای مشاهده لینک ، لطفا با نام کاربری خود وارد شوید یا ثبت نام کنید ]'(x)=2x-1&space;\Rightarrow&space;f'(2)=3\\&space;g'(x)=\frac{\sqrt{2}}{2\sq rt{x}}\Rightarrow&space;g'(2)=\frac{1}{2}\\


[ برای مشاهده لینک ، لطفا با نام کاربری خود وارد شوید یا ثبت نام کنید ](&space;f(x)g(x)\right&space;)'\big |_2=f(2)g'(2)&plus;g(2)f'(2)

که جواب میشه 7

خدا کنه درست باشه [ برای مشاهده لینک ، لطفا با نام کاربری خود وارد شوید یا ثبت نام کنید ]

mehrdad_ati
26-08-2009, 23:37
ددرسته و درسته دمت گرم گرفتم چی شد !

mehrdad_ati
26-08-2009, 23:42
راستی من میتونم از این به بعد مشکلاتم رو اینجا مطرح کنم ؟ ایرادی نداره ؟

Mohammad Hosseyn
28-08-2009, 02:29
با سلام خدمت تمامی دوستان ...
راستش خیلی وقته که ریاضیات رو نتونستم خوب بخونم و همیشه مشکل داشتم توش . اما خوب کنکور آدم رو مجبور میکنه دیگه . چند وقتی هست که دارم کل فیزیک دبیرستان و پیش دانشگاهی رو دوباره مرور میکنم و در این میان مغرم بعضی جاها هنگ میکنه . بخاطر همین تصمیم گرفتم سوالاتمو اینجا مطرح کنم .
امیدوارم که بی جواب نمونن .
فعلا چون تو مبحث دنباله ها و حد دنباله هستم چندتا سوال در مورد این مورد دارم که میپرسم ... فقط تو رو خدا ببخشید که خیلی آماتوریه ، اما نیاز دارم که از بن و ریشه مفهوم جوابها رو بفهمم .
پیشاپیش ممنون ...

سوال اول :

این سوال میگه که یه دنباله داریم به فرم زیر :


[ برای مشاهده لینک ، لطفا با نام کاربری خود وارد شوید یا ثبت نام کنید ]

بعد سوال اینه که چندتا از جملات این دنباله در بازه ی (3.9 , 4) قرار میگیرن ؟
جوابی که من دارم 39 هست ولی نتونستم با محاسبه در بیارم چرا میشه این . (این جوابو از پاسخنامه سوال دارم که تشریحی نیست !!)

سوال دوم :
خدائیش نخندینا ... این سوال خیلی بدیهی به نظر میاد اما من هر کاری میکنم نمی دونم چرا جواب من اونی نیست که تو پاسخ نامه هست !!! (فکر کنم مخم ایراد پیدا کرده !)
سوال اینه که دنباله ی زیر به چه عدید همگراست


[ برای مشاهده لینک ، لطفا با نام کاربری خود وارد شوید یا ثبت نام کنید ]

همینطور در همگرایی زیر هم مشکل دارم . من اصلا این دنباله رو واگرا میبینم . چون ظابطه پائین که همواره برابر 1 هست و ظابطه ی بالا هم نوسان داره بین -1 و +1 . پس دنباله همگرا نباید باشه !


[ برای مشاهده لینک ، لطفا با نام کاربری خود وارد شوید یا ثبت نام کنید ]

و دوتا مورد دیگه هم بود که همین الان با فشار آوردن به مخ حلشون کردم ...

فعلا ...
موفق باشید ...

ali_hp
28-08-2009, 09:34
با سلام خدمت تمامی دوستان ...
راستش خیلی وقته که ریاضیات رو نتونستم خوب بخونم و همیشه مشکل داشتم توش . اما خوب کنکور آدم رو مجبور میکنه دیگه . چند وقتی هست که دارم کل فیزیک دبیرستان و پیش دانشگاهی رو دوباره مرور میکنم و در این میان مغرم بعضی جاها هنگ میکنه . بخاطر همین تصمیم گرفتم سوالاتمو اینجا مطرح کنم .
امیدوارم که بی جواب نمونن .
فعلا چون تو مبحث دنباله ها و حد دنباله هستم چندتا سوال در مورد این مورد دارم که میپرسم ... فقط تو رو خدا ببخشید که خیلی آماتوریه ، اما نیاز دارم که از بن و ریشه مفهوم جوابها رو بفهمم .
پیشاپیش ممنون ...

سوال اول :

این سوال میگه که یه دنباله داریم به فرم زیر :


[ برای مشاهده لینک ، لطفا با نام کاربری خود وارد شوید یا ثبت نام کنید ]

بعد سوال اینه که چندتا از جملات این دنباله در بازه ی (3.9 , 4) قرار میگیرن ؟
جوابی که من دارم 39 هست ولی نتونستم با محاسبه در بیارم چرا میشه این . (این جوابو از پاسخنامه سوال دارم که تشریحی نیست !!)
سلام
قرار میگیرن یا قرار نمیگیرن؟
چون این دنباله به چهار منفی میل می کند،و بی نهایت جمله در این بازه دارد...


سوال دوم :
خدائیش نخندینا ... این سوال خیلی بدیهی به نظر میاد اما من هر کاری میکنم نمی دونم چرا جواب من اونی نیست که تو پاسخ نامه هست !!! (فکر کنم مخم ایراد پیدا کرده !)
سوال اینه که دنباله ی زیر به چه عدید همگراست


[ برای مشاهده لینک ، لطفا با نام کاربری خود وارد شوید یا ثبت نام کنید ]


با توجه به اینکه:
[ برای مشاهده لینک ، لطفا با نام کاربری خود وارد شوید یا ثبت نام کنید ]{2n-1}{n&plus;1}=\frac{2(n&plus;1)-3}{n&plus;1}=2-\frac{3}{n&plus;1}

داخل جز صحیح به دو منفی میل می کند،پس حد عبارت یک می شود.



همینطور در همگرایی زیر هم مشکل دارم . من اصلا این دنباله رو واگرا میبینم . چون ظابطه پائین که همواره برابر 1 هست و ظابطه ی بالا هم نوسان داره بین -1 و +1 . پس دنباله همگرا نباید باشه !


[ برای مشاهده لینک ، لطفا با نام کاربری خود وارد شوید یا ثبت نام کنید ]

و دوتا مورد دیگه هم بود که همین الان با فشار آوردن به مخ حلشون کردم ...

فعلا ...
موفق باشید ...


دقت کنید که هزار جمله اول دنباله هیچ تاثیری در همگرایی و واگرایی و یا حد دنباله ندارد!اگر جملات این دنباله را بنوسید،هزار جمله اول بین منفی یک و یک نوسان دارد،و جملات بعدی برابر یک هستند،پس حد دنباله برابر یک است.به عبارتی برای بررسی حد،جملات دنباله در بینهایت مهم هستند!

Mohammad Hosseyn
29-08-2009, 04:16
با سلام ...


سلام
قرار میگیرن یا قرار نمیگیرن؟
چون این دنباله به چهار منفی میل می کند،و بی نهایت جمله در این بازه دارد...

قرار نمی گیرد درست بود ، من اشتباه تایپ کرده بودم ....


با توجه به اینکه:

[ برای مشاهده لینک ، لطفا با نام کاربری خود وارد شوید یا ثبت نام کنید ]{2n-1}{n&plus;1}=%5Cfrac{2(n&plus;1)-3}{n&plus;1}=2-%5Cfrac{3}{n&plus;1}
داخل جز صحیح به دو منفی میل می کند،پس حد عبارت یک می شود.


دقت کنید که هزار جمله اول دنباله هیچ تاثیری در همگرایی و واگرایی و یا حد دنباله ندارد!اگر جملات این دنباله را بنوسید،هزار جمله اول بین منفی یک و یک نوسان دارد،و جملات بعدی برابر یک هستند،پس حد دنباله برابر یک است.به عبارتی برای بررسی حد،جملات دنباله در بینهایت مهم هستند!

بله اتفاق من هم به همین چیزی که شما میگید فکر کردم ... اما چون یه مساله دیگه بود که با این مسئله یه فرم بود . یعنی دنباله هاش شبیه این بودن اما جواب یه جور دیگه بدست میومد ... اگه پیداش کردم میزارم اینجا حتما .

ممنون بابت پاسخگویی

Mohammad Hosseyn
29-08-2009, 05:39
با سلام ... چند تا سوال مبتدیانه ی دیگه (اسمایل چهره شطرنجی)
دوستان حد این دنباله چند هست ؟

[ برای مشاهده لینک ، لطفا با نام کاربری خود وارد شوید یا ثبت نام کنید ]

گزینه های موجود در تست :
1- 0

2- ∞

3- e

4- e^2

راستی لطفا در مورد عدد e یه توضیحی کوتاه بدید ... چی هست ، از کجا اومده ، کجا ها کاربرد داره .

بسیار سپاسگذار .

mohsen1472
30-08-2009, 09:45
با سلام
با استفاده از قضیه فشردگی ثابت کنید تابع دو ضابطه ای زیر در x=3 حد دارد :
(x - 4)- ایکس عضو مجموعه اعداد گویا
x - 2 ایکس عضو مجموعه اعداد اصم

davy jones
30-08-2009, 12:54
با سلام ... چند تا سوال مبتدیانه ی دیگه (اسمایل چهره شطرنجی)
دوستان حد این دنباله چند هست ؟

[ برای مشاهده لینک ، لطفا با نام کاربری خود وارد شوید یا ثبت نام کنید ]

گزینه های موجود در تست :
1- 0

2- ∞

3- e

4- e^2

راستی لطفا در مورد عدد e یه توضیحی کوتاه بدید ... چی هست ، از کجا اومده ، کجا ها کاربرد داره .

بسیار سپاسگذار .

چون حد داخل پرانتز میشه 2 بنابراین حد کل به سمت بینهایت میره و گزینه 2 صحیحه.
اگر ابهام حد از نوع یک به توان بینهایت بود اونوقت جواب از نوع عدد e میشد. برای مطالعه بیشتر در مورد عدد نپر(e) به این آدرس مراجعه کنید:


برای مشاهده محتوا ، لطفا وارد شوید یا ثبت نام کنید


موفق باشین.
88/6/8

eh_mn
02-09-2009, 15:21
با سلام ... چند تا سوال مبتدیانه ی دیگه (اسمایل چهره شطرنجی)
دوستان حد این دنباله چند هست ؟

[ برای مشاهده لینک ، لطفا با نام کاربری خود وارد شوید یا ثبت نام کنید ]

گزینه های موجود در تست :
1- 0

2- ∞

3- e

4- e^2

راستی لطفا در مورد عدد e یه توضیحی کوتاه بدید ... چی هست ، از کجا اومده ، کجا ها کاربرد داره .

بسیار سپاسگذار .

مي‌دونين اين حد چطور حساب ميشه؟


[ برای مشاهده لینک ، لطفا با نام کاربری خود وارد شوید یا ثبت نام کنید ]{x\to\infty}\left(&space;1&plus;\frac{1}{x}&space;\r ight&space;)^x

فرض كنيد عبارت مقابل حد را f بناميم. در اين صورت


[ برای مشاهده لینک ، لطفا با نام کاربری خود وارد شوید یا ثبت نام کنید ](x)=\frac{\ln&space;\left(1&plus;\frac{1}{x}&space;\ right&space;)}{\frac{1}{x}}

حالا با حد گيري از طرفين رابطه‌ي اخير داريم + هوپيتال


[ برای مشاهده لینک ، لطفا با نام کاربری خود وارد شوید یا ثبت نام کنید ]{x\to\infty}\ln&space;f(x)=\lim_{x\to\inf ty}\frac{\ln&space;\left(1&plus;\frac{1}{x}&space;\right&space;)}{\frac{1 }{x}}=1

(چون تابع ln پيوسته است) پس


[ برای مشاهده لینک ، لطفا با نام کاربری خود وارد شوید یا ثبت نام کنید ]{x\to\infty}f(x)=1

بنابراين


[ برای مشاهده لینک ، لطفا با نام کاربری خود وارد شوید یا ثبت نام کنید ]{\displaystyle\lim_{x\to\infty}\lef t(1&plus;\frac{1}{x}&space;\right&space;)^x=e}

با استفاده از اين ميشه حد مورد سوال رو محاسبه كرد.



Mohammad Hosseyn
02-09-2009, 17:44
با سلام ... به یک مورد در مورد کرانداری دنباله برخوردم که یکم برام سوال بر انگیز بود .
در مورد دنباله ی زیر :

[ برای مشاهده لینک ، لطفا با نام کاربری خود وارد شوید یا ثبت نام کنید ]

اگه بخوایم کرانداریش رو بررسی کنیم ممکنه اشتباه کنیم و اون رو بیکران قلمداد کنیم ، اما در واقع کراندار هست .
روش مناسب برای این دنباله همون قاعده ی ضرب کراندار در کراندار ، دنباله ای کراندار خواهد بود ، هست .
اما من میخوام بدونم که اگه بخوایم با روش معمولی تر ، یعنی اول نشون بدیم دنباله حد داره و حدش برابر L هست و بعد هم نشون بدیم دنباله یکنوا هست . پس اونوقت باید کراندار باشه . (فک کنم هم حدش مشکل باشه و هم یکنواییش) . اما بالاخره باید این دو شرط رو داشته باشه دیگه ؟ درست میگم ؟
.........................
سوالی دیگر در باب دنباله ها دارم که بازم میپرسم . یه تست هست که میگه :
اگر دنباله ی زیر همگرا باشد ، آنگاه حدود x کدام است .

[ برای مشاهده لینک ، لطفا با نام کاربری خود وارد شوید یا ثبت نام کنید ]
گزینه 1 ) x>1/2
گزینه 2 ) x>1/2 و x=1/2
گزینه 3 ) x<1/2
گزینه 4 ) x<1/2 و x=1/2
در گزینه ها علامت "/" همون خط کسری است .
من خودم جواب رو گزینه 2 در آوردم . اما خیلی شک دارم . اگه میشه راه حل این مسائله رو بگید ، حتی اگه گزینه 2 درست باشه .
موفق باشید .

mehdi_7070
02-09-2009, 18:03
در مورد کرانداری دنباله‌ی اول، فکر می‌کنم این استدلال هم درست باشه:

[ برای مشاهده لینک ، لطفا با نام کاربری خود وارد شوید یا ثبت نام کنید ] ght%20%7C%3C%202

Mohammad Hosseyn
02-09-2009, 21:49
در مورد کرانداری دنباله‌ی اول، فکر می‌کنم این استدلال هم درست باشه:

[ برای مشاهده لینک ، لطفا با نام کاربری خود وارد شوید یا ثبت نام کنید ] ght%20%7C%3C%202

بله مهدی جان ، نتیجه این میشه ... ولی من میخوام بدونم چرا این میشه (از راه حل دو مرحله ای که گفتم ..) وگرنه اونجوری که آب خوردنه .

eh_mn
02-09-2009, 22:40
با سلام ... به یک مورد در مورد کرانداری دنباله برخوردم که یکم برام سوال بر انگیز بود .
در مورد دنباله ی زیر :

[ برای مشاهده لینک ، لطفا با نام کاربری خود وارد شوید یا ثبت نام کنید ]

اگه بخوایم کرانداریش رو بررسی کنیم ممکنه اشتباه کنیم و اون رو بیکران قلمداد کنیم ، اما در واقع کراندار هست .
روش مناسب برای این دنباله همون قاعده ی ضرب کراندار در کراندار ، دنباله ای کراندار خواهد بود ، هست .
اما من میخوام بدونم که اگه بخوایم با روش معمولی تر ، یعنی اول نشون بدیم دنباله حد داره و حدش برابر L هست و بعد هم نشون بدیم دنباله یکنوا هست . پس اونوقت باید کراندار باشه . (فک کنم هم حدش مشکل باشه و هم یکنواییش) . اما بالاخره باید این دو شرط رو داشته باشه دیگه ؟ درست میگم ؟
.........................
سوالی دیگر در باب دنباله ها دارم که بازم میپرسم . یه تست هست که میگه :


شايد منظور شما اين قضيه باشه
هر دنباله‌ی يكنوا و كراندار همگراست.؟


دنباله‌ی [ برای مشاهده لینک ، لطفا با نام کاربری خود وارد شوید یا ثبت نام کنید ](-1)^n نه همگراست و نه يكنوا، ولی كرانداره. (بنا براين برای كرانداری يك دنباله يكنوايی يا همگرايی لازم نيست) از طرفي
هر دنباله‌ی همگرا كراندار است. پس اگه بتونيم ثابت كنيم يك دنباله همگراست كراندار هم هست.

Mohammad Hosseyn
03-09-2009, 00:16
شايد منظور شما اين قضيه باشه
هر دنباله‌ی يكنوا و كراندار همگراست.؟


دنباله‌ی [ برای مشاهده لینک ، لطفا با نام کاربری خود وارد شوید یا ثبت نام کنید ](-1)^n نه همگراست و نه يكنوا، ولی كرانداره. (بنا براين برای كرانداری يك دنباله يكنوايی يا همگرايی لازم نيست) از طرفي
هر دنباله‌ی همگرا كراندار است. پس اگه بتونيم ثابت كنيم يك دنباله همگراست كراندار هم هست.

بله منظورم همون قاعدست ... پس در اصل باید بگیم همگرایی و یکنوایی شرط کافی برای کرانداری دنباله هست اما عکس ان صادق نیست .

ممنون .

Mohammad Hosseyn
03-09-2009, 04:09
چندتا مورد دیگه در مبحث کرانداری و یکنوایی دنباله ها ... خوشحال میشوم شک منو برطرف کنید .
دنباله ی زیر از نظر همگرایی ، یکنوایی و کرانداری چگونه هست ؟
[ برای مشاهده لینک ، لطفا با نام کاربری خود وارد شوید یا ثبت نام کنید ]

...............................................

همچنین دنباله ی زیر از نظر یکنوایی و کرانداری چگونه است ؟

[ برای مشاهده لینک ، لطفا با نام کاربری خود وارد شوید یا ثبت نام کنید ]

.................................................

همچنین تست زیر که ، کدام گزینه در مورد دنباله ی زیر درست می باشید ؟

[ برای مشاهده لینک ، لطفا با نام کاربری خود وارد شوید یا ثبت نام کنید ]

گزینه 1) همگرا به صفر
گزینه 2) همگرا به 1
گزینه 3) واگرا
گزینه 4) به ازای بعضی مقادیر n دنباله تعریف نشده است

mehdi_7070
03-09-2009, 15:50
همچنین تست زیر که ، کدام گزینه در مورد دنباله ی زیر درست می باشید ؟

[ برای مشاهده لینک ، لطفا با نام کاربری خود وارد شوید یا ثبت نام کنید ]

گزینه 1) همگرا به صفر
گزینه 2) همگرا به 1
گزینه 3) واگرا
گزینه 4) به ازای بعضی مقادیر n دنباله تعریف نشده است

سرعت رشد توابع این‌جوری بود (a>1)


[ برای مشاهده لینک ، لطفا با نام کاربری خود وارد شوید یا ثبت نام کنید ] 0n%5Ea%20%3E%20%5Csqrt%7Bn%7D%20%3E%20%5Clog%7Bn%7 D

یعنی واگرا... امیدوارم اشتباه نشده باشه!

amintnt
03-09-2009, 16:23
دنباله ی زیر از نظر همگرایی ، یکنوایی و کرانداری چگونه هست ؟
[ برای مشاهده لینک ، لطفا با نام کاربری خود وارد شوید یا ثبت نام کنید ]سرعت رشد مخرج بیشتر از صورته... بنا بر این به صفر همگراست... و چون همگراست کرانداره... نزولی هست تابع...


همچنین دنباله ی زیر از نظر یکنوایی و کرانداری چگونه است ؟

[ برای مشاهده لینک ، لطفا با نام کاربری خود وارد شوید یا ثبت نام کنید ]به یک همگراست... پس کرانداره... و همچنین احتمالا صعودیه... چون کسینوس زاویه از کمتر از یک داره به یک نزدیک میشه و پرانتز هم همینطور... بنابر این هر دو دارن بزرگتر میشن... پس ضربشون هم باید بزرگتر بشه!


همچنین تست زیر که ، کدام گزینه در مورد دنباله ی زیر درست می باشید ؟

[ برای مشاهده لینک ، لطفا با نام کاربری خود وارد شوید یا ثبت نام کنید ]

گزینه 1) همگرا به صفر
گزینه 2) همگرا به 1
گزینه 3) واگرا
گزینه 4) به ازای بعضی مقادیر n دنباله تعریف نشده است
مهدی جان توی پست قبل جواب دادن... واگراست... چون مخرج کرانداره و صورت به بینهایت میل میکنه...

eh_mn
03-09-2009, 23:02
يك سوال جالب: ميتونين به طور دقيق ثابت كنين كه دنباله‌ی sin n همگرا نيست؟

saber57
07-09-2009, 18:28
يك سوال جالب: ميتونين به طور دقيق ثابت كنين كه دنباله‌ی sin n همگرا نيست؟
نقطه همگرایی نقطه ای ایست که وقتی تابع به سمت بینهایت میل میکند، حد تابع به سمت نقطه مشخصی میل میکند .فرض کنیم تابع sin n به سمت نقطه مشخصی مانند a میل کند که این نقطه هم یکتاست از طرفی تابع سینوس تابعی متناوب است (با دوره تناوب [ برای مشاهده لینک ، لطفا با نام کاربری خود وارد شوید یا ثبت نام کنید ]) پس [ برای مشاهده لینک ، لطفا با نام کاربری خود وارد شوید یا ثبت نام کنید ](a)=sin(2\pi&space;&plus;a) پس نقطه همگرایی مشخصی نمیتوان یافت و تابع در هر دوره تناوب تکرار میشود .

eh_mn
09-09-2009, 22:29
نقطه همگرایی نقطه ای ایست که وقتی تابع به سمت بینهایت میل میکند، حد تابع به سمت نقطه مشخصی میل میکند .فرض کنیم تابع sin n به سمت نقطه مشخصی مانند a میل کند که این نقطه هم یکتاست از طرفی تابع سینوس تابعی متناوب است (با دوره تناوب [ برای مشاهده لینک ، لطفا با نام کاربری خود وارد شوید یا ثبت نام کنید ]) پس [ برای مشاهده لینک ، لطفا با نام کاربری خود وارد شوید یا ثبت نام کنید ](a)=sin(2\pi&space;&plus;a) پس نقطه همگرایی مشخصی نمیتوان یافت و تابع در هر دوره تناوب تکرار میشود .

با سلام

تساوي [ برای مشاهده لینک ، لطفا با نام کاربری خود وارد شوید یا ثبت نام کنید ](a)=\sin(2\pi&space;&plus;a) چه چيزي رو نقض ميكنه؟ يعني چه طور با يكتايي حد در تناقضه؟ گمان كنم يه چيزي اشتباه شده. اسم حد رو a گذاشتين بعد دوباره sin گرفتين.

saber57
10-09-2009, 22:46
با سلام

تساوي [ برای مشاهده لینک ، لطفا با نام کاربری خود وارد شوید یا ثبت نام کنید ](a)=\sin(2\pi&space;&plus;a) چه چيزي رو نقض ميكنه؟ يعني چه طور با يكتايي حد در تناقضه؟ گمان كنم يه چيزي اشتباه شده. اسم حد رو a گذاشتين بعد دوباره sin گرفتين.
a نقطه ای از دامنه فرض شده که در آن ،تابع همگراست و مقدار برد تابع sin a هست ولی چون تابع متناوب هست پس در دوره های تناوب دیگر تکرار میشود و تابع همگرا نیست

eh_mn
11-09-2009, 12:58
a نقطه ای از دامنه فرض شده که در آن ،تابع همگراست و مقدار برد تابع sin a هست ولی چون تابع متناوب هست پس در دوره های تناوب دیگر تکرار میشود و تابع همگرا نیست

با اين سوال و جواب، پست‌هاي شما دقيقاً دو برابر پست‌هاي من شد:20:

تا اينجا رو فهميدم. اگه جايي اشتباه كردم تذكر بدين لطفا. فرض كنيم دنباله‌ي sin n همگرا به t باشد. چون sin بين -1 و 1 است پس t هم. بنابراين عددي مانند a در بازه 0 و 2pi موجود است كه sin a=t. بنابراين ....

اگه ممكنه كاملش كنين.

ممنون

saber57
11-09-2009, 14:16
در بازه [ برای مشاهده لینک ، لطفا با نام کاربری خود وارد شوید یا ثبت نام کنید ][&space;0,2\pi&space;\right&space;] دو قسمت زیر را در نظر میگیریم :
برای مقادیر مثبت



[ برای مشاهده لینک ، لطفا با نام کاربری خود وارد شوید یا ثبت نام کنید ][&space;0&space;,\pi&space;\right&space;]\to&space;t=sin(a)&space;=sin(\pi&space;-a)


بنابراین به ازای هر دو نقطه یک مقدار برد وجود دارد و نقطه یکتایی نداریم
برای مقادیر منفی(برد):

[ برای مشاهده لینک ، لطفا با نام کاربری خود وارد شوید یا ثبت نام کنید ][&space;\pi&space;,2\pi&space;\right&space;]\to&space;t=sin(\pi&space;&plus;a)&space;=sin(2\pi&space;-a)


برای نقاط با برد منفی هم همینطور هست. در مورد نقاط اکسترمم هم که مشخصه . نقطه اکسترمم که نقطه همگرایی نیست.

eh_mn
16-09-2009, 22:48
در بازه [ برای مشاهده لینک ، لطفا با نام کاربری خود وارد شوید یا ثبت نام کنید ][&space;0,2\pi&space;\right&space;] دو قسمت زیر را در نظر میگیریم :
برای مقادیر مثبت



[ برای مشاهده لینک ، لطفا با نام کاربری خود وارد شوید یا ثبت نام کنید ][&space;0&space;,\pi&space;\right&space;]\to&space;t=sin(a)&space;=sin(\pi&space;-a)


بنابراین به ازای هر دو نقطه یک مقدار برد وجود دارد و نقطه یکتایی نداریم
برای مقادیر منفی(برد):

[ برای مشاهده لینک ، لطفا با نام کاربری خود وارد شوید یا ثبت نام کنید ][&space;\pi&space;,2\pi&space;\right&space;]\to&space;t=sin(\pi&space;&plus;a)&space;=sin(2\pi&space;-a)


برای نقاط با برد منفی هم همینطور هست. در مورد نقاط اکسترمم هم که مشخصه . نقطه اکسترمم که نقطه همگرایی نیست.

براي همه مقدارهايي كه شما ميگين مقدار حد t ميشه؟ كه يكتا هم هست.

Mohammad Hosseyn
17-09-2009, 04:35
دوستان میشه نحوه ی حل این نامعادله رو توضیح بدید :
[ برای مشاهده لینک ، لطفا با نام کاربری خود وارد شوید یا ثبت نام کنید ]
البته با توجه به نودار تابع متوجه میشیم که جوا نامعادله X>0.5 هست . اما روش حل رو دقیقا میخوام .
[ برای مشاهده لینک ، لطفا با نام کاربری خود وارد شوید یا ثبت نام کنید ] ......................................
یه سوال دیگه در باب نامعادلات
[ برای مشاهده لینک ، لطفا با نام کاربری خود وارد شوید یا ثبت نام کنید ]
مجموعه جواب این نامعادله کدام است :



1. X<-1
2. X>-3
3. X<-2
4. 3<X<-1-
..........................................

amintnt
17-09-2009, 11:23
دوستان میشه نحوه ی حل این نامعادله رو توضیح بدید :
[ برای مشاهده لینک ، لطفا با نام کاربری خود وارد شوید یا ثبت نام کنید ]
البته با توجه به نودار تابع متوجه میشیم که جوا نامعادله X>0.5 هست . اما روش حل رو دقیقا میخوام .
[ برای مشاهده لینک ، لطفا با نام کاربری خود وارد شوید یا ثبت نام کنید ] ......................................
یه سوال دیگه در باب نامعادلات
[ برای مشاهده لینک ، لطفا با نام کاربری خود وارد شوید یا ثبت نام کنید ]
مجموعه جواب این نامعادله کدام است :
1.X<-1
2.X>-3
3.X<-2
4.3<X<-1-
..........................................
در مورد اولی: اول از دو به توان X فاکتور بگیر... دو به توان x همواره مثبته... میمونه x+1/2>0 که جواب میشه x>- 0.5

در مورد دومی: سمت چپ برابره با x+1 به توان 3 ... از طرفین x+1 به توان دو رو ساده کن... چون x+1 به توان 2 مثبت هست، نیازی نیست در علامت نامساوی تغییری داده بشه... بعد از ساده کردن جواب میشه گزینه ی 3